You are on page 1of 92

8 JUNE

• It shall provide support for creation of world class


Q.1) Which of the following statement is/are most infrastructure along with common facilities and
appropriate regarding Scheme for Promotion of
Manufacturing of Electronic Components and amenities, including Ready Built Factory (RBF)sheds /
Semiconductors (SPECS): Plug and Play facilities for attracting major global
(a) It shall provide financial incentive of 25% on capital electronics manufacturers, along with their supply chains.
expenditure for the identified list of electronic goods.
(b) This Scheme shall extend an incentive of 4% to 6% on Q.2) With reference to the G7 artificial intelligence group,
incremental sales (over base year) of goods manufactured consider the following statements:
in India. 1. The US has recently joined G7 artificial intelligence
(c) It is a scheme for development of world class group, an international panel for setting ethical guidelines
infrastructure along with common facilities and amenities for the use of artificial intelligence.
through Electronics Manufacturing Clusters (EMCs). 2. The Group of Seven (G7) is an international
(d) It is a scheme to boost electric mobility and increase intergovernmental economic organization consisting of
the number of electric vehicles in commercial fleets. the seven large IMF- advanced economies in the world.
Which of the statements given above is/are correct?
Answer- (a) It shall provide financial incentive of 25% on (a) 1 only
capital expenditure for the identified list of electronic (b) 2 only
goods. (c) Both 1 and 2
Explanation- (d) Neither 1 nor 2
• The government has launched three incentive schemes
with a total outlay of about ₹48,000 crore to boost large- Answer : Both statement are correct.
scale manufacturing of electronics in the country. The US has joined G7 artificial intelligence group, an
• These three schemes are - international panel for setting ethical guidelines for the
1. Production Linked Incentive use of artificial intelligence. The Trump administration
2. Scheme for Promotion of Manufacturing of had earlier dismissed the idea.
Electronic Components and Semiconductors
(SPECS) ◎ What is it? è This Global Partnership on AI setup by
3. Modified Electronics Manufacturing Clusters the Group of Seven will guide the responsible adoption of
AI based on shared principles of “human rights, inclusion,
Production Linked Incentive diversity, innovation and economic growth.
• Targeted at mobile phone manufacturing and specified ◎ G7-Global Partnership on AI launched recently
electronic components. The government initially plans to through a virtual meeting between national technology
incentivise 10 firms — five global and five local. ministers.
• This Scheme shall extend an incentive of 4% to 6% on
incremental sales (over base year) of goods manufactured Q.3) Which of the following is NOT correct regarding
in India and covered under the target segments, to eligible Matterhorn mountain recently seen in news :
companies, for a period of five years subsequent to the 1. It is a mountain of the Andes.
base year. 2. It is situated on the border between Poland and
Scheme for Promotion of Manufacturing of Electronic Slovakia.
Components and Semiconductors (SPECS) Choose the correct code given below
• It shall provide financial incentive of 25% on capital (a) 1 only
expenditure for the (b) 2 only
identified list of electronic goods, i.e., electronic (c) Both 1 and 2
components, semiconductor/ display fabrication units, (d) Neither 1 nor 2
Assembly, Test, Marking and
Packaging (ATMP) units, specialized sub-assemblies and Answer (c) Both 1 and 2
capital goods for manufacture of aforesaid goods. • First statement is NOT correct because It is a mountain
of the Alps.
Modified Electronics Manufacturing Clusters (EMC 2.0) • Second statement is NOT correct because Matterhorn
Scheme mountain is situated in the Penninsular Alps on the border

1|Page
between Switzerland and Italy. Q.4) Which of the following pair is correctly matched:
Explanation - Bridge River
• It is seen recently in news because Indian Tricolour of 1. Daporijo bridge Subansiri River
more than 1,000 meters in size was projected on 2. Kasowal Bridge Chenab River
Matterhorn Mountain, Zermatt, Switzerland to express 3. Bogibeel bridge Brahmaputra River
solidarity to all Indians in the fight against COVID-19. Choose the correct code given below:
• Matterhorn is a mountain of the Alps. It is situated in the (a) 1 and 2 only
Pennine Alps on the border between Switzerland and (b) 3 only
Italy. (c) 1 and 3 only
• Its summit is 4,478 metres (14,692 ft) high, making it (d) All are matched correctly
one of the highest summits in the Alps and Europe. Answer (c) 1 and 3 only
Sometimes referred to as the Mountain of Mountains, the Explanation- Second match is INCORRECT because
Matterhorn has become an iconic emblem of the Alps in Kasowal Bridge is a long permanent bridge on the river
general. Ravi to connect the Kasowal enclave of Punjab along the
• The first ascent of the Matterhorn was in 1865 from Pakistan border to the rest of the country.
Zermatt by a party led by Edward Whymper; it ended Explanation -
when four of its seven members fell to their deaths on the 1) Daporijo bridge
descent. • The Border Roads Organization (BRO) constructed the
Daporijo bridge over Subansiri river in Arunachal
Pradesh in a record span of just 27 days.
• Significance ➔ The bridge is of utmost importance in
strategic connectivity as it links roads leading up to the
Line of Actual Control (LAC) between India and China.
• Subansiri River (gold river), originates in the Tibet
Plateau and enters India through Miri hills in Arunachal
Pradesh.
• It is the largest tributary of Brahmaputra River.
2) Kasowal Bridge
• It is a 484- metre long permanent bridge on the river
Ravi to connect the Kasowal enclave of Punjab along the
Pakistan border to the rest of the country. It is Built by the
Borders Roads Organization.
• The enclave was formed because it has the Ravi behind
it and the International Border ahead of it.
3) Bogibeel bridge
• Bogibeel river bridge is the longest rail-cum-road bridge
in India, measuring 4.94 kilometres over the Brahmaputra
river.
• It is Asia’s 2nd longest rail-cum-road bridge and has a
serviceable period of around 120 years.
• It is the 5th longest bridge in India after Bhupen
Hazarika Setu, Dibang River Bridge, Mahatma Gandhi
Setu and Bandra-Worli Sea Link.

Q.5) With reference to the PM SVANidhi, consider the


following statements:
1. It is a Special Micro-Credit Facility Scheme for
providing affordable loans to street vendors.
2. The vendors can avail a working capital loan of up to
Rs. 10,000, which is repayable in monthly instalments in
the tenure of one year.
Which of the statements given above is/are correct?

2|Page
(a) 1 only years since the MSME Development Act came into
(b) 2 only existence in 2006.
(c) Both 1 and 2 ◎ It has also been decided that the turnover with respect
(d) Neither 1 nor 2 to exports will not be counted in the limits of turnover for
any category of MSME units whether micro, small or
Answer: Both statement are correct. medium.
The Ministry of Housing and UrbanAffairs has
launched a micro-credit facility for street vendors
under the Swanidhi Scheme.
◎ What is it? è It is a special micro-credit facility scheme
to enable more than 50 lakh urban/peri-urban & rural
street vendors to restart their livelihoods affected due to
the lockdown.
◎ The loans are meant to help kick-start activity for
vendors who have been left without any income since the
lockdown was implemented on March 25. Q.7) Which of the following is/are benefits of National
◎ Duration è The scheme will be applicable till March Air Quality Index (AQI):
2022. 1. It helps in comparing air quality conditions at different
◎ Expected beneficiaries è This loan will be given to locations/cities.
those who run shops on the roadside, handcart or streetcar. 2. It helps in identifying faulty standards and inadequate
Fruit-vegetable, laundry, saloon and paan shops are also monitoring programmes.
included in this category. 3. It is especially useful for people suffering from
◎ Features of the Scheme è illnesses aggravated or caused by air pollution.
○ Under this, each of the street vendors will be given a
credit loan of Rs 10,000 which they can return as Choose the correct code given below:
monthly installments within a year. (a) 1 and 3 only
○ Those who repay their loans on (b) 2 and 3 only
time will get 7% annual interest (c) 1 only
as subsidy which will be transferred in their bank (d) All are correct
accounts.
○ The loans would be without collateral. Answer- d) All are correct
There will be no penalty on early Explanation: Lockdown due to COVID-19 pandemic has
repayment of the loan. pushed pollution levels in Delhi to a 5-year low. Across
India, the number of cities that recorded ‘good’ on the air
Q.6) With reference to the Micro, Small and Medium quality index jumped from 6 to 30 within a fortnight,
Enterprises (MSMEs), consider the following statements: according to data from the Central Pollution Control
1. A company with up to Rs 200 crore investments and up Board.
to Rs 500 crore turnover is classified as a medium Air Quality Index (AQI)
enterprise. • The AQI is an index for reporting daily air quality. It
2. Proposal for equity infusion of Rs. 50,000 crores for tells how much clean or polluted the air is.
MSMEs through fund of funds has also been approved. • The higher the AQI value, the greater the level of air
Which of the statements given above is/are correct? pollution and the greater the health Concern.
(a) 1 only • Research studies have attributed the key sources of
(b) 2 only PM2.5 in summer to be: dust and construction activities
(c) Both 1 and 2 (35%), transport sector (20%) and industry (20%).
(d) Neither 1 nor 2 • It is determined on the basis of concentration of 8
pollutants,
Answer: Statement 2 is correct. including Particulate Matter (PM 2.5, PM 10), sulphur
Union Cabinet has announced policy changes for the dioxide (SO2), nitrogen dioxide, carbon monoxide (CO),
MSME sector as part of the Atmanirbhar Bharat package. ozone (O3), ammonia (NH3) and lead (Pb).
Benefits of Air Quality Index (AQI)
◎ It may be noted that this revision was done after 14 •It helps in comparing air quality conditions at different

3|Page
locations/cities. sometimes seen in the news, is related to-
• It also helps in identifying faulty standards and (a) Protection of religious minorities in the world
inadequate monitoring programmes. (b) Promotion of Indian classical music and culture
• AQI helps in analysing the change in air quality amongst youth
(improvement or degradation). (c) Worldwide conservation of critically endangered
• AQI informs the public about environmental conditions. gangetic dolphins
It is especially useful for people suffering from illnesses (d) Awareness among refugees about human rights.
aggravated or caused by air pollution.
Answer : b
Q.8) Identify the wildlife sanctuary with the information Explanation
given below : SPIC MACAY (Society for the Promotion of Indian
1. It is located in the Dibrugarh and Tinsukia districts of Classical Music And Culture Amongst Youth) is a non-
Assam. political, nationwide, voluntary movement founded in
2. It is also known as the Jeypore Rainforest. 1977 by Dr Kiran Seth. Hence, option B is the correct
3. National Board for Wild Life (NBWL) has recently answer.
recommended coal mining in some part of the sanctuary.
4. It is a deciduous rainforest interspersed with semi- Q.10) Which of the following is/are salient features of
evergreen and lush green flora. Market Intervention Scheme?
1. It is a price support mechanism implemented on the
Choose the correct wildlife sanctuary: request of gram panchayats.
(a) Koundinya Wildlife Sanctuary 2. It is for procurement of perishable and horticultural
(b) Pakhui Wildlife Sanctuary commodities in the event of a fall in market prices.
(c) Dehing Patkai Wildlife Sanctuary 3. The Department of Agriculture & Cooperation is the
(d) Mehao Wildlife Sanctuary implementing agency of the
scheme.
Answer- (c) Dehing Patkai Wildlife Sanctuary
Explanation- Choose the correct code given below:
• Recently National Board for Wild Life (NBWL) has (a) 2 and 3 only
recommended coal mining in a part of an Dehing Patkai (b) 1 only
elephant reserve in Assam. (c) 1 and 3 only
• Location ➔ Dehing Patkai Elephant Reserve includes (d) 1 and 2 only
the Dehing Patkai Wildlife Sanctuary.
• The Dehing Patkai Wildlife Sanctuary is also known as Answer- (a) 2 and 3 only
the Jeypore Rainforest.
• It is located in the Dibrugarh and Tinsukia districts of Explanation: First statement is not correct because It is a
Assam. price support mechanism implemented on the request of
• Dehing is the name of the river that flows through this State Governments.
forest and Patkai is the hill at the foot of which the
sanctuary lies. Market Intervention Scheme
• Type of Fauna: Rare fauna found in the region include • Context: The Central government has asked the state
Chinese pangolin, flying fox, wild pig, sambar, barking governments to procure perishable agricultural and
deer, gaur, serow and Malayan giant squirrels. horticultural items from the farmers under the Market
• It is the only sanctuary in India which is home to seven Intervention Scheme.
different species of wild cats - tiger, leopard, clouded • It is a price support mechanism implemented on the
leopard, leopard cat, golden cat, jungle cat and marbled request of State Governments.
cat. • It is for procurement of perishable and horticultural
• Assamese macaque, a primate found in the forest, is in commodities in the event of a fall in market prices.
the red list of endangered species. • The Scheme is implemented when there is at least 10%
• Type of Flora: Dehing Patkai is a deciduous rainforest increase in production or 10% decrease in the ruling rates
interspersed with semi-evergreen and lush green flora. over the previous normal year.
• Its objective is to protect the growers of these
Q.9) SPIC MACAY's International Convention horticultural/agricultural commodities from making
distress sale in the event of bumper crop during the peak

4|Page
arrival period when prices fall to very low level. supporting, helping and handholding. It is a real one-stop-
• The Department of Agriculture & Cooperation is shop solution of MSME Ministry. This ICT based system
implementing the scheme. is set up to help the MSMEs in present difficult situation
and also to handhold them to become national and
international champions. As part of the system a network
9 JUNE of control rooms is created in a Hub & Spoke Model. The
Hub is situated in New Delhi in the Secretary MSME’s
Q.1) Which of the following would you accept most with office. The spokes are in the States in various offices and
reference to the Persian Gulf countries,: institutions of MSME Ministry. Hence, option (a) is the
1. The lands around the Persian Gulf are shared by eight correct answer.
countries namely, Bahrain, Iran,Iraq, Kuwait, Oman,
Qatar, Saudi Arabia, and the United Arab Emirates. Q.3) With reference to Tribals in India, Consider the
2. Iran and Iraq are not members of the Gulf following statements:
Cooperation Council (GCC). 1. Under Article 342, the President is empowered to
3. Out of Persian Gulf countries, only Iran, Iraq and declare communities as scheduled tribes.
Saudi Arabia are members of OPEC. 2. Tribal people constitute more than 20 % of the
Which of the statements given above is/are correct? nation's total population.
a. 1 and 2 only 3. Rajiv Gandhi National Fellowship Scheme is related
b. 3 only to the tribal student pursuing higher education.
c. 2 and 3 only 4. The XaXa committee was constituted to examine the
d. 1, 2 and 3 overall socio-economic conditions of the tribals.
Which of the statements mentioned above is/are correct?
Answer: (A) Only 1 and 2 are correct. a) 1 and 2 only
▪ The lands around the Persian Gulf are shared by eight b) 2 and 3 only
countries namely, Bahrain, Iran, Iraq, Kuwait, Oman, c) 1, 3 and 4 only
Qatar, Saudi Arabia, and the United Arab Emirates. d) 1, 2, 3 and 4
These all eight countries are members of the United
Nations. Hence, statement 1 is correct. Answer: (c)
▪ United Arab Emirates (UAE), Bahrain, Saudi Arabia, ▪ The President under Article 342 is empowered to
Oman, Qatar, Kuwait are members of the Gulf declare communities as scheduled tribes. While
Cooperation Council (GCC). Hence, statement 2 is Parliament by law can amend the list. Hence,
correct. statement 1 is correct.
▪ Out of Persian Gulf countries, Iran, Iraq, Kuwait, ▪ Tribal people constitute 8.6% of the nation's total
United Arab Emirates (UAE) and Saudi Arabia are population and not 15%. Hence, statement 2 is not
members of OPEC. Hence, statement 3 is not correct. correct.
▪ Rajiv Gandhi National Fellowship Scheme is related
Q.2) Creation and Harmonious Application of Modern to the tribal student pursuing higher
Processes for Increasing the Output and National Strength education. Hence, statement 3 is correct.
(CHAMPIONS), recently seen in news, aims to empower: ▪ The XaXa committee was constituted to examine the
a. Micro Small and Medium Enterprises overall socio-economic conditions of the tribals in
b. Large Edu-tech companies 2013 under the chairmanship of Prof. Virginius
c. International Automakers Xaxa. Hence, Statement 4 is correct.
d. Food Processing Industries
Q.4) Nagar Van scheme was recently seen in news,
Answer: (A) Micro Small and Medium Enterprises consider the following statements about it:
1. In the initial phase, under this program, forests will be
Prime Minister Modi launched the technology platform developed in 200 corporations and cities in the
CHAMPIONS to empower MSMEs. CHAMPIONS country across the country over the next five years.
stands for Creation and Harmonious Application of 2. The scheme will be, in part, paid for by the CAMPA
Modern Processes for Increasing the Output and National (Compensatory Afforestation Fund Act, 2016) funds.
Strength. The portal is basically for making the smaller Which of the statements given above is/are correct?
units big by solving their grievances, encouraging, a) 1 only
b) 2 only

5|Page
c) Both 1 and 2 research.
d) Neither 1 nor 2 • A genome is defined as an organism’s complete set
of Deoxyribose Nucleic Acid (DNA), including all of
Answer: (c) Both 1 and 2 its genes.
▪ Union Environment Minister launched the Urban • Each genome contains all of the information needed
Forest program (Nagar Van scheme) on the occasion to build and maintain that organism.
of World Environment Day. In the initial phase, under
this program, forests will be developed in 200 Q.6) With reference to the Cyclones, consider the
corporations and cities in the country across the following statements:
country over the next five years. 1. Cyclones formed in the Bay of Bengal side of the north
▪ The scheme will be, in part, paid for by the CAMPA Indian Ocean are more frequent and stonger than those on
(Compensatory Afforestation Fund Act, 2016) funds. the Arabian Sea side.
The forests were to come up either on existing forest 2. The relatively hot waters of the Arabian Sea discourage
land or on any other vacant land offered by urban the kind of very strong cyclones that are formed on the
local bodies. The Environment Minister highlighted Bay of Bengal side.
the success of Wajre Urban forest in the city of Pune, Which of the statements given above is/are correct?
Maharashtra which has been developed on a 40 acre a) 1 only
degraded land area. b) 2 only
▪ Hence both statements are correct. c) Both 1 and 2
d) Neither 1 nor 2
Q.5) Consider the following statements with respect Answer: a
to”Genome India”: Explanation
1. It aims to ultimately build a grid of the Indian reference • On the western coast of India, Cyclone Nisarga is
genome, to understand the genetic nature of Indo- headed towards the coastline of north Maharashtra
Aryan people in the Indian population. and south Gujarat.
2. It is being spearheaded (नेतत्ृ व) by the IISc’s Centre for • In strength and intensity, Cyclone Nisarga would be
Brain Research institute. much weaker than Cyclone Amphan that struck on
3. A genome is an organism’s complete set of DNA, May 20 and passed through West Bengal on its way
including all of its genes. to Bangladesh.
Which of the statement(s) given above is/are correct? • Cyclones formed in the Bay of Bengal side of the
a. 2 only north Indian Ocean are more frequent and stonger
b. 2 and 3 only than those on the Arabian Sea side.
c. 1 and 3 only • The relatively cold waters of the Arabian Sea
d. None of the above discourage the kind of very strong cyclones that are
formed on the Bay of Bengal side; Odisha and Andhra
Answer: b Pradesh face the brunt of these cyclones every
• Recently, Scientists at Genome India propose study year. Hence only 1 statements are correct
of COVID’s genetic dimension.
• The team wants to study and develop more insights Q.7) Which of the following is/are incorrect about the
into how this new viral disease, resistance to it and agriculture sector:
susceptibility might have a genetic component. 1. With the amendment to Essential Commodities Act,
• The Government of India has launched a gene- commodities like cereals, pulses, oilseeds, edible oils,
mapping project called “Genome India”. onion and potatoes will be removed from list of
• It aims to build a grid of the Indian “reference essential commodities.
genome”, to understand fully the type and nature of 2. Under 'The Farming Produce Trade and Commerce
diseases and traits that comprise the diverse Indian (Promotion and Facilitation) Ordinance, 2020',
population. farmers and traders will enjoy freedom of choice of
• This is spearheaded by the Centre for Brain sale and purchase of agri-produce.
Research at Bengaluru-based Indian Institute of Which of the pairs given above is/are correctly matched?
Science as the nodal point of about 20 institutions. a) 1 only
• Each of these institutions will do their bit in collecting b) 2 only
samples, doing the computations, and then the c) Both 1 and 2
d) Neither 1 nor 2

6|Page
Ocean. Hence, pair 4 is not correctly matched.
Answer: (d)
▪ The Union Cabinet in its meeting held on 3rd June, Q.9) Consider the following statements with respect to
2020 took several landmark decisions for helping TULIP Program
India’s farmers while also transforming the 1. It is a dedicated window to provide internship
agriculture sector. opportunities for fresh graduates in Urban Local Bodies
▪ Cabinet approved historic amendment to the Essential and in smart cities.
Commodities Act 2. It is an initiative of All India Council for Technical
▪ With the amendment to Essential Commodities Act, Education (AICTE), Ministry of HRD, and Ministry
commodities like cereals, pulses, oilseeds, edible oils, of Housing and Urban Development.
onion and potatoes will be removed from list of Which of the statement(s) given above is/are correct?
essential commodities. a. 1 only
▪ This will remove fears of private investors of b. 2 only
excessive regulatory interference in their business c. Both 1 and 2
operations. The freedom to produce, hold, move, d. Neither 1 nor 2
distribute and supply will attract private
sector/foreign direct investment into agriculture Answer: c
sector. • The Urban Learning Internship Program (TULIP) is
▪ But in situations such as war, famine, extraordinary a program for providing fresh graduates experiential
price rise and natural calamity, such agricultural learning opportunities in the urban sector.
foodstuff can be regulated. However, the installed • The platform is a dedicated window to internship
capacity of a value chain participant and the export opportunities in Urban Local Bodies/smart cities.
demand of an exporter will remain exempted from • The platform will aggregate opportunities from
such stock limit imposition so as to ensure that across India for graduates on a single platform and
investments in agriculture are not discouraged. allow them to apply for internships in the domain of
urban governance.
Q.8) Consider the following pairs: • It is an initiative of AICTE, Ministry of HRD, and
Strait : Connecting Water bodies Ministry of Housing and Urban Development.
1. Strait of Hormuz : Gulf of Persia and Gulf of Aden
2. Strait of Gibraltar : Red Sea and Atlantic Ocean Q.10) With reference to the ‘Social Stock Exchange’,
3. Strait of Malacca : Andaman Sea and South China consider the following statements:
Sea 1. It will promote investors to invest in select social
4. Cook Strait : Arctic Ocean and Pacific Ocean initiatives.
Which of the pairs given above is/are correctly matched? 2. It will function on the lines of major stock exchanges
a) 1 and 4 only like BSE and NSE.
b) 2 and 3 only Which of the statements given above is/are correct?
c) 3 only a) 1 only
d) 1, 2, 3 and 4 b) 2 only
c) Both 1 and 2
Answer: (c) d) Neither 1 nor 2
▪ Strait of Hormuz is a channel linking the Persian Gulf
(west) with the Gulf of Oman and the Arabian Sea Answer: (c) Both are correct.
(southeast). Hence, pair 1 is not correctly matched. ◎ It will function on the lines of major stock exchanges
▪ Strait of Gibraltar is a channel connecting the like BSE and NSE. However, the purpose of the Social
Mediterranean Sea with the Atlantic Ocean, lying Stock Exchange will be different – not profit, but social
between southernmost Spain and north western most welfare.
Africa. Hence, pair 2 is not correctly matched. ◎ Under the regulatory ambit of SEBI, a listing of social
▪ Strait of Malacca, waterway connecting the Andaman enterprises and voluntary organizations will be
Sea (Indian Ocean) and the South China Sea (Pacific undertaken so that they can raise capital as equity, debt or
Ocean). Hence, pair 3 is correctly matched. as units like a mutual fund.
▪ Cook Strait is a strait separating the North and South ◎ Why SSEs?
islands of New Zealand, extending northwest to ○ India needs massive investments in the
southeast from the Tasman Sea to the South Pacific

7|Page
coming years to be able to meet the human 1. The Inter-State Water Disputes Act, 1956 empowers
development goals identified by global bodies like the the Parliament to set up an ad hoc tribunal for the
UN. adjudication of an inter-state water dispute.
○ Currently, social enterprises are very active in India. 2. The decision of the tribunal would be final and
However, they face challenges in raising funds. binding on the parties to the dispute.
◎ Benefits Select the correct answer using the code given below.
○ There is a great opportunity to unlock funds from a) 1 only
donors, philanthropic foundations and CSR spenders, b) 2 only
in the form of zero-coupon zero principal bonds. c) Both 1 and 2
These bonds will be listed on the SSE. d) Neither 1 nor 2
○ At first, the SSE could become a repository of social
enterprises and impact investors. Answer: (b) 2 only
○ The registration could be done through a standard ▪ Article 262 of the Constitution provides for the
process. adjudication of inter-state water disputes. Parliament
may by law provide for the adjudication of any
10 JUNE dispute or complaint with respect to the use,
Q.1) About Lakshadweep group of islands, consider the distribution and control of waters of any inter-state
following: river and river valley.
1. They are made up of coral deposits. ▪ Under this provision, the Parliament has enacted two
2. Recently for the very first time scientists carried out laws [the River Boards Act (1956) and the Inter-State
genetic study on people of Lakshadweep Islands. Water Disputes Act (1956)].
3. 10 degree channel separates entire group of ▪ The River Boards Act provides for the establishment
Lakshadweep islands from Maldives. of river boards for the regulation and development of
Which of the statements given above is/are correct? Inter-state River and river valleys. A river board is
a) 2 and 3 only established by the Central government on the request
b) 3 only of the state governments concerned to advise them.
c) 1 and 2 only ▪ The Inter-State Water Disputes Act empowers the
d) All are correct Central government to set up an ad hoc tribunal for
the adjudication of a dispute between two or more
Answer: (c) Only 1 and 2 are correct. states in relation to the waters of an interstate river or
• Recently, For the very first time scientists carried out river valley. Hence, statement 1 is not correct.
genetic study on people of Lakshadweep Islands. It ▪ The decision of the tribunal would be final and
was conducted by a team of researchers at CSIR’s binding on the parties to the dispute. Neither the
(Council of Scientific & Industrial Research) Centre Supreme Court nor any other court is to have
for Cellular and Molecular Biology (CCMB). The jurisdiction in respect of any water dispute which may
authors studied major islands of Kadmat, Andorth, be referred to such a tribunal under this Act. Hence,
Chetlat, Agatti, Kalpeni, Bitra, Kiltan and Minicoy of statement 2 is correct.
Lakshadweep. Hence statement 2 is correct
• India has two major island groups: Andaman and Q.3) With reference to the Central Administrative
Nicobar, and Lakshadweep. Lakshadweep (and Tribunal (CAT), consider the following statements:
Minicoy) islands are situated in the Arabian Sea off 1. Its Mandate is to adjudicate disputes and complaints
the Malabar coast. They are scattered between 8°N- with respect to recruitment and conditions of service of
12°N latitude and 71°E -74°E longitude. The entire persons appointed to public services
group of islands is made of coral deposits. Hence 2. The Tribunal is guided by the principles of natural
statement 1 is correct. justice in deciding cases and is not bound by the
procedure, prescribed by the Civil Procedure Code.
• Nine Degree Channel separates Minicoy Island from
3. At present there are only 15 regular benches of Central
rest of Lakshadweep. Eight Degree Channel
Administrative Tribunal (CAT).
Separates entire group of Lakshadweep islands from
Select the correct answer using the code given below:
Maldives. Hence statement 3 is not correct.
a) 2 only
b) 1 and 2 only
Q.2) With reference to Inter-State Water Disputes, which
c) 3 only
of the following statements is/are correct?
d) 1, 2 and 3

8|Page
▪ Launched in 2017, PMVVY is a social security
Answer: (b) 1 and 2 only scheme for senior citizens intended to give an assured
▪ Union Minister of Personnel & Public Grievances minimum pension to them based on an assured return
inaugurated the 18th Bench of Central Administrative on the purchase price/ subscription amount.
Tribunal (CAT) for the Union Territories of Jammu ▪ LIC of India has been given the sole privilege to
and Kashmir and Ladakh. operate the scheme.
▪ Setting up of CAT Bench of Jammu to deal ▪ Eligibility: The subscriber must be a senior citizen
exclusively with service matters of government (above the age of 60 years). There is no maximum
employees would reduce the burden of various entry age for PMVVY scheme.
Courts. ▪ One can invest a maximum amount of ₹15 lakh under
▪ Earlier, in a notification issued under section 5(7) of PMVVY scheme and the tenure of the policy is set at
the Administrative Tribunals Act, 1985, the Central 10 years. The subscriber will receive a pension
Government has specified Jammu and Srinagar as the amount based on the premium given by an individual.
places at which the benches of the CAT shall ▪ Subscribers can draw a minimum pension of ₹1,000
ordinarily sit for the UT of Jammu and Kashmir and per month depending on the amount invested in the
UT of Ladakh. scheme. The maximum pension amount is limited at
▪ The Central Administrative Tribunal (CAT) had been ₹10,000 per month.
established under Article 323 - A of the Constitution. ▪ A subscriber can apply for loans after completion of
▪ Mandate is to adjudicate disputes and complaints with three years. The maximum loan that can be granted
respect to recruitment and conditions of service of shall be 75% of the purchase price. The interest on the
persons appointed to public services and posts in loan will be recovered from the pension payment that
connection with the affairs of the Union or other is being made.
authorities under the control of the Government.
▪ Under Section 17 of the Administrative Tribunal Act, Q.5) Which of the following statement(s) is/are correct
1985, the Tribunal has been conferred with the power regarding the Voluntary Retention Route (VRR) scheme?
to exercise the same jurisdiction and authority in 1. It is aimed at attracting foreign portfolio investment
respect of contempt of itself as a High Court. into the equity market.
▪ The Tribunal is guided by the principles of natural 2. Investments through this route is free of the
justice in deciding cases and is not bound by the regulatory norms applicable to foreign portfolio
procedure, prescribed by the Civil Procedure Code. investment without any condition.
▪ Salaries and Allowances and Conditions of Service of Select the correct option using the code given below.
the officers and other employees of the Tribunal are a) 1 only
specified by the Central Government. Hence only b) 2 only
statements 2 is correct. c) Both 1 and 2
d) Neither 1 nor 2
Q.4) Consider the following statements about Pradhan
Mantri Vaya Vandana Yojana (PMVVY): Answer: (d)
1. PMVVY is a social security scheme that offers a • VRR scheme is aimed at attracting long-term and
guaranteed payout of pension. stable FPI investments into debt markets not equity
2. LIC is the sole operator of PMVVY. markets. Hence, statement 1 is not correct.
3. Minimum age of eligibility for PMVVY is 40 years. • Investments through this route are free of the
Which of the statements given above is/are correct? regulatory norms applicable to foreign portfolio
a) 1 and 2 only investment only when investors maintain a minimum
b) 2 and 3 only value of their investments for a fixed period. Hence,
c) 3 only statement 2 is not correct.
d) 1, 2 and 3 only
Q.6) Which one of the following best describes the
Answer: (a) 1 and 2 only term “Syndemic”?
The Union Cabinet has extended the Pradhan Mantri a) It is a situation when two or more epidemics interact
Vaya Vandana Yojana (PMVVY) up to 31st March, 2023, to produce an increased burden of disease in a
for a further period of three years beyond 31st March, population.
2020. b) It is declared when a new disease for which people do
Details of PMVVY

9|Page
not have immunity spreads around the world beyond of good at abroad/ Price of good in the domestic
expectations. country)
c) It is a situation in which a disease affects 90% of the ▪ The real exchange rate measures prices abroad
region’s population and the people in the region learn relative to those at home. If the real exchange rate is
to live with the disease. equal to one, currencies are at purchasing power
d) It is declared when the presence or usual prevalence parity. This means that goods cost the same in two
of its infectious agent is constant within a given countries when measured in the same currency. For
geographical area or population group. instance, if a pen costs $4 in the US and the nominal
exchange rate is Rs 50 per US dollar, then with a real
Answer: (a) exchange rate of 1, it should cost Rs 200 in India.
▪ World Health Organization (WHO) has recently ▪ If the real exchange rises above one, this means that
announced that the Covid-19 would less likely to be goods abroad have become more expensive than
eliminated (i.e. may become endemic). goods at home. Similarly, if the real exchange goes
▪ The possibilities of Covid-19 being syndemic have below one, this means that goods at home have
been raised in the backdrop of the announcement. become more expensive than goods abroad i.e. one
rupee can buy higher amount of same product in the
Classification of Diseases based on spread and USA than in India.
occurrence ▪ The real exchange rate is often taken as a measure of
▪ Epidemic - An epidemic is a large outbreak, one that a country‘s international competitiveness.
spreads among a population or region. It is less severe
than pandemic due to a limited area of spread. Q.8) Consider the following statements about the United
▪ Endemic - A disease is called endemic when the Nations Environment Programme (UNEP):
presence or usual prevalence of its infectious agent is 1. It was founded as a result of United Nations Conference
constant within a given geographical area or on Environment and Development.
population group. 2. It provides the Champions of the Earth award.
▪ Pandemic - A pandemic is declared when a new 3. It is one of the implementing agencies for the Global
disease for which people do not have immunity Environment Facility.
spreads around the world beyond expectations. Which of the statements given above is/are correct?
▪ Syndemic - A Syndemic is a situation when two or a) 1 and 3 only
more epidemics interact synergistically to produce an b) 1 only
increased burden of disease in a population. c) 2 and 3 only
▪ The least possibility of elimination of Covid-19 d) 1, 2 and 3
pandemic and warning about the second wave of
Covid-19 infections worldwide have reinforced the Answer: (c) 2 and 3 only
presence of Covid-19 for the long term. ▪ 1 is not correct: United Nations Environment
▪ Meanwhile, the alarm is being raised about diseases Programme (UNEP) is an UN agency. It was founded
like dengue and malaria emerging with the upcoming as a result of the UN Conference on the Human
monsoon season in tropical South Asia. Environment (Stockholm Conference) in 1972, with
▪ Thus, there is a possibility that the world will face headquarter at Nairobi, Kenya. It coordinates UN‘s
increased burden of the diseases and thus the situation environmental activities, assisting developing
of syndemic. countries in implementing environmentally sound
policies and practices. The United Nations
Q.7) If the real exchange rate of Indian rupee with respect Conference on Environment and Development
to a US dollar is less than one, it indicates that (UNCED), also known as the Rio Summit or Earth
(a) One rupee would buy less amount of same product in Summit of 1992. The outcomes of this summit was:
USA than in India. Rio Declaration on Environment and Development,
(b) Indian economy is facing very high levels of inflation. Agenda 21, Convention on Biological Diversity and
(c) Indian rupee is significantly stronger with respect to UN Framework Convention on Climate Change
US dollars (UNFCCC).
(d) None of the above ▪ Statement 2 is correct: The Champions of the Earth
Award by United Nations Environment Programme.
Answer: (b) It is the United Nations highest environmental
▪ Real exchange rate= Nominal Exchange rate* (Price honour. The award celebrates outstanding figures

10 | P a g e
from the public and private sectors and from civil 1. It intends to address all the issues associated with the
society whose actions have had a transformative, freedom of Navigation.
positive impact on the environment. 2. The Convention was adopted on 1973 at International
▪ Statement 3 is correct: UNEP has also been active in Maritime Organization (IMO).
funding and implementing environment related 3. India is a party to the MARPOL convention and has
development projects. It has aided in the formulation ratified all of its six annexes.
of guidelines and treaties on issues such as the Which of the statement(s) given above is/are correct?
international trade in potentially harmful chemicals, a) 2 only
transboundary air pollution, and contamination of b) 1 and 3 only
international waterways. It is also one of several c) 2 and 3 only
Implementing Agencies for the Global Environment d) 1, 2 and 3
Facility (GEF) and the Multilateral Fund for the Answer (c) is correct 2 and 3 only
Implementation of the Montreal Protocol. UN • International Convention for the Prevention of
Environment is a founding partner of the SEED Pollution from Ships (MARPOL)
Awards, which support innovative, small-scale and • The MARPOL Convention was adopted on 2
locally driven entrepreneurs around the globe who November 1973 at IMO.
integrate social and environmental benefits into their • The Convention includes regulations aimed at
business models. preventing and minimizing pollution from ships -
both accidental pollution and that from routine
Q.9) Consider the following pairs: operations - and currently includes six technical
Island in News : Conflict Between Annexes.
1. Kuril island : Russia and Japan • The MARPOL Convention has six annexes (I to VI)
2. Paracel Islands : China & Vietnam and it deals with prevention of Pollution from ships
3. Spratly Islands : Britain & Argentina by Oil, Noxious liquid substances, Dangerous goods
Which of the above state is/are correctly matched? in packaged form, Sewage, Garbage and Air pollution
a) 1 only from ships respectively.
b) 1 and 2 only • India and MARPOL
c) 2 and 3 only • India has signed the International Convention for the
d) 1,2 and 3 Prevention of Pollution from Ships (MARPOL).
• India has ratified all of its annexes (Annexes I to VI).
Answer (b) is correct. 1 and 2 only
1. Kuril Island: All the island are currently under 11 JUNE
Russian jurisdiction. Japan claims the two
southernmost large island ( Iturup and Kunashir) Q. 1) Which of the following is true regarding Ways and
2. Both the Paracel and Spratly Island are associated Means Advances:
with South China Sea 1. They are permanent loan facilities provided by RBI to
3. The Paracel Islands are disputed among China the government.
,Taiwan and Vietnam 2. The government makes an interest payment to the
4. The Pratas islands are disputed between China and central bank when it borrows money and the rate of
Taiwan. interest is the same as the repo rate.
5. The Macclesfield Bank is disputed among the China Choose the correct code given below
, the Taiwan and the Philippines, with no land above [a] 1 only
sea level. [b] 2 only
6. The Scarborough Shoal is disputed among China , [c] Both 1 and 2
the Taiwan and the Philippines , with only rocks [d] Neither 1 nor 2
above sea level.
7. Over Spratly Island Brunei, China , Malaysia, Answer (b) 2 only is correct
Philippines and Vietnam are involved in a conflict. Explanation : First statement is NOT correct because
8. Falkland Island is the region of conflict between Ways and Means Advances are temporary loan facilities
Britain and Argentina provided by RBI to the government to enable it to meet
temporary mismatches between revenue and expenditure.
Q.10) With respect to MARPOL Convention, consider • Recently, The Reserve Bank of India has raised the limit
the following statements:

11 | P a g e
for short term credit that the government can borrow from National Skill Development Corporation (NSDC) is
the central bank. supporting the implementation of the project.
• The limits for this credit facility, known as ‘Ways and • The collected information will be shared with the
Means Advances’, has been raised sharply to Rs 1.2 lakh companies for suitable
crore for the first half of 2020-21. placement opportunities in the country.
• The increased limit comes at a time when government • SWADES Skills Card ➔ The returning citizens are
expenditure is expected to rise as it battles the fallout of a required to fill up an online SWADES Skills Card.
spreading Coronavirus. • The card will facilitate the returning citizens with
What are Ways and Means Advances? suitable employment opportunities through discussions
• They are temporary loan facilities provided by RBI to with key stakeholders including State Governments,
the government to enable it to meet temporary Industry Associations and Employers.
mismatches between revenue and expenditure.
• The government makes an interest payment to the National Skill Development Corporation (NSDC)
central bank when it borrows money. • Status ➔ NSDC is a not-for-profit public limited
• The rate of interest is the same as the repo rate, while company incorporated under section 25 of the Companies
the tenure is three months. Act, 1956 (corresponding to section 8 of the Companies
• The limits for WMA are mutually decided by the RBI Act, 2013).
and the Government of India. • It was set up by the Ministry of Finance as a Public
• The WMA scheme for the Central Government was Private Partnership
introduced on April 1, 1997. (PPP) model.
• WMA limits are if exceeded, is called overdraft.
• Share capital ➔ The Government of India through the
• Overdrafts are not allowed beyond 10 consecutive
Ministry of Skill Development & Entrepreneurship
working days.
(MSDE) holds 49% of the share capital of NSDC, while
• The interest rate on overdrafts would be 2 percent more
the private sector has the balance 51% of the share capital.
than the repo rate.
• Functions ➔ It aims to promote skill development by
Q.2) The Government of India has launched a initiative catalyzing creation
SWADES to conduct a skill mapping exercise of the of large, quality and for-profit vocational institutions.
returning citizens under which of the following
scheme/mission: Vandhe Bharat mission
[a] National Social Assistance scheme • It is the massive repatriation operation planned by the
[b] Skill India Mission Indian government to bring back stranded Indians in
[c] Atmanirbhar Bharat Abhiyan different parts of the world in the wake of the coronavirus
[d] Vande Bharat Mission crisis.
• This airlift by the MEA would be the largest since the
Answer (d) Vande Bharat Mission Persian Gulf War evacuation of over 170,000 people from
Explanation- Kuwait in 1990. Similar evacuations were also seen in
SWADES Initiative April 2015, when Operation Raahat was launched to
• The Government of India has launched a new initiative evacuate Indians from war-torn Yemen.
SWADES to conduct a skill mapping exercise of the • Citizens in distress will be brought back on commercial
returning citizens under the Vande Bharat Mission. aircraft and naval ships based on lists prepared by the
India’s embassies. Over ten lakhs Indians are expected to
• Objective ➔ To create a database of qualified citizens
return home, and more than two lakhs are from the UAE
based on their skill sets and
alone.
experience and fulfil demand of Indian and
• The Ministry of External Affairs has developed a
foreign companies.
dynamic online platform on which requests received by
• SWADES stands for Skilled Workers Arrival
Indian Missions from Indian nationals wishing to return
Database for Employment Support.
are regularly being uploaded.
• Bodies Involved ➔ This is a joint initiative of the
Ministry of Skill Development & Entrepreneurship, the Q.3) Consider the following statements about Swamitva
Ministry of Civil Aviation and theMinistry of External Yojana:
Affairs. 1. The scheme aims to revolutionise property record
• Implementation ➔ MSDE’s implementation arm maintenance in India.

12 | P a g e
2. The scheme is piloted by the Ministry of Housing and [b] 2 and 3 only
Urban Affairs. [c] 1 only
Which of the above statement is/are correct [d] 2 only
[1] 2 only
[2] 1 only Answer c) 1 only
[3] Both 1 and 2 Explanation: First statement is INCORRECT because
[4] Neither 1 nor 2 Arsenicum album is made by heating arsenic with
distilled water, a process repeated several times over three
Answer (b) 1 only is correct days.
Explanation- Second statement is NOT correct because • Arsenicum album 30, has become a subject of debate
The scheme is piloted by the Panchayati Raj ministry. after several states and AYUSH Ministry recommended it
for prophylactic (preventive) use against Covid-19.
Swamitva Yojana About Arsenicum album 30
• On Panchayati Raj Diwas , The Prime Minister of India • Arsenicum album is made by heating arsenic with
launched ‘Swamitva Yojana’ or Ownership Scheme to distilled water, a process repeated several times over three
map residential land ownership in the rural sector using days.
modern technology like the use of drones. • The health hazards of arsenic contamination in water are
• The scheme aims to revolutionise property record well known, long-term exposure to the metal can cause
maintenance in India. skin cancer, pulmonary and cardiovascular diseases.
• The residential land in villages will be measured using • The homoeopath drug has less than 1% arsenic and
drones to create a non-disputable record. Arsenicum Album is considered to correct inflammation
• Property card for every property in the village will be in the body.
prepared by states using accurate measurements delivered • Its uses ➔ Arsenicum album is used commonly by
by drone-mapping. These cards will be given to property homoeopaths to treat anxiety, restlessness, cold,
owners and will be recognised by the land revenue records ulcerations and burning pains.
department. • Significance in the Context of COVID 19 ➔ The
Ministry of Ayush recommended taking a three-day dose
Benefits of the scheme: on an empty stomach of Arsenicum album 30 as a
1) The delivery of property rights through an official prophylactic medicine against COVID 19 infections.
document will enable villagers to access bank finance
• Concerns ➔ The WHO neither has any guidelines nor
using their property as collateral.
any effective evidence on using Arsenicum album as
2) The property records for a village will also be
Covid-19 treatment.
maintained at the Panchayat level, allowing for the
collection of associated taxes from the owners. The
Q. 5) Identify the correct answer with the information
money generated from these local taxes will be used to
given below :
build rural infrastructure and facilities.
1. It is carved rock as well as a cultural heritage site in
3) Freeing the residential properties including land of title
Gilgit-Baltistan.
disputes and the creation of an official record is likely to
2. It carry carvings on them which are basically
result in appreciation in the market value of the properties.
inscriptions and images from the Prehistoric era.
4) The accurate property records can be used for
3. The site had many Buddhist shelter caves in ancient
facilitating tax collection, new building and structure
times and dates back to the 1st Millennium AD.
plan, issuing of permits and for thwarting attempts at
property grabbing.
Select the correct answer given below:
[a] Saptaparni Cave
Q.4) Which of the following is/are INCORRECT for
[b] Rock of Hunza
Arsenicum album 30:
[c] Pandavleni Caves
1. It is made in extremely low temperatures with distilled
[d] Guntupalle Buddhist Site
water.
2. It is a homoeopathic drug that has less than 1% arsenic.
Answer (b) Rock of Hunza
3. It is used to treat anxiety, restlessness, cold, ulcerations
Explanation-
and burning pains.
•India has reacted strongly to reports of vandalism and
Select the correct code given below:
defacement of ancient Buddhist rock carvings in Gilgit-
[a] 1 and 2 only

13 | P a g e
Baltistan under Pakistan’s control. • What has Russia done so far? ➔ Boom obstacles were
• The Buddhist sites in Hunza in Gilgit-Baltistan have placed in the river,
been desecrated by Islamists, who painted Pakistani flags but they were unable to contain the oil because of shallow
and slogans on the rock-cut art. waters. The
• The Buddhist residents, who found the paintings to be state of emergency declared would bring in extra forces
fresh, said that the vandalism by the Islamists was in and federal resources for the clean-up efforts.
retaliation to the recent protests against the Diamer What is the extent of the damage? ➔
Bhasha dam project, a joint Pakistan-China project. • Environmentalists have said the river would be difficult
to clean, given its
Rock of Hunza shallow waters and remote location, as well as the
• The Sacred Rock of Hunza is carved rock as well as a magnitude of the spill.
cultural heritage site in Gilgit-Baltistan in Pakistan
occupied Kashmir.
• History ➔ The carvings on the rock dates back to the 1st
Millennium AD.
• The rock is on the top of a hill which lies east to the
Hunza River.
• The Sacred Rock is divided into two portions, an upper
portion and a lower one.
• Both the portions carry carvings on them which are
basically inscriptions and images from the Prehistoric era.
• The site had many Buddhist shelter caves in ancient
times which later collapsed or fell over the time.
• The rock carvings and engravings in areas were part of
the Buddhist circle covering Ladakh and Tibet.

Q.6) The Ambarnaya recently seen in news is a river in:


[a] India
[b] China
[c] Russia
[d] Pakistan

Answer (c) Russia


Explanation-
• Russia has declared a state of emergency after a power
plant fuel leak in its Arctic region caused 20,000 tonnes
of diesel oil to escape into a local river, turning its surface
crimson red.
• What is it? ➔ The Ambarnaya River, into which the oil
has been discharged, is part of a network that flows into
the environmentally sensitive Arctic Ocean.
• The Ambarnaya is a river in Siberia which flows in a
northerly direction into Lake Pyasino. • The clean-up effort could take between 5-10 years.
• Emergency measures were announced within Russia’s
Krasnoyarsk Region, located in the vast and sparsely
populated Siberian peninsula.
• How did the leak happen? ➔ The thermoelectric power
plant at Norilsk is
built on permafrost, which has weakened over the years
owing to climate change. The power plant is located near
the Region’s Norilsk city, around 3000 km northeast of
Moscow.

14 | P a g e
Q. 7) Which of the following is/are features of Asteroids: [c] Name given to crater on Mercury
1. These are made up of metals and rocks and orbit the [d] Water Purification system
Sun.
2. They tend to have shorter and elliptical orbits. Answer (b) Mechanical ventilator
3. They produce a coma or tail atmosphere. Ruhdaar:
• It is a low-cost mechanical ventilator to fight against
Choose the correct code given below: COVID19.
[a] 2 and 3 only • It is developed by IIT Bombay team.
[b] 1 only • The cost of production of one ventilator is Rs 10,000.
[c] 1 and 3 only
[d] 1 and 2 only Q. 9) Which of the following statement is true regarding
LiDAR:
Answer (d) 1 and 2 only 1. It is a instrument principally consists of a laser, a
Explanation: Third statement is not correct because They scanner, and a specialized GPS receiver.
do not produce a coma or tail atmosphere. 2. It is a remote sensing method to measure ranges &
• Recently, the National Aeronautics and Space variable distances.
Administration (NASA) has announced that a giant Choose the correct code given below:
asteroid called as 163348 (2002 NN4), is expected to pass [a] 2 only
Earth on 6th June 2020. [b] 1 only
[c] Both 1 and 2
Asteroid [d] Neither 1 nor 2
• Asteroids are made up of metals and rocks and orbit the
Sun and are small bodies in the solar system. Answer (c) Both 1 and 2 are true
• They tend to have shorter and elliptical orbits. Explanation-
• They do not produce a coma or tail atmosphere. • Recently a U.K.-based team of archeologists has
• The asteroid belt is a torus-shaped region in the Solar continued its research over the Tamar Valley through
System, located roughly between the orbits of the planets LiDAR (Light Detection and Ranging) despite lockdown
Jupiter and Mars. due to Covid-19 in the country.
• Asteroids are minor planets, especially of the inner Solar • LiDAR (Light Detection and Ranging) is a remote
System. sensing method that uses light in the form of a pulsed laser
• The scientific interest in comets and asteroids is largely to measure ranges & variable distances.
due to their status as relatively unchanged remnant debris • These light pulses—combined with other data recorded
from the solar system formation process over 4.6 billion by the airborne system— generate precise, three-
years ago. Therefore, they offer clues about the chemical dimensional information about the shape of the Earth and
mixture from the planets formed. its surface characteristics.
• A LiDAR instrument principally consists of a laser, a
Near-Earth Objects (NEOs): scanner, and a specialized GPS receiver.
•NEOs are comets and asteroids pushed by the • Airplanes and helicopters are the most commonly used
gravitational attraction of nearby planets into orbits which platforms for acquiring LiDAR data over broad areas.
allow them to enter the Earth’s neighbourhood. • LiDAR is used for agriculture, hydrology, water
• These objects are composed mostly of water ice with management systems, geology-related applications, and
embedded dust particles. also used in archaeology.
NEOs occasionally approach close to the Earth as they
orbit the Sun. Tamar Valley
• NASA’s Center for Near-Earth Object Study (CNEOS) • Tamar valley is located in the south of England and is a
determines the times and distances of these objects, when rich archaeological landscape with many sites belonging
their approach to the Earth is close, through the Asteroid to the Iron Age and Roman era.
Watch Widget. • The area is a World Heritage Site due to its historic
mining activities.
Q.8) Ruhdaar recently seen in news is a:
Q. 10) Q. Regarding Major Atmospheric Cerenkov
[a] Robot to combat COVID-19 Experiment Telescope(MACE), which of the following is
[b] Mechanical ventilator correct:

15 | P a g e
1. It is an Imaging Atmospheric Cerenkov telescope iCommit initiative is centred around the idea of building
(IACT) located in Himachal Pradesh. an energy resilient future.
2. It is the highest (in altitude) telescope in the World and
was built by Electronics Corporation of India. About iCommit initiative
• Recently, the iCommit initiative was launched on the
Choose the correct code given below- occasion of the
World Environment day.
(a)1 only • Aim ➔ To call upon all stakeholders and
(b) 2 only individuals to continue moving towards energy
(c)Both 1 and 2 efficiency, renewable energy and sustainability to create
(d) Neither 1 nor 2 a robust and resilient energy system for India in the future.
• Nodal agency ➔ The initiative is driven by Energy
Answer (b) 2 only is correct Efficiency Services Limited (EESL), under the
Explanation- administration of the Ministry of Power, Government of
• First statement is NOT correct because Major India.
Atmospheric Cerenkov Experiment Telescope (MACE) • The prerequisite for this goal is to create a flexible and
is an Imaging Atmospheric Cerenkov telescope (IACT) agile power system.
located at Hanle, Ladakh, India.
• Significance ➔ A healthy power sector can help the
• It is the highest (in altitude) Cerenkov telescope in the
nation in meeting the objective of
World and was built by Electronics Corporation of India,
energy access and security for all.
Hyderabad, for the Bhabha Atomic Research Centre.
• It seeks to bring together a diverse spectrum of
• It was assembled at the campus of Indian Astronomical
government and private players to build a new energy
Observatory at Hanle.
future for India.
• Operational since 2016, it is remotely operated and runs
on solar power.
Q.2) Which of the following issues will be examined by
• The telescope is the second-largest gamma ray telescope
JAYA JAITLY TASK FORCE:
in the world and will help the scientific community
[a] Lowering Maternal Mortality Rate
enhance its understanding in the fields of astrophysics,
[b] Workplace discrimination to women
fundamental physics, and particle acceleration
[c] Finding affordable and quality child care
mechanisms.
[d] Limited access of education to girls
• The largest telescope of the same class is the 28-metre-
diameter High Energy Stereoscopic System (HESS)
Answer (a) Lowering Maternal Mortality Rate
telescope being operated in Namibia.
Explanation-

JAYA JAITLY TASK FORCE


12 JUNE • The Women and Child Development Ministry has
constituted a task force to examine issues related to
Q. 1) What is the significance of iCommit initiative -age of motherhood,
launched on the occasion of the -lowering Maternal Mortality Rate and -improvement of
World Environment day: nutritional levels.
1. It will help to increase the forest cover and also protect • Objective ➔The task force will be headed by Jaya Jaitly
the existing forest land. and it will submit its report by 31st July of next month.
2. It will help the nation in meeting the objective of • The Terms of Reference - To examine the issues related
energy access and security for all. to Infant Mortality Rate, Maternal Mortality Rate, Total
Choose the correct code given below Fertility
[a] 2 only Rate, Sex Ratio at Birth, Child Sex Ratio (CSR) and any
[b] 1 only other issues pertaining to health and
[c] Both 1 and 2 nutrition
[d] Neither 1 nor 2
Q.3) Consider the following statements about MAITREE
Answer (a) 2 only is correct program:
Explanation : First statement is NOT correct because 1. It is a part of the Australia-India Energy Dialogue that

16 | P a g e
discuss bilateral engagement on energy and resources. overall public health.
2. This program aimed at accelerating the adoption of • Most buildings in India are not equipped to establish and
cost-effective energy efficiency as a standard practice maintain healthy indoor air quality and need to be
within buildings. upgraded.
Which of the above statement is/are correct
[1] 1 only Energy Efficiency Services Limited (EESL)
[2] 2 only What is it? ➔ Energy Efficiency Services Limited
[3] Both 1 and 2 (EESL), is a joint venture of four National Public Sector
[4] Neither 1 nor 2 Enterprises– NTPC Limited, Power Finance Corporation
Ltd (PFC), REC and POWERGRID, and was set up under
Answer (b) 2 only is correct Ministry of Power.
Explanation- First statement is NOT correct because The • Aim ➔ It aims to create market access for efficient and
Market Integration and Transformation Program for future-ready transformative solutions that create a win-
Energy Efficiency (MAITREE) is a part of the US-India win situation for every stakeholder.
bilateral Partnership between the Ministry of Power and • It is working towards mainstreaming energy efficiency
USAID. and is implementing
the world’s largest energy efficiency portfolio in the
Healthy and Energy Efficient Buildings” initiative country.
• The Energy Efficiency Services Limited (EESL) has • It helps in implementing programmes such as
launched the “Healthy and Energy Efficient Buildings” UnnatJyoti by Affordable LED for All (UJALA), Smart
initiative that will pioneer Meters, etc.
ways to make workplaces healthier and greener.
• Nodal agency ➔ The initiative has been launched by Q.4) Which of the following is/are INCORRECT about
EESL in partnership with the U.S. Agency for Payments Infrastructure Development Fund (PIDF):
International Development’s (USAID) MAITREE 1. It has a corpus of Rs. 500 crore and will encourage
program. acquirers to deploy point of sale (PoS) infrastructure, both
• As part of this initiative, EESL has taken the physical and digital.
leadership by being the first to implement this framework 2. The fund will be managed and administered by an
in its own offices. advisory council.
• This initiative addresses the challenges of retrofitting 3. The PIDF will receive recurring
existing buildings and air conditioning systems so that contributions to cover operational expenses from card-
they are both healthy and energy-efficient. issuing banks and card networks.
• It will pave the way for other buildings to take
appropriate steps to be healthy and energy-efficient. Select the correct code given below:
[a] 1 and 2 only
MAITREE Program [b] 2 and 3 only
What is it? ➔ The Market Integration and Transformation [c] 1 only
Program for Energy Efficiency (MAITREE) is a part of [d] 2 only
the US-India bilateral Partnership between the Ministry of
Power and USAID. Answer d) 2 only
• Aim ➔ It is aimed at accelerating the adoption of cost- Explanation: Second statement is INCORRECT because
effective energy The fund will be governed
efficiency as a standard practice within buildings and through an advisory council but it will be
specifically focuses on managed and administered by the RBI.
cooling.
Significance of the initiative ➔ Payments Infrastructure Development Fund (PIDF)
• Poor air quality has been a concern in India for quite • Recently, the Reserve Bank of India (RBI) has
some time and has become more important in light of the announced the creation of a Rs. 500-crore Payments
COVID pandemic. Infrastructure Development Fund (PIDF).
• As people return to their offices and public spaces, • Objective ➔ This fund has been created to
maintaining good indoor air quality is essential for encourage acquirers to deploy point of sale
occupant comfort, well-being, productivity and the (PoS) infrastructure, both physical and digital,

17 | P a g e
in tier-3 to tier-6 centres and north eastern • It refers to a substantial increase in the degree of
states. extinction or when the Earth loses more than three-
• Governance ➔ The fund will be governed quarters of its species in a
through an advisory council but it will be geologically short period of time.
managed and administered by the RBI. • An extinction event is a widespread and rapid decrease
• Corpus ➔ It has a corpus of Rs. 500 crore in which the in the biodiversity on Earth.
RBI has made an initial contribution of Rs. 250 crore. The • Such an event is identified by a sharp change in the
remaining will come from the card-issuing banks and card diversity and abundance of multicellular organisms. It
networks operating in the country. occurs when the rate of extinction increases with respect
to the rate of speciation.
• Recurring contributions ➔ The PIDF will also receive
• According to researchers at National Academy of
recurring contributions to cover operational expenses
Sciences of the United States of America(PNAS), the
from card-issuing banks and card networks. RBI will also
ongoing sixth mass extinction may be one of the most
contribute to yearly shortfalls, if necessary.
serious environmental threats to the persistence of
• Need ➔ Most of the PoS terminals in the country are civilisation.
concentrated in tier 1 and 2 cities because of the high cost
• Sixth Extinction ➔ The sixth mass extinction which is
of merchant acquisition and merchant terminalisation.
ongoing is referred to as the Anthropocene extinction or
•Benefits ➔ Holocene extinction.
○ It will give a push to digital payments across India. • The research has claimed that this extinction is human-
○ Reduce demand for cash over time. caused and is more immediate than climate destruction.
The rate of extinction of species is also estimated at 100
Q. 5) With reference to Mass extinction, Consider the to 1,000 times higher than natural background rates.
following statements:
1. It refers to the extinction of a large number of species Q.6) World Water Development Report (WWDR) is a
within a relatively short period of geological time. flagship report of:
2. The ongoing sixth mass extinction is referred to as the [a] Global water intelligence
Anthropocene extinction. [b] International water association
Which of the following statements is/are correct: [c] UN-Water
[a] 2 only [d] Stockholm International Water Institute
[b] 1 only
[c] Both 1 and 2 Answer (c) UN-Water
[d] Neither 1 nor 2 Explanation-
• The 2020 edition of the World Water Development
Answer (c) Both 1 and 2 Report (WWDR 2020) entitled ‘Water and Climate
Explanation- Change’ aims at helping the water community to tackle
Mass extinction the challenges of climate change and informing the
climate change community about the opportunities that
improved water management offers in terms of adaptation
and mitigation.
• The United Nations World Water Development Report
(WWDR) is a global report that provides an authoritative,
comprehensive assessment of the world’s freshwater
resources.
• It is produced annually by the World Water Assessment
Programme and released by UN-Water.
• The Report examines the ways that the world’s water
resources are being managed and the varied water
• The extinction of a large number of species within a problems that different regions of the world are
relatively short period of geological time, due to factors experiencing.
such as a catastrophic global event or widespread • It takes a close look at growing water problems
environmental change that occurs too rapidly for most worldwide, such as access to clean water and sanitation,
species to adapt. and the cross-cutting issues which affect them, such as

18 | P a g e
energy, climate change, agriculture, and urban growth. body, or local, affecting a region of the body.
• The Report also offers recommendations on how • Hypoxia is a condition in which the body or a region of
freshwater resources could be managed more sustainably. the body is deprived of adequate oxygen supply at the
• The content of the Report comes from the coordinated tissue level.
efforts of 26 UN agencies that make up UN-Water, • Normal arterial oxygen is approximately 75 to 100
working with governments, international organizations, millimetres of mercury (mm Hg), and normal pulse
non-governmental organizations and other stakeholders. oximeter readings usually range from 95 to 100 per cent.
Values under 90 per cent are considered low.
Q. 7) GARUD PORTAL recently making news is • Symptoms ➔ When levels fall below 90%, patients
launched by: could begin experiencing lethargy, confusion or mental
[a] Ministry of Science and Technology disruptions because of
[b] Ministry of Earth Sciences insufficient quantities of oxygen reaching the brain.
[c] Ministry of Mines and Energy • Levels below 80% can result in damage to vital organs.
[d] Ministry of Civil Aviation
About silent Hypoxia
Answer (d) Ministry of Civil Aviation • What it is? ➔ is a form of oxygen deprivation that is
Explanation: harder to detect than
• Ministry of Civil Aviation and Directorate General of regular hypoxia. Patients appear to be less in distress.
Civil Aviation have launched the GARUD portal to fast- • Covid pneumonia, a serious medical condition found in
track exemptions of coronavirus-related drone operations. severe Covid-19
• GARUD Acronym ➔ Government Authorisation for patients, is preceded by silent hypoxia.
Relief Using Drones. • Symptoms ➔ In many cases, Covid-19 patients with
• Significance ➔ To help state-owned entities in seeking silent hypoxia did not
exemption from central government for coronavirus exhibit symptoms such as shortness of breath or coughing
related drone operations. until their oxygen fell to acutely low levels, at which point
• Also help in addressing the challenges posed by there was a risk of acute respiratory distress (ARDS) and
COVID-19 and will remain in force until further orders. organ failure.
Reasons ➔ The reason why people are left feeling
Q.8) Which of the following statements appropriately breathless is not because of the fall
define Hypoxia often seen in news: in oxygen levels itself but due to the rise in carbon dioxide
levels that occur
[a] It is a condition in which the brain swells with fluid. at the same time, when lungs are not able to expel this gas
[b] It is a condition in which the body or a region of the efficiently.
body is deprived of adequate oxygen supply at the tissue • A medical device called a pulse oximeter can be used in
level. the early detection of silent hypoxia.
[c] Swelling and inflammation in the main passages that
carry air to the lungs. About Pulse Oximeter
[d] It is a condition where there is a collection of fluid
between the lung and the chest wall.
• Application ➔ It is a test used to measure the oxygen
level (oxygen saturation) of the blood.
Answer (b) It is a condition in which the body or a region
of the body is deprived of adequate oxygen supply at the • How ➔ The device measures the saturation of oxygen
tissue level. in red blood cells (RBCs) and can be attached to a
Hypoxia person’s fingers, toes, nose, feet, ears or forehead.
• Recently medical practitioners have reported a condition • Benefits ➔ The method is easy and painless and the
called silent or happy hypoxia, in which Covid-19 device can be reused or disposed of after use.
patients have extremely low blood oxygen levels, yet they
do not show signs of breathlessness. Q. 9) Which of the following statement is true regarding
• What is it? ➔ Hypoxia is a condition wherein there is INS Jalashwa:
not enough oxygen available to the blood and body 1. It is an amphibious transport dock currently in service.
tissues. 2. It is the only Indian naval ship to be acquired from the
• Hypoxia can either be generalised, affecting the whole United States.

19 | P a g e
Choose the correct code given below: 156,000 rials per dollar.
[a] 2 only • Iran’s weak currency and high inflation have led to
[b] 1 only sporadic street protests.
[c] Both 1 and 2
[d] Neither 1 nor 2 13 JUNE

Answer (c) Both 1 and 2 are true Q. 1) Which of the following is true regarding Jeypore
Explanation- Dynasty:
INS Jalashwa 1. It was a kingdom in the Kalinga region ruled by the
• INS Jalashwa is an amphibious transport dock currently kings of the Suryavansh dynasty.
in service with the Indian Navy. • INS Jalashwa is the only 2. It was a princely estate under British rule from 1776 to
Indian naval ship to be acquired from the United States. 1947 and was the largest of all princely states and estates
• It is based in Visakhapatnam under the Eastern Naval in the Madras Presidency.
Command. Choose the correct code given below
Operation Samudra Setu [a] 2 only
• What it is? ➔ The Indian Navy launched ‘Operation [b] 1 only
Samudra Setu’ (Sea Bridge) as a part of national effort to [c] Both 1 and 2
repatriate Indian citizens from overseas. [d] Neither 1 nor 2
• Indian Naval Ships Jalashwa and Magar are enroute to
the port of Male, Republic of Maldives etc to commence Answer (c) Both are 1 and 2 correct
evacuation operations.
Explanation :
Q. 10) Which of the following pair given below is
correctly matched? Recently 30 confirmed cases of chikungunya, has been
reported from the Raygada District.
Currency Country • Rayagada is a district in southern Odisha, a state in India,
1) Yuan Japan which became a separate district in October 1992.
2) Rial Iran • Its population consists mainly of tribes, primarily the
3) Dirham United Arab Emirates Khonds and the Soras.
• It was founded by Maharajah Vishwanath Dev Gajapati
Choose the correct code given below of the Suryavansh dynasty of Jeypore.
[a] 2 only • The district is reportedly rich in bauxite and silicon.
[b] 2 and 3 only
[c] 1 and 3 only Jeypore Dynasty
[d] 3 only • The Kingdom of Jeypore or Jeypore Kalinga Rajya was
a kingdom in the Kalinga region ruled by the kings of the
Answer (b) 2 and 3 only Suryavansh dynasty, who trace their origins from ‘Dev’
Suryavanshis of Jammu & Kashmir and thus claim to be
Explanation- Yuan/Renminbi is the currency of China. the descendants of Rama.
• Japanese Yen is the currency of Japan. • It was an independent princely state from 1443 to 1775
and a princely estate under British rule from 1776 to 1947.
TOMAN Currency • It had three capitals under different rulers: Nandapur,
• The Iran’s parliament has passed a bill allowing the Rayagada and Jeypore, the most prominent.
government to the replacement of the national currency, • It was the largest of all princely states and estates in the
the rial with another basic unit of currency called the Madras Presidency, covering an area of 31,079 km2 in
toman. 1880 and 36,259 km2 in 1925.
• Changes ➔ The plan is set to effectively cut four zeros • Despite being the largest kingdom, it never had the
from its currency, making each toman worth 10,000 rials status of a princely state because it lost the crucial
under the new system. feudatories of Bobbili, Vizianagaram, Kalahandi, Salur,
•Why ➔ There was a sharp fall in the value of the Srungavarapukota, Nowagarh, Khariar and
currency as a result of crippling U.S. sanctions. Gajapathinagaram, which clinched independence from
• As a result, the Iranian currency was trading at about Jeypore and were later recognized as princely estates

20 | P a g e
under British Rule. • No new funds are allocated to this Yojana and funds may
• The last ruler of the kingdom was King Vikram Dev IV be raised through
when the estate was dissolved in 1947 and merged into
the Union of India. 1) Funds from existing schemes, such as the Indira Awas
Yojana, Pradhan Mantri Gram Sadak Yojana, Mahatma
Chikungunya Gandhi National Rural Employment Guarantee Scheme,
• Chikungunya virus is an arbovirus that belongs to the and Backward Regions Grant Fund, etc.,
genus Alphavirus. 2) The Member of Parliament Local Area Development
• It is a viral disease that is spread by the bite of Aedes Scheme (MPLADS),
aegypti and Aedes albopictus mosquitoes. 3)The gram panchayat’s own revenue,
• Symptoms usually appear within a week of infection, It 4)Central and State Finance Commission Grants, and
causes fever and severe joint pain. 5) Corporate Social Responsibility funds.
• Other symptoms include muscle pain, headache, nausea,
fatigue and rash. Q.3) Consider the following statements about Aditya
• Joint pain is often debilitating and can vary in duration. Ferry:
• The disease is most prevalent in in Africa, South- Asia 1. It is India's first solar-powered ferry and the largest
and India. solar-powered boat in India.
2. The vessel was designed and built by NavAlt Solar and
Q. 2) No new funds are allocated to Sansad Adarsh Gram Electric Boats.
Yojana and funds may be raised through: Which of the above statement is/are correct
1. The Member of Parliament Local Area Development [1] 1 only
Scheme. [2] 2 only
2. The gram panchayat’s own revenue [3] Both 1 and 2
3.Central and State Finance Commission Grants [4] Neither 1 nor 2
4.Corporate Social Responsibility funds
Choose the correct code given below Answer (c) Both 1 and 2
[a] 1 and 2 only Explanation-
[b] 2 and 4 only • India’s first solar-powered ferry ‘Aditya’ is among 12
[c] 1 and 4 such ferries around the globe that have been shortlisted
[d] 1,2,3 and 4 for the Gustave Trouvé
Award.
Answer (d) 1,2,3 and 4 • What is it? ➔ The electric ferry boat is operated by the
Explanation- Kerala State Water Transport Department.
Sansad Adarsh Gram Yojana • It is India's first solar-powered ferry and the largest
• It is a rural development programme broadly focusing solar-powered boat in India.
upon the development in the villages which includes • The vessel was designed and built by NavAlt Solar and
social development, cultural development and spread Electric Boats in Kochi, India.
motivation among the people on social mobilization of the • NavAlt is a joint venture firm between Navgathi Marine
village community. Design and Constructions, Alternative Energies (France)
• The programme was launched on the birth anniversary and EVE Systems (France).
of Jayaprakash Narayan, on 11 October 2014. • As a public transport solar-electric ferry, it has proven
• The distinct feature of this Yojana is that it is its performance by transporting 11 lakh passengers and
(a) Demand driven clocking a distance of 70,000 km, without a single drop
(b) Inspired by society of fossil fuel.
(c) Based on people's participation. • The zero-pollution vehicle resulted in saving over 1 lakh
litres of diesel and thus reducing carbon emission.
• Under this Yojana, each Member of Parliament needs to • The per km energy cost of Aditya is low, and the ferry
choose one village each from the constituency that they normally operates 22 trips a day, covering a total of 66
represent, except their own village or their in-laws village km, carrying 75 passengers per trip.
and fix parameters and make it a model village by 2019. • It needs just ₹180 per day in energy cost, compared to
• Villages will be offered smart schools, universal access about ₹8,000 for a diesel-run ferry of similar size.
to basic health facilities and Pucca housing to homeless
villagers.

21 | P a g e
3)Tocilizumab
Gustave Trouvé Award • This is an immunosuppressant commonly used to treat
for rheumatoid arthritis.
• The Gussies electric boat award is a global award • This is an expensive drug, that costs Rs 40,000-60,000
instituted after Gustave Trouvé, a French electrical per dose, it is used as a preventive against ventilator
engineer and pioneer in electric cars and boats. requirement, government hospitals are giving it free.
• Gustave Trouvé was a prolific inventor with over 75 • Tocilizumab is manufactured by Roche Pharma, and
patents. marketed by Cipla.
• This is the inaugural edition of the award in the • In India it is sold under the brand name Actemra.
remembrance of the inventor & aimed to promote
innovative excellence with electric boats. 4)Itolizumab
• This drug is commonly used for the skin disorder
Q.4) Which of the following is/are used for the treatment psoriasis, rheumatoid arthritis, multiple sclerosis, and
of COVID-19 in India- autoimmune disorders. In India, Biocon launched it in
1. Remdesivir 2013.
2. Favipiravir • It is being trailled in Mumbai and Delhi on moderately
3. Hydroxychloroquine to severely ill Covid patients. Initial results will come by
4. Itolizumab July.
5. Tocilizumab
Select the correct code given below: 5)Hydroxychloroquine
[a] 1 and 3 only • This antimalarial drug is a subject of debate over its
[b] 1,3 and 4 only efficacy against Covid.
[c] 2 and 3 only • India is the largest producer of this drug.
[d] 1,2 ,3,4 and 5 • Doctors use HCQ use in Covid patients with symptoms
Answer d) 1, 2 ,3,4 and 5 as mild as headache, fever, body pain, and even in
critically ill patients.
Explanation: • ICMR guidelines recommend low doses for nine days.
List of Treatments for COVID-19 used in India • This drug is helping in faster recovery, but that is just
preliminary assessment.
1)Remdesivir • Side effects of the drug includes chaotic heartbeat.
• Remdesivir, an antiviral drug first developed for treating
Ebola in 2014, is one of the possible Covid-19 treatments 6)Doxycycline + ivermectin
being investigated in the WHO’s Solidarity Trial. • Doxycycline is an antibiotic used to fight infection in the
• It inhibits viral replication in the body. urinary tract, eye, or respiratory tract.
US National Institutes of Allergies and Infectious • Ivermectin is an anti-parasite drug for treatment of
Diseases released preliminary trial results showing scabies, head lice, and filariasis.
recovery time of Covid patients given remdesivir • The combination is used to treat Covid patients with
improved from 15 to 11 days. acute symptoms.
• The Drug Controller General of India on June 1
approved a five-day regime of remdesivir. 7)Ritonavir + lopinavir
• Doctors are currently prescribing it for moderately to • These antivirals are commonly used to treat HIV
severely ill patients. patients.
Remdesivir costs Rs 10,000-20,000 per vial. • They are being investigated in the Solidarity Trial.
• Some studies suggest they reduce mortality risk in
2)Favipiravir Covid-19 patients, while others have found no major
•Favipiravir is an antiviral given to inhibit viral improvement.
replication, It is used as an anti-influenza drug. • Over a dozen manufacturers supply ritonavir and
• First manufactured by Japan’s Fujifilm Toyama lopinavir in India.
Chemical Ltd, it is manufactured in India by Glenmark • Doctors sometimes use the combination for severely ill
Pharmaceutical and Strides Pharma. patients.
• It is being used for moderately symptomatic to severely
ill Covid patients, but access is not easy. 8)Plasma therapy
• Basis of the Therapy: The convalescent plasma therapy

22 | P a g e
seeks to make use of the antibodies developed in the participants and art enthusiasts to create and learn from
recovered patient against the coronavirus. practicing artists.
• The whole blood or plasma from such people is taken, • Activities ➔ The programme includes online workshops
and the plasma is then injected in critically ill patients so sessions on painting, sculpture, printmaking and indrajaal
that the antibodies are transferred and boost their fight (an interdisciplinary creative workshop).
against the virus. • The exhibition of selected artworks from the program
• This is meant for critical patients with low oxygen will be displayed onSo’ham, the cultural media platform
saturation levels, or those suffering a cytokine storm. of NGMA.
• Patients who have recovered from severe Covid-19
donate their plasma, which is then injected into other About So’ham
critical patients to boost their immunity. • It is the first cultural media platform of India, under the
• A protocol approved by ICMR is used to select which banner of NGMA.
patient is best suited for plasma therapy. • Aim ➔ The platform aims to develop a dialogue
• Preference is given to those at risk of cytokine storm, between NGMA, artists and art enthusiasts.
extreme breathlessness with severe pneumonia. • It is inspired by the Vedic philosophy of So’ham, which
stands for one's identity and its connection to the universe.
Facts about plasma
• Plasma is the largest part of your blood. It, makes up About NGMA
more than half (about 55%) of its overall content.
• Established ➔ NGMA was established in 1954, at the
• When separated from the rest of the blood, plasma is a
Jaipur House, NewDelhi.
light yellow liquid.
• Plasma carries water, salts and enzymes. • Parent Organization ➔ It is run and administered as a
• The main role of plasma is to take nutrients, hormones, subordinate office to the Ministry of Culture, Government
and proteins to the parts of the body that need it. of India.
• Cells also put their waste products into the plasma. • The NGMA has two branches, one at Mumbai and the
• The plasma then helps remove this waste from the body. other at Bengaluru.
• Blood plasma also carries all parts of the blood through • Objective ➔ One of its objectives is to acquire and
your circulatory system. preserve works of modern art from the 1850s onward.

Q. 5) With reference to So’ham, Consider the following Q.6) Environmental performance index is released by
statements: Yale and Columbia universities
1. It is the first cultural media platform of India, under in collaboration with :
banner of Ministry of Culture. [a] World Wide Fund for Nature
2. It is inspired by the Vedic philosophy of So’ham, which [b] United Nations Environment Programme
stands for one's identity and its connection to the universe. [c] Worldwatch Institute
Which of the following statements is/are correct: [d] World Economic Forum
[a] 2 only
[b] 1 only Answer (d) World Economic Forum
[c] Both 1 and 2 Explanation-
[d] Neither 1 nor 2 • 12th edition of the biennial Environment Performance
Index (EPI Index 2020) has been released.
Answer (b) 2 only • Released by ➔ Yale and Columbia universities in
Explanation- First statement is NOT correct because It is collaboration with the World Economic Forum(WEF).
the first cultural media platform of India, under the banner • The index is released biennially (once every two years).
of National Gallery of Modern Art (NGMA). • The EPI measures the environmental
performance of 180 countries.
About NAIMISHA 2020 • Indicators ➔ It considers 32 indicators of environmental
• The National Gallery of Modern Art (NGMA), New performance, giving a snapshot of the 10-year trends in
Delhi will organize online NAIMISHA 2020- Summer environmental performance at the national and global
Art Program from 8th June 2020 to 3rd July 2020, to reach levels.
its audiences during the Covid-19 outbreak. • Top ranks ➔ Denmark was ranked first
• What is it? ➔ It is an initiative to provide a chance to followed by Luxembourg and Switzerland.

23 | P a g e
• Performance of India ➔ India secured 168th from the Jurassic era, a vermicomposting unit, an
rank. The country scored 27.6 out of 100 in interpretation center, and a state-of-the-art weather
the 2020 index. station.
• India’s rank was 177 (with a score of 27.6 out • The Park has 40 unique sections having 479 rare plant
of 100) in 2018. species of cactus, medicinal herbs, different types of trees,
• All South Asian countries except Afghanistan etc.
are ahead of India in the ranking. • The various species of plants have been brought to the
• Remarks for India ➔ The report indicated that black park from diverse terrains like Niti Mana Valley and even
carbon, carbon dioxide emissions and greenhouse from some glaciers around Kedarnath.
emissions per capita increased in 10 years. • Niti Mana Valley is located near the India Tibetan
• India needs to re-double national sustainability efforts Border in Chamoli district of Uttarakhand.
on all fronts, • The plant species in the biodiversity park is divided into
according to the index. spiritual & religious, scientific, human health, and
• It needs to focus on a wide spectrum of sustainability aesthetic value sections.
issues, with a high- priority to critical issues such as air • The spiritual section has trees that find mention in holy
and water quality, biodiversity and climate change. scriptures like Guru Granth Sahib, Quran, Bible, and
others.
• It showcases the different kinds of soil found in various
topographies of Uttarakhand — alpine, bhabhar, sub-
mountainous, mountainous, tertiary, loam, terai.

Yamuna biodiversity park


• This park was in news because of the locusts attack and
its impact on Delhi as it has a large green area as well –
providing foraging material for the locusts. Delhi has
around 22% of its area under green cover.
• It located on Yamuna river front is a 9770 hectares
biodiversity area in Delhi, India.
• It is developed by Delhi Development Authority (DDA)
with the technical help of Centre for Environmental
Q. 7) Consider the following pair: Management of Degraded Ecosystems (CEMDE),
University of Delhi.
1) Haldwani - Uttar Pradesh Bio-Diversity Park • It serves as an ideal alternative habitat for migratory and
2) Dibru-Saikhowa - Assam National Park resident bird species.
3) Yamuna - Bihar Biodiversity Park • It also is designed to conserve the wild genetic resources
of agricultural crops and enhance groundwater recharge
Which of the following pair given above is correctly and augment freshwater availability.
matched?
[a] 2 only Q.8) Consider the following statements regarding UNCS:
[b] 2 and 3 1) The United nation Security Council consists of 10 non-
[c] 1 and 3 permanent members and five permanent members.
[d] 3 only 2) It is the only UN body with the authority to issue
binding resolutions to member states.
Answer (d) 2 only Choose the correct answer from the code given below
Explanation:
• Haldwani Bio-Diversity Park is in Uttrakhand [a] 2 only
• Yamuna biodiversity park is in Delhi [b] 1 only
[c] Both 1 and 2
Haldwani Bio-Diversity Park [d] Neither 1 nor 2
• Recently, Uttrakhand opened its biggest biodiversity
park in Haldwani on the World Environment Day . Answer (c) Both 1 and 2
• Inside the Park, there are thematic gardens, a soil Explanation-
museum, species of plants, lichens, mosses and algae • Recently, India has released a campaign brochure ahead

24 | P a g e
of the vote for the non-permanent member of the United • Ocean currents are primarily horizontal water
Nations Security Council (UNSC) in 2021-22. movements.
• An ocean current flows for great distances and together
About UNSC they create the global conveyor belt, which plays a
• What is it? ➔ The UNSC is one of the six principal dominant role in determining the climate of many of
organs of the United Nations and is charged with the Earth’s regions.
maintenance of international peace and security. • More specifically, ocean currents influence the
• Powers ➔ Its powers include the establishment of temperature of the regions through which they travel.
peacekeeping operations, the establishment of • For example, warm currents traveling along more
international sanctions, and the authorization of military temperate coasts increase the temperature of the area by
action through Security Council resolutions. warming the sea breezes that blow over them.
• It is the only UN body with the authority to issue binding • Perhaps the most striking example is the Gulf Stream,
resolutions to member states. which makes northwest Europe much more temperate
than any other region at the same latitude.
• Members ➔ The Security Council consists of fifteen
• Surface oceanic currents are sometimes wind driven and
members. Russia, the United Kingdom, France, China,
develop their typical clockwise spirals in the northern
and the United States—serve as the body’s five
hemisphere and counter-clockwise rotation in the
permanent members.
southern hemisphere due to imposed wind stresses.
• These permanent members can veto any substantive
• Deep ocean currents are driven by density and
Security Council resolution, including those on the
temperature gradients.
admission of new member states or candidates for
Secretary-General.
Q. 10) Lima, Peru, where the climate is cooler, being sub-
• The Security Council also has 10 non-permanent
tropical, than the tropical latitudes in which the area is
members, elected on a
located, due to the effect of the:
regional basis to serve two-year terms.
• The body’s presidency rotates monthly among its
[a] Labrador Current
members.
[b] Humboldt Current
[c] West wind drift
Q. 9) Which of the following factor/factors influence a
[d] Canary current
current's direction and strength:
1. wind
Answer (b) Humboldt Current
2. Coriolis effect
Explanation
3. Temperature and salinity differences
• Peru Current, also called Humboldt Current is a current
Choose the correct code:
of the southeast Pacific Ocean.
(a) 3 only
• It is a cold, low-salinity ocean current that flows north
(b) 1 and 3 only
along the western coast of South America.
(c) 1 and 2 only
• The zone where Peru Cold current meets the warm
(d) 1, 2 and 3
equatorial ocean waters is an important fishing zone.
• The Humboldt Current is a highly productive ecosystem.
Answer: (d) 1, 2 and 3
It is the most productive eastern boundary current system.
Explanation-
It accounts for roughly 18-20% of the total worldwide
About Ocean Currents
marine fish catch.
•An ocean current is a continuous, directed movement of
• The Humboldt Current is named after the Prussian
sea water generated by a number of forces acting upon the
naturalist Alexander von Humboldt.
water, including
• The Humboldt has a considerable cooling influence on
a) wind,
the climate of Chile, Peru and Ecuador.
b) the Coriolis effect,
• It is also largely responsible for the aridity of Atacama
c) breaking waves,
Desert in northern Chile and coastal areas of Peru and also
d) cabbeling, and
of the aridity of southern Ecuador.
e) temperature and salinity differences.
• Depth contours, shoreline configurations, and
interactions with other currents influence a current's
direction and strength.

25 | P a g e
15 JUNE

Q. 1) With reference to Shapes of economic recovery,


Consider the following statements:
1. U-shaped recovery is the most-
optimistic scenario in which the economy
quickly rises after an economic crash.
2. Z-shaped recovery is a scenario in which the economy,
after falling, struggles around a low growth rate for some
time, before rising gradually to usual levels.
Which of the following statements is/are correct
(a) 2 only
(b) 1 only
(c) Both 1 and 2
(d) Neither 1 nor 2

Answer (d) Neither 1 nor 2


Explanation :
• Z-shaped recovery ➔ It is the most- optimistic scenario
in which the economy quickly rises after an economic
crash.
• U-shaped recovery ➔ It is a scenario in which the
economy, after falling, struggles around a low growth rate
for some time, before rising gradually to usual levels.

Other Shapes of economic recovery


• V-shaped recovery ➔ A V-shaped rebound would bring
a sharp dip in economic activity matched by an equally
brisk surge back to normality.
• W-shaped recovery ➔ A W-shaped recovery is a
dangerous creature. In this, growth falls and rises, but falls
again before recovering, thus forming a W-like chart. The
double-dip depicted by a W- shaped recovery can be due
to the second wave of the pandemic.
• L-shaped recovery ➔ In this, the economy fails to regain
the level of GDP even after years go by. The shape shows
that there is a permanent loss to the economy’s ability to
produce.
• J-shaped recovery ➔ The J-shaped recovery is a
somewhat unrealistic scenario, in which growth rises
sharply from the lows much higher than the trend-line and
stays there.

Q. 2) Which of the following statement is correct about


Bharat Stage Norms:
1. These are standards instituted by the Government of
India in 2000 to regulate emission of air pollutants from
motor vehicles.
2. The standards and the timeline for implementation of
norms are set by the
Ministry of Petroleum and Natural Gas

26 | P a g e
Choose the correct code given below 2. This gas is not poisonous but if inhaled in large
(a) 2 only amounts it may be fatal.
(b) 1 only Which of the above statement is/are correct
(c) Both 1 and 2 (a) 1 only
(d) Neither 1 nor 2 (b) 2 only
(c) Both 1 and 2
Answer (b) 1 only (d) Neither 1 nor 2
Explanation- Second statement is NOT CORRECT
because The standards and the timeline for Answer (c) Both 1 and 2
implementation are Explanation-
set by the Central Pollution Control Board(CPCB) under • Recently there was leakage of Styrene gas from LG
the Ministry of Polymers' plant owned by South Korean electronics giant
Environment, Forest and Climate Change. LG, located at RRV Puram near Gopalapatnam, about 15
• Recently, The Ministry of Road Transport and kms from the coast city.
Highways (MoRTH) has issued an order mandating a
coloured strip to identify four-wheeled BSVI vehicle. About Styrene Gas
• The order mandates a strip of green colour of 1 cm width • What is it? ➔ It is a flammable liquid
on top of the existing sticker carrying details of • Uses ➔ used in the manufacturing of
registration for BS-VI vehicles of any fuel type i.e. for polystyrene plastics, fiberglass, rubber, and
petrol or CNG which have a light blue colour sticker and latex.
a diesel vehicle which is of orange colour. • It is also found in vehicle exhaust, cigarette
• These stickers will now have a green strip of 1 cm on smoke, and in natural foods like fruits and
top for BS VI, as mandated. vegetables.
• When exposed to styrene ➔ Short-term exposure to the
Bharat stage norms
substance can result in respiratory problems, irritation in
• These are standards instituted by the Government of
the eyes, irritation in the mucous membrane, and
India in 2000 to regulate emission of air pollutants from
gastrointestinal issues.
motor vehicles. They are based on the European(EURO)
• Long-term exposure could drastically affect the central
emission standards.
nervous system and
• Implementation ➔ The standards and the timeline for lead to other related problems like peripheral neuropathy.
implementation are It could also lead to cancer and depression in some cases.
set by the Central Pollution Control Board(CPCB) under
• Symptoms ➔ Symptoms include headache, hearing
the Ministry of
loss, fatigue, weakness, difficulty in
Environment, Forest and Climate Change.
concentrating etc.
• The BS-VI emission standards, which have been
• "This gas is not poisonous and hence there is nothing to
mandated w.e.f. 1st April,
worry to that extent. If inhaled in large amounts it may be
2020, provide for stricter and cleaner emission norms.
fatal. Gas by itself gives you irritation and causes
• Benefits of BS-VI Standards ➔ Particulate matter(PM) irritation in mucus membrane.
→ BS-VI norms brings down the emission of particulate • Water acts as an antidote to styrene and those with
matter by almost 80% in case of petrol as well as diesel exposure should douse themselves with water and drink a
power plants. lot of water.
• Sulphur content → Sulphur traces in BS-VI fuel is five
times lower (10 ppm) as compared to sulphur traces in Q.4) Which of the following is/are true about Sarfaesi
BS-IV fuel (50 ppm). Act:
• Nitrogen Oxide Content → Nitrogen oxide level for BS- 1. It promotes the setting up of asset reconstruction
VI diesel engines and petrol engines will be brought down companies (ARCs) and asset securitization companies
by 70% and 25% respectively. (SCs).
2. It allows banks and other financial institution to auction
Q.3) Consider the following statements about Styrene residential or commercial properties to recover loans.
gas: 3. The law does not apply to loans below ₹100,000 or
1. It is a flammable liquid used in the manufacturing of where remaining debt is below 20% of the original
polystyrene plastics, fiberglass, rubber, and latex. principal.

27 | P a g e
Select the correct code given below: also its owners.
(a) 1 and 3 only • Services ➔ These banks provide a wide range of regular
(b) 2 only banking and financial services.
(c) 1 and 2 only • Co-operative Banks are broadly classified into Urban
(d) 1, 2 and 3 and Rural co-operative banks based on their region of
operation.
Answer (d) 1, 2 and 3 • Who oversees these banks? ➔
Explanation: - Registered under the States Cooperative Societies Act.
• The Securitization and Reconstruction of Financial - They also come under the regulatory ambit of the
Assets and Enforcement of Security Act of 2002 Reserve Bank of India
(SARFAESI Act) applies to cooperative banks as well, a
• Sificance of Cooperative Banks ➔ It has an extensive
five-judge Constitution bench has ruled.
branch network all over the country, making credit easily
• Banks utilize Sarfaesi Act as an effective tool for bad
available even to rural areas. It accounts for 67 per cent of
loans (Non-Performing Asset) recovery and is effective
total rural credit.
against secured loans.
• It provides support to small and marginal farmers for
• It promotes the setting up of asset reconstruction
buying inputs, storage and marketing assistance.
companies (ARCs) and asset securitization companies
Q.5) With reference to India’s first online waste exchange
(SCs) to deal with NPAs accumulated with the banks and
platform, Consider the following statements:
financial institutions.
1. Its purpose is to ensure safe disposal of toxic
• The Securitisation and Reconstruction of Financial
wastes with the help of monitoring, scrutinizing and
Assets and Enforcement of Securities Interest Act, 2002
auditing of the waste.
(also known as the SARFAESI Act) is an Indian law.
2. It will be implemented by Ministry of Environment,
• It allows banks and other financial institution to auction
Forest and Climate Change.
residential or commercial properties (of Defaulter) to
recover loans.
Which of the following statements is/are correct:
• It provides the bank with the power to take over and
(a) 1 only
auction authority of residential or commercial properties,
(b) 2 only
to recover the loan.
(c) Both 1 and 2
• The first asset reconstruction company (ARC) of India,
(d) Neither 1 nor 2
ARCIL, was set up under this act.
• The law does not apply to unsecured loans, loans below
Answer (a) 1 only
₹100,000 or where remaining debt is below 20% of the
Explanation- Second statement is NOT correct because It
original principal.
will be implemented by Andhra Pradesh Environment
• This law allowed the creation of asset reconstruction
Management Corporation(EMC).
companies (ARC) and allowed banks to sell their non-
performing assets to ARC's.
About India’s first online waste exchange platform
• Banks are allowed to take possession of the collateral
• Andhra Pradesh(AP) government has launched
property and sell it without the permission of a court.
country’s first online Waste Exchange Platform for
• Under this act secured creditors (banks or financial
industrial waste.
institutions) have many right for enforcement of security
interest under section 13 of SARFAESI Act, 2002. If • Purpose ➔ To ensure safe disposal of toxic
borrower of financial assistance makes any default in wastes with the help of monitoring, scrutinizing and
repayment of loan or any installment and his account is auditing of the waste.
classified as Non performing Asset by secured • Besides, the platform will also encourage proper
creditor,then secured creditor may require before expiry utilisation of waste and promote 6Rs — reduce, reuse,
of period of limitation by written notice. recycle, refurbish, redesign and re- manufacturing of
waste items.
About Co-operative Banks
Q.6) MARPOL Convention recently seen in news is a:
• What it is? ➔ Co-operative banks are financial entities
(a) Human rights convention
established on a co-operative basis and belonging to their
(b) Marine environmental convention
members.
(c) Cultural and natural heritage convention
• This means that the customers of a co-operative bank are
(d) Arms control convention

28 | P a g e
an antimicrobial and washable face mask ‘NSafe’, which
Answer (d) Marine environmental convention is reusable up to 50 launderings, thus greatly cutting down
Explanation- the cost of use.
• The Ministry of Shipping has informed about the steps • It is a triple-layered product consisting of ➔
taken for prevention and control of pollution arising from 1. Inner hydrophilic layer for comfort,
ships in the sea and in the inland waterways under the 2. Middle layer having antimicrobial activity and
MARPOL Convention(Convention for Prevention of 3. Outer most layer having water and oil repellent
Marine Pollution). behaviour.
• What is it? ➔ MARPOL is the main international • Efficiency ➔ The mask has 99.2% bacterial filtration
convention aimed at the prevention of pollution from efficiency along with breathability and splash resistance.
ships caused by operational or accidental causes. • It is believed to be the first fabric based antimicrobial
• MARPOL is short for International Convention for the face mask launched in India.
Prevention of Pollution from Ships and 73/78 short for the
years 1973 and 1978) is one of the most important Q.8) Spiders building their webs on plants is an example
international marine environmental conventions. of:
• It was developed by the International Maritime (a) Commensalism
Organization with an objective to minimize pollution of (b) Parasitism
the oceans and seas, including dumping, oil and air (c) Amensalism
pollution. (d) Mutualism
• Timeline ➔ The Protocol of 1978 was
adopted in response to a number of tanker accidents in Answer (a) Commensalism
1976–1977. Explanation - Commensalism describes a relationship
• The current convention is a combination of 1973 between two living organisms where one benefits and the
Convention and the 1978 Protocol, which entered into other is not significantly harmed or helped. Hence spiders
force on 2 October 1983. building their webs on plants is an example of
• As of January 2018, 156 states are parties to the Commensalism where spider get benefit but there is no
convention, being flag states of 99.42% of the world's significant harm on plant.
shipping tonnage.
• All ships flagged under countries that are signatories to About Symbiosis
MARPOL are subject to its requirements, regardless of • Symbiosis is any type of a close and long-term
where they sail and member nations are responsible for biological interaction between two different biological
vessels registered on their national ship registry. organisms, be it mutualistic, commensalistic, or parasitic.
• India Status ➔ India is a signatory to MARPOL. 1) Mutualism or interspecies reciprocal altruism is a long-
• Functions ➔ The Convention includes regulations term relationship between individuals of different species
aimed at preventing and minimizing pollution from ships where both individuals benefit. For example, Coral reefs
– both accidental pollution and that from routine are the result of mutualisms between coral organisms and
operations – and currently includes six technical Annexes. various types of algae which live inside them.
• It has six annexes (I to VI) and it deals with prevention 2) Commensalism describes a relationship between two
of (1) Pollution from ships by Oil, (2) Noxious liquid living organisms where one benefits and the other is not
substances, (3) Dangerous goods in packaged form, (4) significantly harmed or helped.
Sewage, (5) Garbage and (6) Air pollution from ships 3) Parasitism is a symbiotic relationship between species,
respectively. where one organism, the parasite, lives on or in another
organism, the host, causing it some harm, and is adapted
Q. 7) ‘NSafe’ often seen in news is a: structurally to this way of life.
(a) Face Mask 4) Amensalism is an asymmetric interaction where one
(b) Hand Sanitizer species is harmed or killed by the other, and one is
(c) Personal Protective Equipment kit unaffected by the other.
(d) Ventilator

Answer (a) Face Mask


Explanation:
• An IIT Delhi startup ‘Nanosafe Solutions’ has launched

29 | P a g e
Q. 9) In the map given below, Which of the following lake • Lake Superior is the second-largest lake in the world by
of North America is being marked with the arrow: area, and the largest freshwater lake by surface area in
North America.

(a) Lake Erie


(b) Lake Ontario
(c) Lake Huron
(d) Lake Michigan

Answer: (a) Lake Erie

Q. 10) Consider the following statements regarding Great


Lakes of North America:
1) These are a series of interconnected saltwater lakes.
2) Lakes Michigan and Huron join at the Straits of
Mackinac.

Choose the correct answer from the code given below


(a) 2 only
(b) 1 only 16 JUNE
(c) Both 1 and 2
(d) Neither 1 nor 2 Q. 1) With reference to Coral Triangle Initiative on Coral
Answer (a) 2 only Reefs, Fisheries, and Food Security (CTI-CFF) , Consider
Explanation- First statement is NOT correct because the the following statements:
Great Lakes are a series of interconnected freshwater 1. It is a multilateral partnership of three countries
lakes which connect to the Atlantic Ocean through the working together to sustain extraordinary marine and
Saint Lawrence Seaway. coastal resources.
2. The Coral Triangle countries support people-centered
About Great lakes of North America biodiversity conservation, sustainable development,
• They comprise lake Superior, Huron, Michigan, Erie, poverty reduction and equitable benefit sharing.
and Ontario in the order of largest to smallest. Which of the following statements is/are correct
• Hydrologically, there are only four lakes, because lakes (a) 2 only
Michigan and Huron join at the Straits of Mackinac. (b) 1 only
• The lakes form the Great Lakes Waterway. (c) Both 1 and 2
• The Great Lakes are the largest group of freshwater (d) Neither 1 nor 2
lakes on Earth by total area, and second-largest by total
volume, containing 21% of the world's surface fresh water Answer (a) 2 only
by volume. Explanation : First statement is NOT correct because
• Due to their sea-like characteristics (rolling waves, Coral Triangle Initiative is a multilateral partnership of
sustained winds, strong currents, great depths, and distant six countries (Indonesia, Malaysia,Philippines, Papua
horizons) the five Great Lakes have also long been New Guinea, Solomon Islands, Timor-Leste).
referred to as inland seas.
More to Know

30 | P a g e
•The Coral Triangle Day 2020 was observed recently. gases into space over the span of one Martian year.
• The Coral Triangle Day is an annual event started by
Coral Triangle Initiative(CTI) since 2012 in conjunction Q.3) Recently the Brown Rock Chat has almost
with the World Oceans Day on June 8. disappeared from the heart
• Aim ➔ To celebrate and raise awareness of of the National Capital, visible only on the outskirts. With
the ocean conservation and protection especially of the reference to this, Consider the following statements about
Coral Triangle. Brown Rock Chat:
• Coral Triangle Initiative(CTI) ➔ It is a multilateral 1. It is nearly endemic to India and found mainly in
partnership of six countries (Indonesia, Malaysia, northern and central India.
Philippines, Papua New Guinea, Solomon Islands, Timor- 2. It is listed as vulnerable as per IUCN Red List of
Leste). Threatened Species.
Which of the above statement is/are correct
• Coral Triangle ➔ It is a vast ocean expense that
(a) 1 only
geographically spreads across 6 countries in Asia and the
(b) 2 only
Pacific: Indonesia, Malaysia, Philippines, Papua New
(c) Both 1 and 2
Guinea, Solomon Islands, and Timor Leste.
(d) Neither 1 nor 2
• Through the CTI-CFF, the Coral Triangle countries
support people-centered biodiversity conservation,
Answer (a) 1 only
sustainable development, poverty reduction and equitable
Explanation- Second statement is NOT correct because It
benefit sharing.
is listed as ‘Least Concern’ as per IUCN Red List of
• The CTI-CFF seeks to address both poverty reduction
Threatened Species.
through economic development, food security,
sustainable livelihoods for coastal communities and
About Brown Rock chat
biodiversity conservation through the protection of
• It is also called as Indian Chat which is a bird
species, habitats and ecosystems.
species of the family Muscicapidae.
Q. 2) Which of the following statement is correct about • Distribution ➔ It is found mainly in northern and central
Hope Mars Mission: India.
1. It is the first uncrewed, interplanetary satellite • It is nearly endemic to India, distributed north of the
spearheaded by the Canadian Space Agency. Narmada, west to Gujarat and east to Bengal bordered on
2. It will be the first probe to provide a complete picture the north by the Himalayas.
of the Martian atmosphere and its layers. • It is commonly found in agricultural fields, in buildings,
Choose the correct code given below and suburban areas.
(a) 1 only • It is often found on old buildings and rocky areas.
(b) 2 only • It is protected under Schedule IV of the Wildlife
(c) Both 1 and 2 Protection Act, 1972.
(d) Neither 1 nor 2

Answer (b) 2 only

Explanation-
• First statement is NOT CORRECT because The Hope
Mars Mission, also called the Emirates Mars Mission,
spearheaded by the United Arab Emirates Space Agency.
• It is the first planetary science mission led by an Arab- Q.4) Which of the following is/are an indicator of QS
Islamic country. World University rankings :
• It is the first uncrewed, interplanetary satellite of the 1. Academic reputation
UAE. 2. Faculty/student ratio
• It will be the first probe to provide a complete picture of 3. Salary given to Faculty
the Martian atmosphere and its layers when it reaches the Select the correct code given below:
red planet's orbit in 2021. (a) 2 and 3 only
• It will help answer key questions about the global (b) 2 only
Martian atmosphere and the loss of hydrogen and oxygen (c) 1 and 2 only

31 | P a g e
(d) 1, 2 and 3 versa, as assigned by the Customs’ automated system.
• Turant Customs is a mega reform for the ease of doing
Answer (c) 1 and 2 only business.
Explanation: Salary given to faculty is not an indicator in • The start of Turant Customs at Bengaluru and Chennai
QS World University rankings will be the first phase of the All India roll out which would
get completed by 31st December this year.
About QS World University rankings • The first phase will cover imports of Mechanical,
• 17th edition of the QS World University rankings has Electrical and Electronics machineries at the ports,
been released recently. airports and ICDs of Bengaluru and Chennai.
• Published by ➔ They are published annually by global • Benefits ➔ Turant Customs will benefit the importers
higher education consultancy Quacquarelli Symonds. by eliminating routine interface with the Customs officers
• Parameters ➔ It is based on six indicators: and providing uniformity in assessment across the
1)academic reputation, country.
2)employer reputation,
3)citations per faculty, Central Board of Indirect Taxes and Customs
4) faculty/student ratio, • Central Board of Indirect Taxes and Customs is a
5)international faculty ratio and subsidiary of the Department of Revenue under the
6) international student ratio. Ministry of Finance.
• It rates the worlds top 1000 universities. • It is the nodal national agency responsible for
• Massachusetts Institute of Technology (MIT) is ranked administering Customs, GST, Central Excise, Service
as the world’s best university followed by Stanford Tax & Narcotics in India.
University and Harvard University. • Tasks ➔ It deals with the tasks of formulation of policy
• IIT Bombay is ranked 172nd while IISc is 185th and IIT concerning levy and collection of Customs, Central
Delhi is at 193rd place. A total of 21 Indian higher Excise duties, Central Goods & Services Tax and IGST,
education institutions are in the top 1000. and it also works for the prevention of smuggling and
administration.
Q. 5) With reference to Central Board of Indirect Taxes •Administrative authority ➔ The Board is the
and Customs (CBIC), Consider the following statements: administrative authority for its subordinate organizations,
1. It is the nodal national agency responsible for including Custom Houses, Central Excise and Central
administering Central Excise, Service Tax & Narcotics in GST Commissionerates and the Central Revenues
India. Control Laboratory.
2. It is a subsidiary of the Department of Revenue under
the Ministry of Finance. Q.6) Consider the following statements with reference to
Which of the following statements is/are correct: ANANYA recently seen in news :
(a) 1 only
(b) 2 only 1. It is a Nano-technology based disinfectant
(c) Both 1 and 2 spray to disinfect all types of surfaces.
(d) Neither 1 nor 2 2. It is developed by Defence Research and Development
Organisation.
Answer (c) Both 1 and 2 Which of the following statements is/are correct:
Explanation- Central Board of Indirect Taxes and (a) 1 only
Customs launches its flagship programme 'Turant (b) 2 only
Customs' at Bengaluru & Chennai. (c) Both 1 and 2
(d) Neither 1 nor 2
About Turant customs
• It is a giant leap forward to leverage technology for Answer (a) 1 only
faster Customs clearance of imported goods. Explanation- Second statement is NOT correct because
• Importers will now get their goods cleared from ANANYA is developed by a deemed university, Defence
Customs after a faceless assessment is done remotely by Institute of Advanced Technology (DIAT).
the Customs officers located outside the port of import.
• Now, the goods imported at Chennai may be assessed • ANANYA can be used by anyone, from a common man
by the Customs officers located at Bengaluru and vice to a healthcare worker, for individual as well as large scale

32 | P a g e
use. years. The last Census was conducted in 2015.
• This spray can be used on masks, PPEs, hospital linens, • The Lion Census usually runs for more than two days,
as well as other likely contaminated surfaces such as including a preliminary census and a final census.
medical instruments, elevator buttons, door knobs, • However, the Poonam Avlokan is carried out for 24
corridors and rooms. hours assessing the number of lions and their locations in
• It is developed through a Nano-technology based their respective jurisdictions.
disinfectant spray to combat COVID-19 by disinfecting • The Forest Department invites NGOs, experts and
all types of surfaces. wildlife enthusiasts to join the Census for transparency
and augmenting manpower.
Q. 7) Which of following statements is/are correct about • Whereas the Poonam Avlokan exercise is conducted
Asiatic Lions: only by forest staff.
1. The first Lion Census was conducted by the Nawab of • Thus, the Lion Census experiences larger participation
Junagadh in 1936. of the people compared to the Poonam Avalokan exercise.
2. They live in a compact tract of dry deciduous forest and • The Lion Census is done using the block counting
open grassy scrublands in southwestern part of Saurashtra method — in which census enumerators remain stationed
region of Gujarat. at water points in a given block and estimate abundance
Which of the following statements is/are correct: of lions in that block, based on direct sighting of lions who
(a) 1 only need to drink water at least once in 24 hours during the
(b) 2 only summer.
(c) Both 1 and 2 • Under Poonam Avlokan exercise, the teams keep
(d) Neither 1 nor 2 moving in their respective territories and make their
estimates based on inputs provided by lion trackers and
Answer (c) Both a and b on chance sightings.
Explanation:
Gir National Park
ASIATIC LION • The Gir National Park and Wildlife Sanctuary is located
• According to a recent census, the number of Asiatic in the Junagadh district of Gujarat.
lions have now risen by 29% over five years to an • The Gir Forests is the only natural habitat of Asiatic
estimated 674 in the Gir forest region and other revenue lions. It was declared as a sanctuary in 1965 and a national
areas of coastal Saurashtra. park in 1975.
• Scientific Name ➔ Panthera Leo Persica. • Gir is often linked with "Maldharis" who have survived
• Characteristics ➔ Asiatic lions are slightly smaller than through the ages by having a symbiotic relationship with
African lions. the lions.
• The most striking morphological character is a • Maldharis are religious pastoral communities living in
longitudinal fold of skin running along belly of Asiatic Gir. Their settlements are called "nesses".
Lions.
Q.8) Babesia recently seen in news is a:
• Habitat and Distribution ➔ At present Gir National Park
(a) Tiny parasite that infects red blood cells.
and Wildlife Sanctuary in Gujarat is the only abode of the
(b) Robots to help health workers in conducting a thermal
Asiatic lion.
screening.
• They live in a compact tract of dry deciduous forest and
(c) A new type of matter discovered inside neutron stars.
open grassy scrublands in southwestern part of Saurashtra
(d) Group of viruses affecting nervous system.
region of Gujarat.
• Status ➔ Listed in Schedule I of Wildlife (Protection) Answer (a) Tiny parasite that infects red blood cells.
Act 1972, in Appendix I of CITES and as Endangered on Explanation -
IUCN Red List. Babesia
• Babesia is a tiny parasite that infects red blood cells.
Poonam Avlokan • Infection with Babesia is called babesiosis.
• The first Lion Census was conducted by the Nawab of • The parasitic infection is usually transmitted by a tick
Junagadh in 1936. bite, Babesiosis occurs in both Animals and Humans.
• Since 1965, the Forest Department has been regularly • Babesiosis outbreak was reported in Girforest in 2020,
conducting the Lion Census every five years. and around two dozen lions were reported to be killed.
• The regular Lion Census is conducted once every five

33 | P a g e
Q. 9) Brook’s Range is a mountain range in far northern
North America is considered as an extension of the:

(a) Andes Mountain


(b) Caucasus Mountains
(c) Blue Ridge Mountains
(d) Rocky Mountains

Answer: (d) Rocky Mountains


Brook’s Range
• Located in Northern Alaska.
• Extends in the east-west direction and is a continuation
of the Rocky.

Sierra Nevada
• The Sierra Nevada Mountain Range runs north-to-south
along the west coast of the United States mostly in the
state of California and some in the state of Nevada.
• The highest point in the Sierra Nevada Mountains is
Mount Whitney at 14,505 feet, which is also the tallest
mountain in the lower 48 United States.
• The largest trees in the world, the giant sequoia trees,
live in the Sierra Nevada. They can grow up to 270 feet
tall and over 25 feet in diameter.
• The Sierra Nevada is also home to Yosemite National
Park and Lake Tahoe.
• The Sierra Nevada Mountains are fault-block
mountains, meaning they were formed along a fault in the
Earth's crust.
Q. 10) What is the correct order of Mountain ranges of
North America from West to East? Rocky Mountains
•The Rocky Mountains form the longest mountain range
(a) Appalachian Mountains, Rocky Mountains, Sierra in North America and the second longest range in the
Nevada world.
(b) Rocky Mountains, Appalachian Mountains, Sierra • They stretch 3,000 miles north-to-south from New
Nevada Mexico, across the United States to Montana, and well
(c) Sierra Nevada, Rocky Mountains, Appalachian into Canada. The highest point of the Rockies is Mount
Mountains Elbert in Colorado which rises 14,440 feet above sea
(d) Sierra Nevada, Appalachian Mountains, Rocky level.
Mountains • The Continental Divide for North America is located
along the Rocky Mountains. It is at this point that water
Answer (c) Sierra Nevada, Rocky Mountains, flows either to the Atlantic Ocean in the east or to the
Appalachian Mountains Pacific Ocean in the west.
Explanation- • The Rockies are known for their distinct seasons with
warm, rainy summers and cold snowy winters.
• Much of the Rocky Mountains is covered with forests of
spruces trees, pines, oaks, junipers, and firs. A wide
variety of wildlife can be found in the Rockies including
bighorn sheep, badgers, grizzly bears, black bears,
coyotes, elk, bighorn sheep, and white-tailed deer.

Appalachian Mountains
• The Appalachian Mountains run for 1,500 miles along

34 | P a g e
the east coast of the United States from northern Alabama one of the world's highest airstrip.
to Maine. • DBO is located only 9 km away from the Line of Actual
• The highest point of the Appalachians is 6,684 feet at Control with China.
Mount Mitchell . • Daulat Beg Oldi lies near the easternmost point of the
• The Appalachians are part of a temperate forest biome Karakoram Range in a cold desert region in the far north
and are mostly covered with a variety of trees including of India, just 8 km south of the Chinese border and 9 km
pine trees, spruce, birch, and maple trees. northwest of the Aksai Chin Line of Actual Control
• Animals that can be found in the Appalachians include between China and India.
squirrels, cottontail rabbits, white-tailed deer, wolves, • Other than Siachen Glacier military bases, it is India's
beavers, black bears, and the red-tailed hawk. northernmost settlement. The nearest civilian town is
• Some of the smaller ranges within the Appalachians Murgo to the south, which has a small population of
include the Great Smoky Mountains, the Blue Ridge Baltis.
Mountains, Green Mountains, White Mountains, • Timeline ➔ The construction of the DSDBO first started
Longfellow Mountains, and the Berkshires. in 2000 and was to be completed by 2012. The road,
which was being monitored by the PMO, was aligned
17 JUNE with the Shyok riverbed that was damaged every summer
during flooding. Later, major portions of the road were
Q. 1) With reference to Daulat Beg Oldie , Consider the realigned, keeping them away from the river to avoid the
following statements: annual damage.
1. It is India’s northernmost corner and known as Sub- • Colonel Chewang Rinchen bridge ➔ In October,
Sector North. defence minister Rajnath Singh inaugurated the 430-m
2. It lies near the easternmost point of the Karakoram Colonel Chewang Rinchen bridge that joins Durbuk to
Range. DBO. India has decided to not stop road construction on
Which of the following statements is/are correct its side of the LAC despite the border confrontations.
(a) 2 only Significance ➔
(b) 1 only • The road will help manage the border and the areas
(c) Both 1 and 2 adjoining Aksai Chin, Chip Chap River and Jiwan Nalla.
(d) Neither 1 nor 2 • It will also ensure faster deployment of troops in the
area. Before the laying of the road, the only way to reach
Answer (c) Both 1 and 2 the area was via the ALG, where heavy- lift aircraft, such
Explanation : as the C-130J, can land.
• In the reporting on the LAC stand-off, the Darbuk-
Shyok-Daulat Beg Oldie (DSDBO) road has often
appeared in news.
• Location ➔ The road from Darbuk traverses at an
altitude of 14,000 feet and reaches Shyok, the last Indian
village in the region, between Shyok and Karakoram Pass
(that divides Ladakh from China’s Xinjiang province) lies
DBO, a plateau at an altitude of over 16,000 feet and the
location of an Advanced Landing Ground (ALG) used by
the air force to drop supplies.
• DBO is India’s northernmost corner, which in army
parlance is called Sub- Sector North and building a road
till here has been of vital importance to India.
• Daulat Beg Oldi is a historic campsite and current
military base located in Ladakh, India on an ancient trade
route connecting Ladakh to the Tarim Basin.
• It is named after Sultan Said Khan (Daulat Beg), who
died here on his return journey after the invasion of
Ladakh and Kashmir. The Chip Chap River flows just to
the south of Daulat Beg Oldi from east to west. Daulat
Beg Oldie also has an airstrip at an altitude of 5065 meters

35 | P a g e
Q.3) Consider the following statements about Black
Q. 2) Consider the following Pair: Panther:
1. Vamsadhara River - Andhra Pradesh & 1. It is as shy as a normal leopard and very difficult to
Water Dispute - Odisha detect.
2. Krishna River Water Dispute - Karnataka & Goa 2. It is mostly found in densely forested areas of Western
3. Cauvery River Water Dispute - Karnataka &Tamil India.
Nadu Which of the above statement is/are correct
Which of the following Pair given below is correctly (a) 1 only
matched? (b) 2 only
(a) 1 and 2 only (c) Both 1 and 2
(b) 2 and 3 only (d) Neither 1 nor 2
(c) 1 and 3 only
(d) 1,2,3 Answer (a) 1 only

Answer (c) 1 and 3 only Explanation- Second statement is NOT correct because
Explanation- Second match is NOT correct because Black panther is mostly found in densely forested areas of
States concerned in Krishna river water dispute is southern India.
Karnataka, Maharashtra, Andhra pradesh. •Recently, A black panther was spotted in Goa’s Netravali
• Goa, Karnataka and Maharashtra are the concerned Sanctuary.
states in Mahadayi Water Disputes Tribunal.
• Cauvery Water Disputes Tribunal- States concerned: About Black Panther
Karnataka, Tamil Nadu, Puducherry and Kerala • A black panther is the melanistic colour variant of any
• Andhra Pradesh and Odisha CM recently held talks to Panthera, particularly of the leopard in Asia and Africa,
iron out all differences with regard to the sharing of and the jaguar in the Americas.
Vamsadhara River waters. • It is as shy as a normal leopard and very difficult to
detect.
About Vamsadhara River • Habitat ➔ It is mostly found in densely forested areas
• Location ➔ River Vamsadhara is an important east- of southern India.
flowing river between Rushikulya and Godavari, in
Odisha and Andhra Pradesh. Areas where black panther has been spotted earlier ➔
• Origin ➔ The river originates in the border of Thuamul ○ Periyar Tiger Reserve (Kerala)
Rampur in the Kalahandi district and Kalyansinghpur in ○ Bhadra Tiger Reserve, Dandeli-Anshi Tiger Reserve
Rayagada district of Odisha. and Kabini Wildlife Sanctuary (Karnataka)
• It is an east-flowing river which originates in Kalahandi ○ Achanakmar Tiger Reserve (Chhattisgarh)
district of Odisha, flows in Odisha, along its boundary ○ Mhadei Wildlife Sanctuary (Goa)
with Andhra Pradesh and finally joins the Bay of Bengal ○ Mudumalai Tiger Reserve (Tamil Nadu)
at Kalingapatnam, Andhra Pradesh.
• It is the main river of north-eastern Andhra region and Netravali Wildlife Sanctuary
the Boddepalli Rajagopala Rao Project was constructed •Location ➔ Netravali Wildlife Sanctuary is located in
on it to meet the irrigation needs of the region. South Eastern Goa and constitutes one of the vital
• It runs for a distance of about 254 kilometres, where it corridors of the Western Ghats.
joins the Bay of Bengal at Kalingapatnam, Andhra • Bounded By ➔ Cotigao wildlife sanctuary on the
Pradesh. eastern side and Bhagwan Mahaveer Sanctuary
• The total catchment area of the river basin ➔ 10,830 andMollem National Park on the northern side.
square kilometres. •River ➔ Netravali or Neturli is an important tributary of
• The dispute ➔ Andhra Pradesh wants to build the River Zuari, which originates in the sanctuary.
Neradi bridge across the river which will be possible only • Important waterfalls ➔ Savari and Mainapi.
after Odisha’s consent. • Forests ➔ mostly consist of moist deciduous vegetation
• Odisha argues that the flood flow canal would result in
interspersed with evergreen and semi-evergreen habitat.
drying up the existing river bed and consequent shifting
of the river affecting the groundwater table. • Fauna ➔ Leopard, Giant Squirrel, Mouse Deer, Nilgiri

36 | P a g e
Wood Pigeon and Ceylon Frogmouth. intersected by multiple tidal streams and channels.
Other Protected Areas in Goa ➔ •Four protected areas in the Sundarbans are enlisted as
○ Cotigao Wildlife Sanctuary UNESCO World Heritage Sites, viz Sundarbans National
○ Mhadei Wildlife Sanctuary Park, Sundarbans West, Sundarbans South and
○ Bhagwan Mahaveer Sanctuary Sundarbans East Wildlife Sanctuaries.
○ Bondla Wildlife Sanctuary • It is home to many rare and globally threatened wildlife
○ Mollem National Park species such as the estuarine crocodile, Royal Bengal
○ Dr Salim Ali Bird Sanctuary Tiger, Water monitor lizard, Gangetic dolphin, and olive
ridley turtles.
Q.4) Which of the following is/are INCORRECT about
Sundarbans: Q. 5) Who among the following was also known as
1. The Sundarbans is a mangrove area in the delta formed Gurudev, Kabiguru, and Biswakabi?
by the confluence of the Ganges, Brahmaputra and (a) Rabindranath Tagore
Meghna Rivers in the Bay of Bengal. (b) Bankimchandra Chatterjee
2. It is home to threatened wildlife species such as the (c) Maithlisharan Gupta
estuarine crocodile, Royal Bengal Tiger, Northern River (d) Bhartendu Harishchnadra
Terrapin, Gangetic dolphin, and Fishing Cat.
Select the correct code given below: Answer (a) Rabindranath Tagore
(a) 1 only Explanation- National Gallery of Modern Art will
(b) 2 only organise a Virtual Tour titled “Gurudev – Journey of the
(c) Both 1 and 2 Maestro through his visual vocabulary” to commemorate
(d) Neither 1 nor 2 the 159th birth anniversary of Rabindranath Tagore.

Answer (d) Neither 1 nor 2 About Rabindranath Tagore


Explanation: • Rabindranath Tagore also known by his pen name
According to the latest estimation of tiger numbers in the Bhanu Singha Thakur (Bhonita), and also known by
Indian Sunderbans by the West Bengal Forest names such as Gurudev, Kabiguru, and Biswakabi, was a
Department, the tiger count for the year 2019-20 rose to Bengali poet, writer, music composer, and painter from
96, from 88 in 2018-19. the Indian subcontinent.
• He is sometimes referred to as "the Bard of Bengal"
•He was founder of a unique educational institution
About Sundarbans Visva-Bharati
• The Sundarbans is a mangrove area in the delta formed • He reshaped Bengali literature and music, as well as
by the confluence of the Ganges, Brahmaputra and Indian art with Contextual Modernism in the late 19th and
Meghna Rivers in the Bay of Bengal. early 20th centuries.
• The Sunderbans Delta is the only mangrove forest in the • Tagore was primarily known as a writer, poet,
world inhabited by tigers. philosopher and aesthetician, music composer .
• It spans from the Hooghly River in India's state of West Contributions ➔
Bengal to the Baleswar River in Bangladesh. ◎ He wrote the National Anthems of India and
• Indian Sundarbans constitute over 60% of the country’s Bangladesh.
total mangrove forest area. ◎ He left his imprint on art and played a role in
◎ Indian Sundarbans has been recognised as UNESCO transforming its practices and ushering into modernism.
World Heritage Site in 1987 and ‘Wetland of Awards ➔
International Importance’ under the Ramsar Convention ◎ In 1913, he became the first Indian to receive a Nobel
in January, 2019. Prize in Literature for his novel ‘Geetanjali’.
•Life in these mangrove forests ➔ It is home to rare and ◎ Role in the freedom struggle ➔ He denounced British
globally threatened species, such as estuarine crocodile , imperialism, yet he did not fully support or agree with
Northern River Terrapin, Water monitor lizard, Gangetic Gandhi and his Non-cooperation Movement.
dolphin, olive ridley turtles, Irrawaddy Dolphin, and the ◎ In his writings, he also voiced his support of Indian
Fishing Cat. nationalists.
•It comprises closed and open mangrove forests,
agriculturally used land, mudflats and barren land, and is Q.6) Consider the following statements with reference to

37 | P a g e
Official Secrets Act, 1923 : • The Act only empowers persons in positions of authority
to handle official secrets, and others who handle it in
1. It states that one cannot approach, inspect, or even pass prohibited areas or outside them are liable for
over a prohibited government site or area. punishment.
2. A person prosecuted under this Act can be charged with • Under the Act, search warrants may be issued at any
the crime even if the action was unintentional and not time if the magistrate determines that based on the
intended to endanger the security of the state. evidence there is enough danger to the security of the
3.If the magistrate determines, search warrants may be state.
issued at any time under this Act. • When a company is seen as the offender under this Act,
Which of the following statements is/are correct: everyone involved with the management of the company,
(a) 1 only including the board of directors, can be liable for
(b) 2 only punishment.
(c) 1 and 3 only • In the case of a newspaper, everyone – including the
(d) 1, 2, 3 editor, publisher and the proprietor — can be imprisoned
for an offense.
Answer (d) 1, 2, 3
Explanation- Under operation named Desert Chase, Q. 7) Which of following statements is/are correct about
Rajasthan Police arrested two Challenger Deep:
civil defence employees in Jaipur based on Military 1. It is the deepest known point in the Earth’s oceans.
Intelligence (MI) inputs that they had been passing on 2. The Challenger Deep is located in the Western Pacific
sensitive information to Ocean at the southern end of the Mariana Trench.
Pakistan's spy agency ISI . Both were arrested under Choose the correct code given below
relevant sections of the Official Secrets Act,1923. (a) 1 only
• Involved bodies ➔ The operation was jointly carried out (b) 2 only
by the Army, UP ATS, and Rajasthan Police, and was (c) Both 1 and 2
named Operation ‘Desert Chase’. (d) Neither 1 nor 2
• Background ➔ The work on the “operation” began in
August 2019, when the MI Lucknow, through its sleuths, Answer (c) Both a and b
learnt about an espionage agent near Sri Ganganagar who Explanation:
was passing military information to his handlers in • Kathy Sullivan, known as the first American woman to
Pakistan. walk in space has also become the first woman to reach a
Challenger Deep.
Official Secrets Act 1923 • Location ➔ It is the deepest known point in
• The Official Secrets Act 1923 is India's anti-espionage the Earth’s oceans with a depth of around
act held over from the British colonial period. 10,984m.
• It states clearly that actions which involve helping an • It is located in the Western Pacific Ocean at
enemy state against India are strongly condemned. the southern end of the Mariana Trench
• It also states that one cannot approach, inspect, or even near the Mariana Islands group.
pass over a prohibited government site or area like an • Significance of Deep Ocean Areas ➔
electrical substation. Finding out about the deep ocean areas can potentially
• According to this Act, helping the enemy state can be in reveal new sources for medical drugs, food, energy
the form of communicating a sketch, plan, model of an resources and other products.
official secret, or of official codes or passwords, to the • It can also help to predict earthquakes and tsunamis and
enemy. help us understand how we are affecting and getting
affected by the Earth’s
Prosecution and penalties environment.
• Punishments under the Act range from three to life
imprisonment (if intent is to declare war against India - Q.8) Recently released, Nature Index 2020 is a database
section 5 )imprisonment. compiled by?
• A person prosecuted under this Act can be charged with
the crime even if the action was unintentional and not (a) Nature Research
intended to endanger the security of the state. (b) GreenPeace International

38 | P a g e
(c) UN Environment Programme
(d) International Union for Conservation of Nature

Answer : Nature Research


• Three of the autonomous institutions of the Department
of Science & Technology, Government of India have
found their place among top 30 Indian Institutions
including universities, IITs, IISERs, and Research
Institutions and Labs as per Nature Index 2020 ratings
based on the research published in the top journals, a
measure of research quality.
• These are the Indian Association for the Cultivation of
Science (IACS), Kolkata at 7th position, Jawaharlal
Nehru Centre for Advanced Scientific Research
(JNCASR), Bangalore at 14th position and S. N. Bose
National Centre for Basic Sciences, Kolkata at 30th
position.
• Keeping out CSIR, which is a cluster of institutions,
Q. 10) Which of the following is largest river system in
IACS is among the top three institutions in quality
the United States?
Chemistry Research in India. JNCASR ranks 4th among
(a) Potomac
academic institutions in life sciences, 10th in Chemistry
(b) Colorado
and Physical Sciences, 10th among Indian academic
(c) Mississippi
institutions, and 469th in global ranking.
(d) Columbia
Nature Index
Answer (c) Mississippi
• The Nature Index is a database of author affiliation
Explanation :
information collated from research articles published in
• Mississippi is the largest river system in the United
an independently selected group of 82 high-quality
States
science journals.
• The Mississippi River is the second-longest river and
• The database is compiled by Nature Research.
chief river of the second-largest drainage system on the
• The Nature Index provides a close to real-time proxy of
North American continent, second only to the Hudson
high-quality research output and collaboration at the
Bay drainage system
institutional, national and regional level.
• With its many tributaries, the Mississippi's watershed
drains all or parts of 32 U.S. states and two Canadian
Q. 9) Which of the following Rivers of North America are
provinces between the Rocky and Appalachian
arranged correctly from North to South:
mountains.
• The Mississippi ranks as the fourth-longest river and
(a) Rio Grande, Mackenzie River, Columbia River,
fifteenth-largest river by discharge in the world.
Colorado River
(b) Colorado River, Columbia River, Mackenzie River,
Rio Grande
(c) Mackenzie River, Columbia River, Colorado River,
Rio Grande
(d) Mackenzie River, Colorado River, Rio Grande,
Columbia River

Answer: (c) Mackenzie River, Columbia River, Colorado


River, Rio Grande

39 | P a g e
Q. 2) Consider the following Pair:

1. Lonar Lake Gujrat


2. Pangong Lake Assam
3. Loktak Lake Manipur
Which of the following Pair given below is correctly
matched?
(a) 2 and 3 only
(b) 3 only
(c) 1 and 3 only
(d) 1,2,3

Answer (b) 3 only


18 JUNE Explanation- Lonar Lake is in Maharashtra and Pangong
lake is in Ladhakh
Q. 1) Recently, The first population estimation exercise •Recently, The colour of water in Maharashtra’s Lonar
of the Indian gaur carried out in the Nilgiris Forest Lake, formed after a meteorite hit the Earth some 50,000
Division has revealed that more than an estimated 2,000 years ago, has changed to
Indian gaurs inhabit the entire division. With reference to glaring.
Indian Gaur, Consider the following statements: Why there’s a color change?
1. It is one of the largest extant bovines found in India and • The salinity and algae can be responsible for this change.
Native to South and South-East Asia. • There is no oxygen below one meter of the lake’s water
2. It is the State animal of Karnataka and have threats of surface.
hunting for consumption. • There is an example of a lake in Iran, where water
Which of the following statements is/are correct becomes reddish due to increase in salinity.
(a) 2 only • The level of water in the Lonar Lake is currently low as
(b) 1 only compared to the few past years and there is no rain to pour
(c) Both 1 and 2 fresh water in it.
(d) Neither 1 nor 2 • The low level of water may lead to increased salinity and
change in the behaviour of algae because of atmospheric
Answer (b) 1 only changes.
Explanation : Second statement is NOT correct because It
is the State animal of Goa. About Lonar lake
• The Indian Gaur also called the Indian bison is one of • Location ➔ Lonar Lake, also known as Lonar crater, is
the largest extant bovines found in India. a notified National Geo-heritage Monument, saline (pH
• Geographic range ➔ Native to South and South-East of 10.5), Soda Lake, located at Lonar in Buldhana district,
Asia. Maharashtra.
• Distribution ➔ In India, the population was estimated to • Created by ➔ an asteroid collision with earth impact
be 12,000–22,000 in the mid-1990s. during the Pleistocene Epoch.
• The Western Ghats and their outflanking hills in • It is one of the four known, hyper-velocity, impact
southern India constitute one of the most extensive extant craters in basaltic rock anywhere on Earth.
strongholds of gaur, in particular in the Wayanad – • It sits inside the Deccan Plateau—a massive plain of
Nagarhole – Mudumalai – Bandipur complex. volcanic basalt rock created by eruptions some 65 million
• Conservation Status ➔ years ago.
○ IUCN Status: Vulnerable
○ Wildlife Protection Act,1972: Schedule I About Loktak Lake
•Threats ➔ a) Habitat Fragmentation b) Hunting for • Loktak Lake is the largest freshwater lake in Northeast
consumption and c) India and is famous for the phumdis (heterogeneous mass
contract diseases transmitted by domestic cattle. of vegetation, soil and organic matter at various stages of

40 | P a g e
decomposition) floating over it. • Top Institutes in overall category ➔
• The lake is located at Moirang in Manipur. 1) IIT-Madras
• The largest of all the phumdis and is situated on the 2) IISc Bangalore and
southeastern shore of the lake. 3) IIT-Delhi
• Located on this phumdi, Keibul Lamjao National Park • Top Universities in India ➔
is the only floating national park in the world. 1) Indian Institute of Science(IISc)
• The park is the last natural refuge of the endangered 2) Jawaharlal Nehru University and
Sangai (state animal), brown- antlered deer. 3) Banaras Hindu University.
• It is designated as a wetland of international importance
• Top engineering institutes in India ➔
under the Ramsar Convention and It was also listed under
1) IIT-Madras
the Montreux Record.
2) IIT-Delhi and
3) IIT- Bombay.
About Pangong Lake
• It is an endorheic lake in the Himalayas situated at a • Top College ➔ Miranda House, Delhi.
height of about 4,350 m • Top Dental College ➔ Maulana Azad Institute of Dental
• Pangong Lake is located in the Union Territory of Sciences, Delhi.
Ladakh. • The Dental category was included for the first time in
• It is situated at a height of almost 4,350m and is the NIRF Rankings.
world’s highest saltwater lake.
• Extending to almost 160km, one-third of the Pangong Q.4) Which of the following is/are correct regarding
Lake lies in India and the other two-thirds in China. Sahakar Mitra:
• It is not a part of the Indus river basin area and 1. It is an initiative by the National Cooperative
geographically a separate landlocked river basin. Development Corporation (NCDC) for young
• During winter the lake freezes completely, despite being professionals.
saline water. 2. This scheme will help cooperative institutions access
new and innovative ideas.
Q.3) Which of the following are parameters under Select the correct code given below:
National Institutional Ranking (a) 1 only
Framework (NIRF)? (b) 2 only
1. Learning and Resources (c) Both 1 and 2
2. Research and Professional Practices (d) Neither 1 nor 2
3. Graduation Outcomes
4. Performance in High school Answer (c) Both 1 and 2
Which of the above statement is/are correct Explanation:
(a) 1, 2 , 4 only
(b) 1, 2 and 3 only Sahakar Mitra scheme
(c) 2, 3 and 4 only • Ministry of Agriculture & Farmers’ Welfare has
(d) 1,2,3 and 4 launched a Scheme on Internship Programme (SIP) i.e.
Sahakar Mitra.
Answer : (b) 1, 2 , 3 only • It is an initiative by the National Cooperative
Development Corporation (NCDC) for young
Explanation- Parameters under National Institutional professionals.
Ranking Framework (NIRF) • It is expected to be beneficial for both i.e. cooperatives
a) Teaching as well as for the young professionals.
b) Learning and Resources • Earlier, the government launched 'The Urban Learning
c) Research and Professional Practices Internship Program (TULIP) Portal' to provide internship
d) Graduation Outcomes opportunities to thousands of fresh graduates and
e) Outreach and Inclusivity and engineers of the country under the 'Smart City' projects.
f) Perception. • The Sahakar Mitra scheme will help cooperative
• The National Institute Ranking Framework (NIRF) institutions access new and innovative ideas of young
ranking list has been released by the Ministry of Human professionals while the interns will gain experience of
Resource Development (MHRD). working in the field to be self-reliant.
• It will provide the young professionals an opportunity of

41 | P a g e
practical exposure and learning from the working of Oxford COVID-19 Government Response Tracker.
NCDC and cooperatives as a paid intern. • The Tracker involves a team of 100 Oxford
• It would also provide an opportunity to professionals community members who have continuously
from academic institutions to develop leadership and updated a database of 17 indicators of
entrepreneurial roles through cooperatives as Farmers government response.
Producers Organizations (FPO). • Indicators ➔ These indicators examine containment
• In line with the AtmaNirbhar Bharat (Self Reliant India), policies such as school
it focuses on the importance of Vocal for Local. and workplace closings, public events, public transport,
• The eligible participants to the scheme are as follows: stay-at-home policies.
1)Professional graduates in disciplines such as • Weightage ➔ The Stringency Index is a number from 0
Agriculture and allied areas, IT, etc. to 100 that reflects these indicators. A higher index score
2)Professionals who are pursuing or have completed their indicates a higher level of stringency.
MBA degrees in Agribusiness, Cooperation, Finance,
Stringency Index tell us ➔
International Trade, Forestry, Rural Development, Project
• It provides a picture of the stage at which any country
Management, etc.
enforced its strongest measures.
• NCDC has designated funds for the paid internship
• Oxford provides an overlay of countries’ death curve
program under which each intern will get financial
and their stringency score.
support over a 4 months internship period.
• Some countries saw their deaths just begin to flatten as
they reached their highest stringency, such as Italy, Spain,
National Cooperative Development Corporation
or France.
• The National Cooperative Development Corporation
(NCDC) was established by an Act of Parliament in 1963 • Indian scenario ➔ When compared to other countries
as a statutory Corporation under the Ministry of with the similar or higher caseload, India called its strict
Agriculture & Farmers’ Welfare. lockdown at a much earlier point on its case and death
• NCDC functions through its Head Office at New Delhi curves
and multiple Regional Offices.
• The objectives of NCDC are planning and promoting Q.6) Consider the following statements with reference to
programmes for production, processing, marketing, Asian Infrastructure Investment Bank (AIIB) :
storage, export and import of agricultural produce,
foodstuffs, industrial goods, livestock and certain other 1. It is a multilateral financial institution proposed by
notified commodities and services on cooperative China to provide finance to
principles. infrastructure projects in the Asia-Pacific region.
• The NCDC has the unique distinction of being the sole 2. The voting shares are based on the basis of contribution
statutory organization functioning as an apex financial to the bank’s authorized capital.
and developmental institution exclusively devoted to the
cooperative sector. Which of the following statements is/are correct:
(a) 1 only
Q. 5) Stringency Index sometimes seen in news is (b) 2 only
composed and published by: (c) 1 and 3 only
(a) Cornell University (d) 1, 2, 3
(b) World Economic Forum
(c) University of Oxford
(d) The Institute of Management Development Answer (a) 1 only
Explanation- Second Statement is NOT correct because
Answer (c) University of Oxford The voting shares are based on the size of each member
Explanation- Recently, the Stringency Index composed country’s economy (GDP in PPP terms) and not on the
and published by the University of Oxford has found that basis of contribution to the bank’s authorized capital.
India has enforced one of the strongest lockdown • Recently, the Asian Infrastructure Investment Bank
measures in the World at a 100 score. (AIIB) has approved US$ 500 million for ‘Covid-19
Emergency Response and Health Systems Preparedness
About Stringency Index Project’ initiated by India.
•What is it? ➔ It is among the metrics being used by the
About Asian Infrastructure Investment Bank (AIIB)

42 | P a g e
• What is it? ➔ The Asian Infrastructure Investment Bank Choose the correct code given below
(AIIB) is an multilateral financial institution proposed by (a) 1 only
China. (b) 2 only
• Purpose ➔ The purpose of the multilateral development (c) Both 1 and 2
bank is to provide finance to infrastructure projects in the (d) Neither 1 nor 2
Asia-Pacific region.
Answer (a) 1 only
• Headquartered ➔ Beijing, China
Explanation: Second statement is NOT correct because
• Formation ➔ January 2016 Olive Ridley Turtles are predominantly carnivores.
• Membership ➔ open to all members of the World Bank
or the Asian Development Bank and is divided into About Olive ridley turtles
regional and non-regional members. • What is it? ➔ The Olive ridley turtles are the second
• AIIB allows for non-sovereign entities to apply for AIIB smallest and most abundant of all sea turtles found in the
membership, assuming their home country is a member. world.
• These turtles are carnivores and get their name from
Structure their olive colored carapace.
•Voting pattern ➔ The voting shares are based on the size • Habitat ➔ They are found in warm waters of the Pacific,
of each member country’s economy (GDP in PPP terms) Atlantic and Indian oceans.
and not on the basis of contribution to the bank’s
• Migration ➔ They migrate thousands of kilometers
authorized capital.
between feeding and mating grounds in the course of a
• China, India and Russia are the three largest
year.
shareholders.
• Bank's major decisions will require the support of at • Arribada (Mass Nesting) ➔ They are best known for
least 75 per cent of the votes. their unique mass nesting called Arribada, where
• China is the largest shareholder with 26.61 % voting thousands of females come together on the same beach to
shares in the bank followed by India (7.6%), Russia lay eggs.
(6.01%) and Germany (4.2 %). • They lay their eggs over a period of five to seven days
in conical nests about one and a half feet deep which they
Various organs of AIIB dig with their hind flippers.
• Board of Governors ➔ One Governor and one Alternate ◎ Protection Status ➔
Governor appointed by each member country. • IUCN Red List: Vulnerable
• CITES: Appendix I
•Board of Directors ➔ Non-resident Board of Directors is
• Indian Wildlife (Protection) Act, 1972: Schedule I
responsible for the direction of the Bank’s general
operations, exercising all powers delegated to it by the • Step Taken for Conservation ➔ To reduce accidental
Board of Governors. killing in India, the Odisha government has made it
mandatory for trawls to use Turtle Excluder Devices
• International Advisory Panel ➔ To support the
(TEDs), a net specially designed with an exit cover which
President and Senior Management on the Bank’s
allows the turtles to escape while retaining the catch.
strategies and policies as well as on general operational
issues.
Q.8) Which of the following has launched
• Significance of AIIB ➔ The capital of the bank is $100 the Consortium for Affordable & Rapid Diagnostics
billion, equivalent to 2⁄3 of the capital of the Asian (CARD) to scale up India’s
Development Bank and about half that of the World Bank. capacity to make corona virus testing kit?
Q. 7) Which of following statements is/are correct about (a) Indian Council of Medical Research(ICMR)
Olive Ridley turtles: (b) Department of Science & Technology
1. The olive ridley is classified as vulnerable according to (c) Niti Ayog
the International Union for Conservation of Nature and (d) Council of Scientific and Industrial Research
Natural Resources (IUCN) and is listed in Appendix I of
CITES. Answer : (c) Niti Ayog
2. The Olive ridley turtles are the most abundant of all sea
turtles found in the world and they are predominantly Project CARD to push local production of testing kits
herbivores. • Niti Aayog and the Department of Biotechnology have

43 | P a g e
launched the Consortium for Affordable & Rapid •Nor'easter - Strong storm with winds from the northeast
Diagnostics (CARD) to scale up India’s capacity to make on the north eastern coast of the United States and the east
coronavirus testing kit. coast of Canada.
Aim ➔ The plan involves bringing scientists and • Santa Ana winds -dry downslope winds that affect
laboratories in touch with private firms to coastal Southern California and northern Baja California
produce millions of testing kits, including • Williwaw -strong, violent wind occurring in the Strait of
around 10 million rapid antibody tests that Magellan, the Aleutian Islands, and the coastal fjords of
offer quick results. Southeast Alaska

About Rapid Test Some more winds of North America


• What is it? ➔ It is a speedy test conducted to determine •Cordonazo- also referred to as el cordonazo de San
whether there has been any kind of recent viral infection Francisco or the Lash of St Francis -southerly hurricane
in a person’s body. winds along the west coast of Mexico
•Coromuel -south to south-west wind in the La Paz area
• How it works? ➔ When a pathogen enters a human
of the Baja California peninsula and the Gulf of California
body, specific antibodies are released as a response to the
• Norte -strong cold northeasterly wind in Mexico
virus. A rapid test can detect the presence of such
• Papagayo -periodic wind which blows across Nicaragua
antibodies in blood, serum or plasma samples quickly
and Costa Rica
indicating a viral infection.
• Brookings Effect -off-shore wind on the southwestern
Oregon coast, United States; also known as the Chetco
Q. 9) Which of the following wind of North America is
Effect
also known as Snow eater:
• Diablo- hot, dry, offshore wind from the northeast in the
San Francisco bay
(a) Foehn Wind
• The Hawk -cold winter wind in Chicago
(b) Chinook Wind
• Plough Wind -straight line wind which precedes
(c) Zonda Wind
thunderstorms or thunderstorm clusters
(d) Pampero Wind
•Sundowner- strong offshore wind off the California coast
• Washoe Zephyr - seasonal diurnal wind in parts of
Answer: (b) Chinook Wind
western Nevada
Explanation
• Witch of November, or November Witch- strong winds
• Chinook winds are winds in the interior West of North
blowing across the Great Lakes in autumn
America, where the Canadian Prairies and Great Plains
meet various mountain ranges.
• When the warm, dry, fast-moving Chinook winds pass
over snow-covered areas, the snow often vapourizes
before it has a chance to melt, according to the U.S.
Geological Survey. The winds can easily vaporize a foot
of snow within hours. Because of this, chinook winds are
often known as the "snow eaters."
• Winds in USA and Canada move down the west slopes
of the Rockies .
• It is beneficial to ranchers east of the Rockies as it keeps
the grasslands clear of snow during much of the winter.

Q. 10) Which of the following is NOT a local wind in


North America:
(a) Nor'easter Wind
(b) Santa Ana winds
(c) Williwaw Wind 19 JUNE
(d) Berg Wind
Q. 1) With reference to Sal Forest Tortoise, Consider the
Answer (d) Berg Wind following statements:
Explanation : Berg wind is a South African katabatic wind 1. It is a species of tortoise found in Southeast Asia and

44 | P a g e
parts of the Indian Subcontinent, particularly in Northeast (c) Gopal Krishna Gokhale
India. (d) Dadabhai Naoroji
2. It is listed as Critically Endangered in IUCN red list and
Legally protected in Schedule IV of the Indian Wildlife Answer (c) Gopal Krishna Gokhale
(Protection) Act, 1972. Explanation- Recently, PM paid tributes to Gopal Krishna
Which of the following statements is/are correct Gokhale on his birth
(a) 2 only anniversary. He was born on 9 May 1866.
(b) 1 only •Recently, The colour of water in Maharashtra’s Lonar
(c) Both 1 and 2 Lake, formed after a meteorite hit the Earth some 50,000
(d) Neither 1 nor 2 years ago, has changed to
glaring.
Answer (c) Both 1 and 2
About Gopal krishna Gokhale
Explanation : It is in news because over 90% of the • Ideology ➔ Gokhale worked towards social
potential distribution of the Sal forest tortoise falls outside empowerment, expansion of education , struggle for
current protected area’s network. freedom in India for three decades and rejected the use of
reactionary or revolutionary ways.
About Sal forest Tortoise • Role in INC ➔ associated with the Moderate Group of
• Scientific name ➔ Indotestudo elongate. Indian National Congress (joined in 1889). He became
• Common name ➔ Elongated tortoise, Yellow Tortoise president of INC in 1905 in Banaras session.
and Sal forest tortoise.
• Distribution ➔ It is a species of tortoise found in Role in Colonial Legislatures ➔
Southeast Asia and parts of the Indian Subcontinent, • Between 1899 and 1902, he was a member of the
particularly Northeast India. Bombay Legislative Council followed by work at the
• Physical description ➔ These up to 1 foot-long tortoises Imperial Legislative Council from 1902 till his death
have elongated somewhat narrow carapaces and yellow (1915).
heads. Shells are typically a pale tannish-yellow to ◎ He was instrumental in the formation of the Minto-
caramel color, with blotches of black. Morley Reforms of 1909, which eventually became law.
• IUCN Red List Status ➔ Critically Endangered.
• Legally protected ➔ Schedule IV of the Indian Wildlife Related Societies and Other Works ➔
(Protection) Act, 1972 as amended up to 2006. • He established the Servants of India Society in 1905 for
• Population ➔ According to the IUCN the population of the expansion of Indian education.
• He was also associated with the Sarvajanik sabha journal
the species may have fallen by about 80% in the last three
started by Govind Ranade.
generations (90 years).
• In 1908, Gokhale founded the Ranade Institute of
• Threats ➔ It is heavily hunted for food and collected Economics.
both for local use, such as decorative masks, and • He started english weekly newspaper, The Hitavada
international wildlife trade. In china, a mixture, made by (The people's paper).
grinding up the tortoise's shell, also serves as an • Published a daily newspaper entitled Jnanaprakash,
aphrodisiac. which allowed him to voice his reformist views on politics
and society.
Q. 2) Consider the following statement: • He contributed articles to the English weekly Mahratta.
1. He was associated with the moderate group of Indian • He served as Secretary of the Deccan Education Society.
National Congress and became president of INC in 1905
Mentor to Gandhi ➔
in Banaras session.
2. He was instrumental in the formation of the Minto- ◎ As a liberal nationalist, he is regarded by Mahatma
Morley Reforms of 1909. Gandhi as his political guru.
3. He founded the Ranade Institute of Economics in 1908. • Gandhi wrote a book in Gujarati dedicated to the leader
Choose the correct answer from the statements given titled ‘Dharmatma Gokhale’.
above:
(a) Mahadev Govind Ranade Q.3) With reference to Sample Registration System
(b) Lala Lajpat Rai recently seen in news, Consider the following statements:
1. It aims to provide reliable estimates of birth and death

45 | P a g e
rates for the States and also at all India level. Census of India and Linguistic Survey of India.
2. The sample units, villages in rural areas and urban • Registrar ➔ The position of Registrar is usually held by
blocks in urban areas are replaced once in ten years in this a civil servant holding the rank of Joint Secretary.
survey.
Which of the above statement is/are correct Q.4) Which of the following is/are correct regarding
(a) 2 only Maharana Pratap:
(b) 1 only 1. He was the King of Mewar and was the eldest son of
(c) Both 1 and 2 Udai Singh II.
(d) Neither 1 nor 2 2. He built a new capital known as Chavand near modern
Dungarpur.
Answer :(c) Both 1 and 2 Select the correct code given below:
Explanation- The Registrar General of India released its (a) 1 only
Sample Registration System (SRS) bulletin based on data (b) 2 only
collected for 2018. (c) Both 1 and 2
(d) Neither 1 nor 2
Key highlights
Birth rate ➔ The national birth rate in 2018 stood at 20 Answer (c) Both 1 and 2
per one thousand of the population. Bihar has the highest Explanation: Recently birth anniversary of Maharana
birth rate at 26.2. Pratap is celebrated , the 13th
◎ Death Rate ➔ The national death rate in 2018 stood at Rajpur king of Mewar. Maharana Pratap was born in
6.2 per one thousand of the population. Chhattisgarh has 1540 and died at the age of 56 in 1597.
the highest death rate at 8.
Infant mortality rate (IMR) ➔ The national IMR in 2018 About Maharana Pratap
stood at 32 per one thousand of the population. The IMR • Description ➔ Rana Pratap Singh also known as
at an all-India level has declined from 50 to 32 in the last Maharana Pratap was born on May 9th 1540 in
decade. Madhya Pradesh has an IMR of 48 and Nagaland Kumbhalgarh, Rajasthan.
4. • He was the 13th King of Mewar and was the eldest son
of Udai Singh II.
Sample Registration System
• Initiated In ➔ the late 1960s. Battle of Haldighati ➔
• Aim ➔ to provide reliable estimates of birth and death • It was fought on 18 June 1576 between the forces
rates for the States and also at All India level. Maharana Pratap; and the Mughal emperor Akbar's
forces, led by Man Singh I of Amber.
• Functions ➔ At present, the Sample Registration
• The Mughals were the victors but failed to capture
System (SRS) provides reliable annual data on fertility
Pratap, who escaped.
and mortality at the state and national levels for rural and
urban areas separately. • Resurgence ➔ Mughal pressure on Mewar relaxed after
• In this survey, the sample units, villages in rural areas 1579 following rebellions in Bengal and Bihar.
and urban blocks in urban areas are replaced once in ten • Taking advantage of the situation, Pratap recovered
years. Western Mewar including Kumbhalgarh, Udaipur and
• The data largely reflects past trends as far as the position Gogunda.
of states is concerned with marginal improvements at • During this period, he also built a new capital, Chavand,
national level on health indices. near modern Dungarpur.
• Timeline ➔ Initiated on a pilot basis by the Registrar • Chetak ➔ Chetak is the name given in traditional
General of India in a few states in 1964-65, it became literature to the horse ridden by Maharana Pratap at the
fully operational during 1969-70. Battle of Haldighati. However, some Historians debate it.
• Pratap Gaurav Kendra ➔ It is a tourist spot at Tiger Hill
About Registrar General of India in Udaipur city, Rajasthan. It aims at providing
• Formation ➔ 1961 information about Maharana Pratap and the historical
heritage of the area with the help of modern technology.
• Parent organization ➔ Ministry of Home Affairs.
• Functions ➔ It arranges, conducts and analyses the
results of the demographic surveys of India including Q.5) Naku La Pass, Nathu La Pass and Jelep La Pass

46 | P a g e
sometimes seen in news is in: by Mitsubishi Heavy Industries.
(a) Uttrakhand • Significance ➔ Observational data suggests the
(b) Sikkim existence of water in the polar regions of Moon. The
(c) Himachal Pradesh mission intends to obtain data on the quantity and forms
(d) Assam of water resources present, in order to determine the
feasibility of utilizing such resources for sustainable
Answer (b) Sikkim space exploration activities in the future.
Explanation- Recently, Indian and Chinese troops
engaged in a temporary and short duration face-off along Q. 7) Which of following statements is/are correct about
the Line of Actual Control (LAC) at Naku La (Sikkim) Ukai Dam:
and near Pangong Tso Lake (Eastern Ladakh). 1. It is the second-largest reservoir in Gujarat after Sardar
Sarovar Dam which is built on Sabarmati river.
About Naku la 2. It is an earth-cum-masonry dam which was constructed
• Naku La sector is a pass at a height of more in 1972.
than 5,000 metres above Mean Sea Level (MSL) in the Choose the correct code given below
state of Sikkim. (a) 1 only
• It is located ahead of Muguthang or Cho Lhamu (source (b) 2 only
of River Teesta). (c) Both 1 and 2
• At Muguthang, the road on the Chinese side is (d) Neither 1 nor 2
motorable, and on the Indian side, it is a
remote area. Answer (b) 2 only
-The other passes located in the state of Sikkim are Nathu Explanation: First statement is NOT correct because It is
La Pass and Jelep La Pass, Cho-la pass, Dongkia Pass , the second-largest reservoir in Gujarat after Sardar
Chiwabhanjang Pass. Sarovar Dam which is built on Tapi River.
• Recently, the water level of Ukai Dam has stood at
Q.6) Consider the following statements with reference to 319.86 feet after 46 years.
LUNAR POLAR EXPLORATION (LPE) MISSION :
About Ukai Dam
1. It is a joint lunar mission between Russia and India. • It is the second-largest reservoir in Gujarat after Sardar
2. The mission aims to put a lander and Sarovar Dam which is built on Tapi River.
rover on Moon’s surface and mission will be launched • It is also known as Vallabh Sagar.
after 2023. • It is an earth-cum-masonry dam which was
Which of the following statements is/are correct: constructed in 1972.
(a) 1 only
(b) 2 only Tapi River
(c) Both 1 and 2 • It is one of the three peninsular Rivers in India that flow
(d) Neither 1 nor 2 in an east to the west itinerary and the other two rivers are
the Mahi River and Narmada River.
• Origin ➔ It originates in the eastern portion of the
Answer (b) 2 only Satpura Mountain Ranges in south Madhya Pradesh.
Explanation- First Statement is NOT correct because • It pours into the Gulf of Cambay in the Arabian Sea in
Japan and India will launch a joint lunar mission called the district of Surat in Gujarat.
the Lunar Polar Exploration (LPE).
• Tributaries ➔ Girna River, Purna River, Panzara River,
• Aim ➔ The mission aims to put a lander and Bori River, Waghur River and Aner River are the major
rover on Moon’s surface. tributaries of the river.
• Launch year ➔ The mission will be launched
after 2023. Q.8) Recently launched, Aarogya Path aims to provide
• Japanese space agency JAXA would be building the real-time availability of critical healthcare supplies for
overall landing module and the rover, while ISRO would manufacturers, suppliers and customers. It is National
develop the lander system. Healthcare Supply Chain Portal of which of the following
• The mission will be launched from Japan, and the :
designated launch vehicle is the H3 rocket, manufactured (a) Indian Council of Medical Research(ICMR)

47 | P a g e
(b) Department of Science & Technology
(c) National Accreditation Board for Testing and
Calibration Laboratories
(d) Council of Scientific and Industrial Research

Answer : (d) Council of Scientific and Industrial Research


Aarogya Path
• Aarogya Path is an information platform launched by
union government, it aims to provide real-time
availability of critical healthcare supplies for
manufacturers, suppliers and customers.
• It is a Council of Scientific & Industrial Research
(CSIR) National Healthcare Supply Chain Portal.
• It is an integrated public platform that provides single-
point availability of key healthcare goods such as medical
equipment, drugs, apparel, etc.
• It will set up an information management and
forecasting database platform at national level to capture
demand and supply scenarios for key healthcare needs
items.
• CSIR expects AarogyaPath to become the national
healthcare information platform of choice in the years to
come, filling a critical gap in last-mile delivery of patient
care within India through improved availability and
affordability of healthcare supplies.
• It will also create opportunities for business expansion
due to an expanded network of buyers and visibility of
new requirements for products.

Council of Scientific & Industrial Research


• It was established by the Government of India in
September 1942 as an autonomous body. It comes under
the Ministry of Science & Technology.
• It is known for its cutting edge research and
development knowledge base in diverse science and
technology areas. Q. 10) Consider the following statements with reference
• It has been ranked first in the Nature Ranking Index- to Prairies:
2020. 1. Prairies are part of Great plains in North America
between the mountain ranges of Rocky mountain and
Q. 9) Arrange the following physical feature of North Appalachian mountains.
America from West to East: 2. They are temperate grasslands with nutritious grass
1. Colorado Plateau known for wheat cultivation and animal rearing.
2. Canadian Shield Choose the correct statement from the code given below
3. Great Plains (a) 1 only
Choose the correct code given below (b) 2 only
(a) 1,2,3 (c) Both 1 and 2
(b) 3,1,2 (d) Neither 1 nor 2
(c) 2,1,3
(d) 2,3,1 Answer (c) Both 1 and 2

Answer(b) 1,3,2

48 | P a g e
1. Sohrai Khovar painting - Uttar Pradesh
2. Telia Rumal - Telangana
3. Arumbavur wood carvings - Tamil Nadu
Which of the following pairs given above is/are correctly
matched:
(a) 2 only
(b) 1 and 3 only
(c) 2 and 3 only
(d) 1, 2 and 3

Answer (c) 2 and 3 only


Explanation : Sohrai Khovar painting belongs to
Jharkhand
• Recently, Jharkhand’s Sohrai Khovar painting and
Telangana’s Telia Rumal, Thanjavur Netti works,
Arumbavur wood carvings of Tamil Nadu received the
prestigious GI tag.

About SOHRAI KHOVAR PAINTING


• What is it? ➔ The Sohrai Khovar painting is a
traditional and ritualistic mural art.
• Practiced ➔ It is being practiced by local
tribal women during local harvest and marriage seasons
using local, naturally
available soils of different colors in the area
of Hazaribagh district of Jharkhand.
• Features ➔ Style features a profusion of lines, dots,
animal figures and plants, often representing religious
iconography i.e.visual image and symbols.
• Where is it painted? ➔ It has been painted on the walls
of important public places in Jharkhand, such as the Birsa
Explanation : Munda Airport in Ranchi.
• The continent has Rocky mountain ranges to western
side and Appalachian mountains to the eastern coast and About Telia Rumal Handloom
In between the ranges there is the Great plains are present
◎ What is it? ➔ Telia Rumal cloth involves intricate
called Prairies “Granaries of the World”.
• Prairies lie in USA and 3 states of Canada. handmade work with cotton loom displaying a variety of
• They are temperate grasslands with nutritious grass designs and motifs in three particular colors such as red,
known for wheat cultivation and animal rearing. black and white.
• The Great Plains is a broad expanse of flat land (a plain), • Features ➔ It is an art of Ikat tradition using natural
much of it covered in prairie, steppe, and grassland, vegetable dyes.
located in the United States and Canada. • Ikkat is a dyeing technique used to pattern textiles.
• The region is known for supporting extensive cattle • Uniqueness ➔ Telia Rumal can only be created using
ranching and dry farming. the traditional handloom process and not by any other
mechanical means as otherwise, the very quality of the
Rumal would be lost.
20 JUNE • History ➔ The telia fabrics were used by nobles
(Nizam’s dynasty) in Hyderabad. The fabric was exported
Q. 1) Consider the following statements Pairs: to Persian Gulf, Middle East, East Africa, Singapore and
Geographical State Burma.
Indication

49 | P a g e
Arumbavur wood carvings • What it is? ➔ It is a technology driven Control Room-
• The Arumbavur wood carvings made by the artisans of Cum-Management Information System which utilises
Perambalur are mainly made out of wooden logs of modern information and communication technology
lingam tree, mango, Indian ash tree, neem and rosewood (ICT) tools.
tree. • Name ➔ The name CHAMPIONS stands here for
• The wood supply comes from the Pachamalai Hills Creation and Harmonious Application of Modern
along the Trichy - Perambalur boundary and the Processes for Increasing the Output and National
Thanjavur - Kumbakonam zone. Strength.
• The important feature of Arumbavur work is that the
• Aim ➔ The system utilising modern ICT tools is aimed
whole object is carved out of a single block of wood due
at assisting Indian MSMEs march into big league as
to the reason that even one mistake can damage the whole
National and Global CHAMPIONS.
work.
• Features ➔ The portal is for making the smaller units
Thanjavur Netti Works big by solving their grievances, encouraging, supporting,
• Thanjavur Netti Works (Thanjavur Pith Work) is crafted helping and handholding. It is a real one-stop-shop
from Netti (pith), a marshy plant known as solution of MSME Ministry.
Aeschynomene Aspera. • Hub & Spoke Model ➔ As part of the system a network
• Being a traditional craft, it has been handed down from of control rooms is created in a Hub & Spoke Model. The
one generation to another. Hub is situated in New Delhi in the Secretary MSME’s
• Netti Works are visible in the Brihadeeswara Temple, office. The spokes will be in the States in various
garlands, idols of Hindu deities, door hangings and offices and institutions of Ministry.
decorative items. • The portal is also fully integrated with the Government’s
• The pith work industry has been recognised as one of the main grievances portal CPGRAMS and MSME
significant handwork symbols. Ministry’s web based mechanisms.
• Netti works are produced in Thanjavur, Kumbakonam
and Pudukkottai. About Centralized Public Grievance Redress And
• Due to their fragile nature, the Pith works are kept inside Monitoring System (CPGRAMS):
a glass box to preserve them well. • What is it? ➔ It is an online web-enabled system
developed by National Informatics Centre in association
Q. 2) Consider the following statement about with Directorate of Public Grievances (DPG) and
CHAMPIONS PORTAL recently seen in news : Department of Administrative Reforms and Public
1. It is a technology driven Control Grievances(DARPG).
Room-Cum-Management Information System. • Aim ➔ To enable submission of grievances by the
2. It aims to help consumers register complaints easily and aggrieved citizens to Ministries or departments who
get redressal within 60 days. scrutinize and take action for speedy and favorable
Choose the correct answer from the statements given redress of these grievances
above:
(a) 2 only Q.3) With reference to Malabar flying frog recently seen
(b) 1 only in news, Consider the following statements:
(c) Both 1 and 2 1. It is with slender body ,webbed feet and endemic to the
(d) Neither 1 nor 2 rainforests of western ghats.
2. Its IUCN Conservation status is Least Concern.
Answer (b) 1 only Which of the above statement is/are correct
Explanation- Second statement is NOT correct because it (a) 2 only
aimed at assisting Indian MSMEs march into big league (b) 1 only
as National and Global CHAMPIONS. (c) Both 1 and 2
• Union Ministry of MSME has launched CHAMPIONS (d) Neither 1 nor 2
portal www.Champions.gov.in, a Technology driven
Control Room-Cum- Answer :(c) Both 1 and 2
Management Information System. Explanation- Recently, a rare amphibian, Malabar
Gliding Frog (Rhacophorus
About CHAMPIONS Portal malabaricus) was spotted in Pullad, Kerala.

50 | P a g e
• Sometimes called rainforests of the sea, shallow coral
About Malabar Gliding Frog reefs form some of Earth's most diverse ecosystems.
• Scientific name ➔ Rhacophorus malabaricus. • They are most commonly found at shallow depths in
• It is aslo known as Malabar flying frog tropical waters, but deep water and cold water coral reefs
• It is endemic to the rain forests of Western exist on smaller scales in other areas.
Ghats. • Coral reefs deliver ecosystem services for tourism,
• Features ➔ It is a green frog with slender body, webbed fisheries and shoreline protection.
feet, unusual body positions, very well camouflaged and Coral reefs are fragile, partly because they are sensitive to
gliding in the air. water conditions.
• They are under threat from excess nutrients (nitrogen
Q.4) Which of the following is/are correct regarding and phosphorus),rising temperatures, oceanic
Coral reefs: acidification, overfishing (e.g., from blast fishing,
1. Corals secrete hard carbonate exoskeletons that support cyanide fishing, spearfishing on scuba), sunscreen use,
and protect the coral. and harmful land-use practices, including runoff and
2. Most reefs grow best in warm, shallow, clear, sunny seeps (e.g., from injection wells and cesspools).
and agitated water.
Select the correct code given below: The distribution of coral reefs in India
(a) 1 only
(b) 2 only
(c) Both 1 and 2
(d) Neither 1 nor 2

Answer (c) Both 1 and 2


Explanation: According to recent study Cyclones
Amphan and Nisarga that unleashed destruction in eastern
and western India have saved the Gulf of Mannar corals
from mass bleaching as windstorms along with two low
pressures have significantly lowered seawater
temperature in summer.
• The Gulf of Mannar (GoM), spread around 21 islands,
suffers significant damage caused by livelihood-linked
human threats and climate change.
• The islands and the reef areas collectively constitute the
Gulf of Mannar Marine National Park (GMMNP).
• Corals in the GoM usually bleach in summer if the water
temperature surpasses 30˚C, but they recover when it
drops in August. Q. 5) Global Nutrition Report 2020 is recently released
by:
About Coral reefs (a) World Health Organisation
• A coral reef is an underwater ecosystem characterized (b) International Food Policy Research Institute
by reef-building corals. (c) Food and Agriculture Organization
• Reefs are formed of colonies of coral polyps held (d) Nutrition International
together by calcium carbonate.
• Most coral reefs are built from stony corals, whose Answer (a) World Health Organisation
polyps cluster in groups. Explanation- The Global Nutrition Report 2020 has
• Coral belongs to the class Anthozoa in the animal released recently which identifies India as having the
phylum Cnidaria, which includes sea anemones and highest rates of inequalities in malnutrition.
jellyfish.
• Unlike sea anemones, corals secrete hard carbonate About Global Nutrition Report
exoskeletons that support and protect the coral. • Released by ➔ World Health Organisation (WHO)
• Most reefs grow best in warm, shallow, clear, sunny and • Timeline ➔ The Global Nutrition Report was conceived
agitated water. following the first Nutrition for Growth Initiative Summit

51 | P a g e
(N4G) in 2013.
• The first report was published in 2014. Regional Maritime Information Fusion Centre
• It acts as a report card on the world’s nutrition— (RMIFC):
globally, regionally, and country by country—and on • It functions under the aegis of the Indian
efforts to improve it. Ocean Commission (IOC) and is based in
• It is a multi-stakeholder initiative, consisting of a Madagascar.
Stakeholder Group, Independent Expert Group and • It is designed to deepen maritime domain
Report Secretariat. awareness by monitoring maritime activities and
promoting information sharing and exchange.
Key Takeaways
• India is among the 88 countries that are likely to miss The European maritime surveillance initiative in the Strait
global nutrition targets by 2025. of Hormuz (EMASOH)
• Stunting and wasting among children ➔ 37.9% of • It is composed of Belgium, Denmark, Netherlands and
children under 5 years are stunted and 20.8% are wasted French officers and based at the French naval base in Abu
compared to the Asia average of 22.7% and 9.4% Dhabi.
respectively. • The aim is to monitor maritime activity and guarantee
• Overweight and Obesity ➔ Rate of overweight and freedom of navigation in the Persian Gulf and the Strait
obesity continues to rise affecting almost a fifth of the of Hormuz.
adults at 21.6% of women and 17.8% of men. How this will help India? ➔ This will be in the overall
• Anaemia ➔ One in two women of reproductive age is realm of improving linkages of the Navy’s Information
anaemic. Fusion Centre for Indian Ocean Region (IFC-IOR) in
Gurugram with other IFCs and become the repository for
• Underweight children ➔ Between 2000 and 2016, rates
all maritime data in the IOR.
of underweight have decreased from 66.0% to 58.1% for
boys and 54.2% to 50.1% in girls. However, it is still high
Indian Ocean Commission(IOC):
compared to the average of 35.6% for boys and 31.8% for
• It is an intergovernmental organization that was created
girls in Asia.
in 1982 at Port Louis, Mauritius and institutionalized in
1984 by the Victoria Agreement in Seychelles.
Q.6) Consider the following statements with reference to
Regional Maritime Information Fusion Centre (RMIFC) : • Member States ➔ Comoros, Madagascar, Mauritius,
Réunion (an overseas region of France) and Seychelles.
1. It is designed to deepen maritime domain • Observer States ➔ China, Malta, European Union,
awareness by monitoring maritime activities and International Organisation of La Francophonie(OIF) and
promoting information sharing and exchange. now India.
2. It functions under the aegis of the International • Secretariat ➔ located in Mauritius.
Maritime Organization. • Objectives of IOC ➔ Political and diplomatic
cooperation, Economic and commercial cooperation,
Which of the following statements is/are correct: Sustainable development in a globalisation context,
(a) 1 only cooperation in the field of agriculture.
(b) 2 only
(c) Both 1 and 2 Information Fusion Centre – Indian Ocean Region(IFC-
(d) Neither 1 nor 2 IOR):
•Established ➔ in 2018 at Gurugram, Haryana.
• Aims ➔ to advance Maritime Safety and Security in the
Answer (a) 1 only
Indian Ocean Region through information sharing with
Explanation- Second Statement is NOT correct because It
partner nations and agencies.
functions under the aegis of the Indian Ocean
• It is jointly administered by the Indian Navy and Indian
Commission (IOC) and is based in Madagascar.
Coast Guard.
• It is in news because India is looking to post Navy
Liaison Officers at the RMIFC in
Q. 7) Which of following statements is/are correct about
Madagascar and also at the European maritime
Amoebiasis:
surveillance initiative
1. It is a parasitic infection of the colon with the amoeba
in the Strait of Hormuz for improved MDA.

52 | P a g e
Entamoeba histolytica.
2. The protozoan causing the infection cannot survive Etalin Hydropower Project
high concentrations of oxygen. •Etalin Hydropower Project is based on the river Dibang,
Choose the correct code given below it is also known as Dibang Valley project.
(a) 1 only • Dibang is a tributary of the Brahmaputra river which
(b) 2 only flows through the states of Arunachal Pradesh and Assam.
(c) Both 1 and 2 • It envisages construction of two dams over the
(d) Neither 1 nor 2 tributaries of Dibang: Dir and Tangon.

Answer (c) Both 1 and 2 Significance ➔


Explanation: A team of researchers from the Jawaharlal • The Project is in accordance with the Government’s
Nehru University (JNU) has push to establish prior user rights on rivers that originate
developed new drug molecules against the protozoa that in China and an effort to fast- track projects in the north-
causes amoebiasis.. east.
• It is expected to be one of the biggest hydropower
About Amoebiasis projects in India in terms of installed capacity.
• What is it? ➔ It is a parasitic infection of the colon with
the amoeba Entamoeba histolytica. Concerns ➔
• According to WHO, Entamoeba histolytica is the third- - The Project falls under the “richest bio-geographical
leading cause of morbidity and mortality due to parasitic province of the Himalayan zone” and would be located at
disease in humans. the junction of the Palaearctic, Indo-Chinese and Indo-
• This protozoan is anaerobic or micro-aerophilic in Malayan bio-geographic regions.
nature such that it cannot survive high concentrations of - A total of 18 villages consisting of 285 families are
oxygen. expected to be affected by the proposed project.
• However, during infection, it faces a high surge of - It would entail felling 2,80,677 trees and threatens the
oxygen inside the human body. The organism synthesizes existence of globally- endangered mammal species.
large amounts of cysteine to counter oxidative stress. • According to the decision the 3097 MW Etalin
• Spread of the disease ➔ It spreads through drinking or Hydropower project, in the State’s Dibang Valley, has
eating uncooked food, such as fruit, that may have been been delayed for over six years.
washed in contaminated local water. • This is because it required diverting 1165 hectares of
• Symptoms ➔ forest in a region of rich biodiversity.
- Pain areas: in the abdomen
- Gastrointestinal: blood in stool, diarrhoea, or flatulence Q. 9) The Mojave or Mohave Desert is situated between
- Whole body: fatigue, fever, or loss of appetite which of the following two deserts given below:
- Also common: weight loss (a) Great Basin Desert and Sonoran Desert
• Treatment ➔ Treatment consists of self care and (b) Sonoran Desert and Chihuahuan Desert
antiparasitics. (c) Great Basin Desert Chihuahuan Desert
(d) Great Sandy Desert and Sonoran Desert
Q.8) Etalin Hydropower Project sometimes seen in news
is based on the river: Answer (a) Great Basin Desert and Sonoran Desert
(a) Brahmaputra Explanation :
(b) Subansiri River
(c) Dibang • The Mojave Desert is bordered by the Great Basin
(d) Brahmani River Desert to its north and the Sonoran Desert to its south and
east.
Answer : (c) Dibang • The Mojave Desert is an arid rain-shadow desert and the
driest desert in North America.
Explanation
• The Forest Advisory Committee, the apex body of the Great Basin Desert
Environment Ministry, has once again deferred its
decision on a controversial
hydropower project in Arunachal Pradesh.

53 | P a g e
• The Great Basin Desert is part of the Great Basin • With an area of about 362,000 km2 (139,769 sq mi), it
between the Sierra Nevada and the Wasatch Range. is the second largest desert of the Americas and the largest
in North America.

Q. 10) Arrange the following Bay/Gulf from North to


South:
(a) Baffin Bay, Gulf of Mexico, Hudson Bay
• The desert is a geographical region that largely overlaps (b) Gulf of Mexico, Hudson Bay, Baffin Bay
the Great Basin shrub steppe • It is a temperate desert with (c) Baffin Bay, Hudson Bay, Gulf of Mexico
hot, dry summers and snowy winters. (d) Hudson Bay, Baffin Bay, Gulf of Mexico
• Its boundaries are generally noted by the presence of
Joshua trees, which are native only to the Mojave Desert Answer (c) Baffin Bay, Hudson Bay, Gulf of Mexico
and are considered an indicator species, and it is believed
to support an additional 1,750 to 2,000 species of plants.

Chihuahuan Desert
• The Chihuahuan Desert is a desert and ecoregion
designation covering parts of northern Mexico and the
southwestern United States.

54 | P a g e
22 JUNE Answer (c) 3 only
Explanation- Third statement is NOT correct because
Q. 1) Angiotensin Converting Enzyme 2 (ACE2) often IUCN Conservation Status of Indian pangolin is
seen in news is/are found in: Endangered.
1. Lungs • China has accorded the pangolin the highest level of
2. Heart protection and removed the scales of the Pangolin from
3. Intestines its list of approved
4. Blood vessels traditional medicines.
5. Hypothalamus
Choose the correct code given below About Pangolin
(a) 1 and 2 only • They are scaly anteater mammals of the order Pholidota.
(b) 4 and 5 only • They have large, protective keratin scales covering their
(c) 1, 2,3 and 5 only skin and they are the only known mammals with this
(d) 1, 2 ,3 ,4 and 5 feature.
• Out of the eight species of pangolin, the Indian Pangolin
Answer (d) 1, 2 ,3 ,4 and 5 and the Chinese Pangolin are found in India:
Explanation : Recently, several studies revealed that men • Indian Pangolin → It is widely distributed in India,
are more vulnerable to COVID-19 infection due to higher except the arid region, high Himalayas and the North-
concentrations of Angiotensin-Converting Enzyme 2 East.
(ACE2) in their blood than women. • The species is also found in Bangladesh, Pakistan, Nepal
and Sri Lanka.
About Angiotensin-Converting Enzyme 2 (ACE2) • Chinese Pangolin → It is found in Himalayan foothills
• What is it? ➔ Angiotensin Converting Enzyme in Eastern Nepal, Bhutan, Northern India, North-East
2(ACE2) is an enzyme molecule that connects the inside Bangladesh, through Southern China and Taiwan.
of our cells to the outside via the cell membrane. • IUCN Conservation Status ➔
• It is an enzyme attached to the cell membranes of cells ○ Indian pangolin – Endangered
in the lungs, arteries, heart, kidney, and intestines. ○ Chinese pangolin – Critically Endangered
• Where is it found? ➔ The enzyme is found in our lungs, •Threats:
heart, intestines, blood vessels muscles, the cerebral a) Hunted for its meat across the northeastern States and
cortex, striatum, hypothalamus, and brainstem. in central India. and
• The enzyme essentially acts as a port of entry that allows b) Smuggled for scales especially in China as it is
the coronavirus to invade our cells and replicate. believed that scales possess magic or charms and have
• Since men have higher concentrations of ACE2 medicinal properties.
compared to women, they are more vulnerable to
coronavirus. More About Indian Pangolin
• The Indian pangolin is listed on CITES Appendix I since
Q. 2) Which of following statement is INCORRECT January 2017 and is protected in all range countries.
regarding Indian Pangolin: • The Indian pangolin is a solitary, shy, slow-moving,
1. It is a solitary, shy, slow-moving, nocturnal mammal. nocturnal mammal.
2. The pangolin has no teeth, but has strong stomach • The pangolin has no teeth, but has strong stomach
muscles to aid in digestion. muscles to aid in digestion.
3. IUCN Conservation Status of Indian pangolin is • It prefers soft and semi-sandy soil conditions suitable for
critically Endangered. digging burrows.
4. It is widely distributed in India, except the arid region, • The Indian pangolin does not climb trees, but it does
high Himalayas and the North-East. value the presence of trees, herbs, and shrubs in its habitat
Choose the correct answer from the statements given because it is easier to dig burrows around them.
above:
(a) 1, 2 and 4 only Q.3) With reference to Global Partnership on Artificial
(b) 2 only Intelligence (GPAI) recently seen in news, Consider the
(c) 3 only following statements:
(d) 2 and 3 only 1. India is one of its founding member and it will help

55 | P a g e
India to actively participate in the global development of and Malnad regions of Karnataka and Kerala.
Artificial Intelligence. • It is a derived form of Yakshagana—a classical dance or
2. It is an international and multi- stakeholder initiative to musical form of art from the same region.
guide the responsible development and use of AI. • How is it different from Yakshagana? ➔
Which of the above statement is/are correct Unlike the Yakshagana performance, in the conventional
(a) 2 only ‘talamaddale,’ the artists sit across in a place without any
(b) 1 only costumes and engage in testing their oratory skills based
(c) Both 1 and 2 on the episode chosen.
(d) Neither 1 nor 2 • The music is common for both Yakshagana performance
and ‘talamaddale’, the latter has only spoken word
Answer :(c) Both 1 and 2 without any dance or costumes.
Explanation- India joins Global Partnership on Artificial • Hence it is an art form minus dance, costumes and stage
Intelligence (GPAI) as a conventions.
founding member to support the responsible and human-
centric development and use of AI. More to know
• Yakshagaana, traditional theatre form of Karnataka, is
About GPAI based on mythological stories and Puranas.
• What is it? ➔ It is an international and multi- • The most popular episodes are from the Mahabharata i.e.
stakeholder initiative to guide the responsible Draupadi swayamvar, Subhadra vivah, Abhimanyu vadh,
development and use of AI, grounded in human rights, Karna-Arjun yuddh and from Ramayana i.e.
inclusion, diversity, innovation, and economic growth. Rajyabhishek, Lav-Kush yuddh, Baali-Sugreeva yuddh
• This is also a first initiative of its type. and Panchavati.
• Secretariat ➔ GPAI will be supported by a Secretariat, • Gombeyatta puppet theatre closely follows Yakshagana.
to be hosted by Organization for Economic Cooperation • Other important forms of theatre in India:
and Development (OECD) in Paris, as well as by two 1) Nautanki (Uttar Pradesh) which often draws on
Centers of Expertise- one each in Montreal and Paris. romantic Persian literature for its themes,
• Founding members ➔ Australia, Canada, France, 2) Tamasha (Maharashtra),
Germany, India, Italy, Japan, Mexico, New Zealand, the 3) Bhavai (Gujarat),
Republic of Korea, Singapore, Slovenia, the United 4) Jatra (West Bengal),
Kingdom, the United States of America, and the European 5) Koodiyaattam, one of the oldest traditional theatre
Union. forms of Kerala, is based on Sanskrit theatre traditions,
5) Mudiyettu, traditional folk theatre form of Kerala,
• Objectives ➔ In collaboration with partners and
6) Bhaona, Assam,
international organizations, GPAI will bring together
7) Maach, Madhya Pradesh,
leading experts from industry, civil society, governments,
8) Bhand Pather, the traditional theatre form of Kashmir,
and academia to collaborate to promote responsible
etc.
evolution of AI.
Q. 5) Which of the following has released the annual
Q.4) The traditional art of Talamaddale was streamed live
rankings of the global Energy Transition Index:
on social media recently. It is a variant of:
(a) International Energy Agency
(a) Mudiyettu
(b) Sustainable Energy for All
(b) Bhand Paather
(c) World Economic Forum
(c) Yakshagana
(d) International Gas Union
(d) Koodiyattam
Answer (c)World Economic Forum
Answer (c) Yakshagana theatre
Explanation- The Global Nutrition Report 2020 has
Explanation: The traditional art of Talamaddale, a variant
released recently which identifies India as having the
of Yakshagana theatre
highest rates of inequalities in malnutrition.
was streamed live on social media recently.
About Energy Transition Index
About Talamaddale
•Recently, the World Economic Forum (WEF) has
• Talamaddale is an ancient form of performance dialogue
released the annual rankings of the global Energy
or debate performance in Southern India in the Karavali

56 | P a g e
Transition Index. About Nila River:
• The index benchmarks 115 economies on the current • Bharathappuzha also known as the Nila and ponani
performance of their energy systems across river, is a river in India in the state of Kerala.
1)Economic development and growth, 2)Environmental • Nila has groomed the culture and life of south Malabar
sustainability and energy security part of Kerala.
3)Access indicators • Kannadipuzha originating from Thrimoorthy Hills of
4)Readiness for transition to secure, sustainable, Anamalais in Tamil Nadu joins with Kalpathipuzha at
affordable and inclusive energy systems. Parali in Palakkad District and forms Nila or
• According to the recent report Sweden has topped the Bharathappuzha river.
Index for the third consecutive year and is followed by • It is also referred to as "Peraar" in ancient scripts and
Switzerland and Finland in the top three. documents.
• Only 11 out of 115 countries have made steady • River Bharathapuzha is an interstate river and lifeline
improvements in ETI scores since 2015. water source for a population residing in four
Argentina, China, India and Italy are among the major administrative districts, namely Malappuram, Thrissur
countries with consistent annual improvements. and Palakkad districts of Kerala and Coimbatore, and
• In China (78th), problems of air pollution have resulted Tiruppur of Tamil Nadu.
in policies to control emissions, electrify vehicles and • The Bharathappuzha is extensively dammed and here
develop the world’s largest capacity for solar photovoltaic are 11 reservoirs along the course of the river.
(SPV) and onshore wind power plants. Malampuzha dam is the largest among the reservoirs built
• Scores for the US, Canada, Brazil and Australia were across Bharathapuzha and its tributaries. Other dams in
either stagnant or declining. the Bharathapuzha basin are Walayar Dam, Mangalam
• India has moved up two positions to rank 74th with Dam, Pothundi Dam, Meenkara Dam, Chulliyar Dam,
improvements in all three dimensions of the energy Thirumoorthy, Aliyar, Upper Aliyar, Chitturpuzha
triangle namely: regulator and Kanjirapuzha Dam
1)Economic development and growth. • It flows westward through Palakkad Gap (most
2) Energy access and security. prominent discontinuity in the Western Ghats) and drains
3) Environmental sustainability. into the Arabian Sea.
• For India, gains have come from a government- • Main Tributaries:
mandated renewable energy expansion programme i.e. to 1)Kannadipuzha (Chitturpuzha),
add 275 GW by 2027. 2)Kalpathipuzha (Korapuzha),
• India has also made significant strides in energy 3)Gayathripuzha
efficiency through bulk procurement of LED bulbs, smart 4)Thuthapuzha
meters and programs for labelling of appliances. Similar
measures are being experimented to drive down the costs Q. 7) Which of following statements is/are correct about
of electric vehicles (EVs). Ultraviolet Light:
1. UV light from the sun has shorter wavelengths and,
Q.6) The information given below is talking about which therefore is easily visible to the naked eye.
river: 2. UV-C rays are the most harmful and are completely
1. It is also known as Ponnani river. absorbed by the Earth’s atmosphere.
2. Malampuzha dam is the largest among the reservoirs 3. Exposure to UV-B rays can cause DNA and cellular
built across this river. damage in living organisms.
3. It is referred to as "Peraar" in ancient scripts and Choose the correct code given below
documents. (a) 1 and 2 only
Choose the correct answer given below: (b) 2 and 3 only
(a) Periyar River (c) 1 and 3 only
(b) Nila River (d) All are correct
(c) Tungabhadra River
(d) Kabini River Answer (b) 2 and 3 only
Explanation: First statement is NOT correct because UV
Answer (b) Nila River light from the sun has shorter wavelengths than visible
Explanation- Recently, a webinar ‘Exploring River Nila’ light and, therefore, is not visible to the naked eye.
was organized by Ministry of tourism. •It is in news because Scientists are studying the use of
ultraviolet germicidal radiation (UVGI) to detect

57 | P a g e
Coronavirus in schools, restaurants and other public Dhanbad and CSIR-Indian Institute of Chemical Biology
places. (Kolkata) have developed the Z-scan method to monitor
• Through this method, ultraviolet (UV) lights would be the origin as well as the progression of Parkinson's disease
able to disinfect contaminated public spaces to stop the in human beings.
transmission of the virus. • The discovered Z-scan method is expected to help in
UVGI uses the “destructive properties” of UV light to monitoring both the early as well as late stages of the
target pathogens. aggregation of ASyn and death of neuronal cells.
• UVGI replicates UV wavelengths that disinfects
contaminated spaces, air and water. Parkinson Disease
• UVGI lamps can also be installed in the corners of a • Parkinson’s disease is a chronic, degenerative
room and alternatively, can be installed in air ducts of neurological disorder that affects the central nervous
ventilation systems or portable or fixed air cleaners. system.
• It damages nerve cells in the brain dropping the levels
About UV rays of dopamine. Dopamine is a chemical that sends
• UV light from the sun has shorter wavelengths than behavioral signals from the brain to the body.
visible light and, therefore, is not visible to the naked eye. • The disease causes a variety of "motor" symptoms
• The full spectrum of UV radiation is sourced from the (symptoms related to movement of the muscles),
sun and can be subdivided into UV-A, UV-B and UV-C including rigidity, delayed movement, poor balance, and
rays. tremors.
• In this spectrum, UV-C rays are the most harmful and • Medication can help control the symptoms of the disease
are completely absorbed by the Earth’s atmosphere. but it can't be cured.
• While both UV-A and UV-B rays are harmful, exposure • It affects the age group from 6 to 60 years.
to UV-B rays can cause DNA and cellular damage in
living organisms. ASyn
• Increased exposure to it can cause cells to become • An aggregation of a protein called Alpha-synuclein
carcinogenic, thereby increasing the risk of getting (ASyn) plays a crucial role in the development of
cancer. Parkinson’s disease.
• Protein aggregation is a biological phenomenon in
Benefits of UV rays which destabilized proteins aggregate (i.e., accumulate
• UV light causes the body to produce vitamin D and clump together) leading to many diseases.
(specifically, UVB), which is essential for life. The • Alpha-synuclein is a protein found in the human brain,
human body needs some UV radiation to maintain while smaller amounts are found in the heart, muscle and
adequate vitamin D levels; however, excess exposure other tissues.
produces harmful effects that typically outweigh the • In the brain, alpha-synuclein is found mainly at the tips
benefits. of neurons in specialized structures called presynaptic
• UV rays also treat certain skin conditions. Modern terminals.
phototherapy has been used to successfully treat psoriasis, • The release of neurotransmitters relays signals between
eczema, jaundice, vitiligo, atopic dermatitis, and localized neurons and is critical for normal brain function.
scleroderma. • Until now, worldwide studies could not establish any
strong relation between ASyn aggregations and
Q.8) Recently, scientists from IIT (Indian School of subsequent death of neuronal cells observed in
Mines) Dhanbad and CSIR-Indian Institute of Chemical Parkinson’s disease.
Biology have developed the Z-scan method to monitor the
origin as well as the progression of: CSIR-Indian Institute of Chemical Biology
(a) Covid-19 • Indian Institute of Chemical Biology (IICB) was
(b) Alzheimer disease established in 1935 as the first non-official center in India
(c) Parkinson's disease for biomedical research and was included within the aegis
(d) Cancer of Council of Scientific and Industrial Research (CSIR) in
1956.
Answer : (c)Parkinson's disease • It is located in Kolkata (West Bengal).
• CSIR-IICB is engaged in research on diseases of
Explanation national importance and biological problems of global
• Recently, scientists from IIT (Indian School of Mines) interest and also helps to maintain momentum in life

58 | P a g e
science research. seven South American Country.
• It conducts research in a variety of areas including 1. Bolivia
chemistry, biochemistry, cell biology, molecular biology, 2. Paraguay
neurobiology and immunology which promotes 3. Peru
productive interdisciplinary interaction. 4. Brazil
Choose the correct code given below
Q. 9) Consider the following features of Andes (a) 1 and 2 only
Mountains: (b) 2 and 4 only
1. The Andes Mountains are the highest mountain range (c) 1 and 3 only
outside Asia. (d) 2 and 3 only
2. These are the longest continental mountain range in the Answer (b) 2 and 4 only
world along the eastern edge of South America.
3. The world's highest volcanoes are in the Andes, Explanation : The Andes extend from north to south
including Ojos del Salado on the Chile-Argentina border. through seven South American countries: Venezuela,
Which of the following feature is/are correct: Colombia, Ecuador, Peru, Bolivia, Chile, and Argentina.
(a) 1 and 2 only
(b) 2 and 3 only
(c) 1 and 3 only
(d) All are correct

Answer (c) 1 and 3 only


Explanation : Second statement is NOT correct because
Andes Mountains are the longest continental mountain
range in the world, forming a continuous highland along
the western edge of South America.

About Andes Mountains


• Andes Mountain Range (Fold Mountain) lies in South
American continent which is the second highest mountain
range of the world, after Himalayas.
• These ranges are in turn grouped into three major
divisions based on climate: the Tropical Andes, the Dry
Andes, and the Wet Andes.
• The Andes Mountains are the highest mountain range
outside Asia.
• The highest mountain outside Asia, is Argentina's
Mount Aconcagua which is highest peak in Andes
Mountains
• The peak of Chimborazo in the Ecuadorian Andes is
farther from the Earth's center than any other location on
the Earth's surface, due to the equatorial bulge resulting
from the Earth's rotation.
• The world's highest volcanoes are in the Andes,
including Ojos del Salado on the Chile-Argentina border.
• The Andes are also part of the American Cordillera, a
chain of mountain ranges (cordillera) that consists of an
almost continuous sequence of mountain ranges that form
the western "backbone" of North America, Central
America, South America and Antarctica.

Q. 10) The Andes extend from north to south through


seven South American countries. With reference to this
statement, which of the following is NOT part of the

59 | P a g e
Securities and Exchange Board of India (SEBI):
1. The chairman of SEBI is nominated by the Union
Government of India.
2. It can compel certain companies to list their shares in
one or more Securities exchanges.
3. It is a non-statutory body for regulating
securities and commodity market in India.
Choose the correct code given below
(a) 1 and 2 only
(b) 2 and 3 only
(c) 1 and 3 only
(d) All are correct

Answer (a) 1 and 2 only


Explanation : Third Statement is NOT correct because It
is a Statutory Body and given statutory powers through
the SEBI Act, 1992.

About SEBI
• It was first established in 1988 (originally formed in
1992) as a non-statutory body for regulating the securities
market.
• It was given Statutory Powers through the SEBI Act,
1992.
• It was constituted as the regulator of capital markets in
India under a resolution of the Government of India.
• After the amendment of 1999, collective investment
schemes were brought under SEBI except Nidhis, chit
funds and cooperatives.
• The SEBI is managed by its members, which consists of
the following:
1)The chairman is nominated by the Union Government
of India.
2) Two members, i.e., Officers from the Union Finance
Ministry.
3) One member from the Reserve Bank of India.
4) The remaining five members are nominated by the
Union Government of India, out of them at least three
shall be whole-time members.

Powers
• For the discharge of its functions efficiently, SEBI has
been vested with the following powers:
1)to approve by−laws of Securities exchanges.
2) to require the Securities exchange to amend their
by−laws.
3) inspect the books of accounts and call for periodical
returns from recognised Securities exchanges.
4) inspect the books of accounts of financial
intermediaries.
23 JUNE
5) compel certain companies to list their shares in one or
more Securities exchanges.
Q. 1) Which of the following statements is true about
• SEBI has three functions rolled into one body: quasi-

60 | P a g e
legislative, quasi-judicial and quasi-executive. membership to eight countries when India and Pakistan
• It drafts regulations in its legislative capacity, it conducts joined SCO as full members on 9 June 2017 at a summit
investigation and enforcement action in its executive in Astana, Kazakhstan.
function and it passes rulings and orders in its judicial ○ Eight member states ➔ India, Kazakhstan, China,
capacity. Kyrgyzstan, Pakistan, Russia, Tajikistan, and Uzbekistan.
• Though this makes it very powerful, there is an appeal ○ Four observer states ➔ Afghanistan, Belarus, Iran and
process to create accountability. Mongolia.
• There is a Securities Appellate Tribunal which is a three-
• Language ➔ The SCO’s official languages are Russian
member tribunal
and Chinese.
• A second appeal lies directly to the Supreme Court.
Q.3) With reference to Pashupatinath Temple, Consider
the following statements:
Q. 2) Which of following statement is CORRECT
1. This temple stretches on both sides of the Bagmati
regarding Shanghai Cooperation Organization:
River.
1. It was formed in 2001 and was
2. Only Hindus are allowed through the gates of the main
preceded by the Shanghai Five mechanism.
temple and was conferred the status of a World Heritage
2. It has Eight member states with India being one of its
Site in 1979.
founding member State.
Which of the above statement is/are correct
3. The Heads of State Council (HSC) is the supreme
(a) 1 only
decision-making body in the SCO.
(b) 2 only
Choose the correct code from the statements given above:
(c) Both 1 and 2
(a) 1 and 2 only
(d) Neither 1 nor 2
(b) 1 and 3 only
(c) 2 and 3 only
Answer :(c) Both 1 and 2
(d) All are correct
Explanation- Recently, a Memorandum of Understanding
(MoU) was signed under Nepal-Bharat Maitri:
Answer (b) 1 and 3 only
Development Partnership, for the construction of a
sanitation facility at the Pashupatinath Temple,
Explanation- Second statement is NOT correct because
Kathmandu.
India is Not founding member state of SCO.
• The facility will be constructed under the Nepal- Bharat
• External Affairs Minister of India S Jaishankar recently
Maitri: Development Partnership as a high impact
represented India at foreign minister meet. The meet was
community development scheme by India.
attended by foreign ministers through video conferencing.
• Financial Assistance ➔ India has pledged to extend
About SCO financial assistance amounting to Rs. 2.33 crore.
• What is it? ➔ The SCO is a permanent • Implemented by ➔ Kathmandu Metropolitan City.
intergovernmental international organisation. • Norms and Completion ➔ Norms to be laid out by the
• It is a Eurasian political, economic, and security alliance Government of Nepal and completion within 15 months.
and has been the primary security pillar of the region. • Earlier in 2018, Nepal-Bharat Maitri Pashupati
• History ➔ It was formed in 2001. It was preceded by the Dharmashala was also inaugurated in Kathmandu, Nepal.
Shanghai Five mechanism. • Significance ➔ This initiative is another milestone in
• The Heads of State Council (HSC) ➔ is the supreme strengthening cultural ties and people-to-people contacts
decision-making body in the SCO. It meets once a year between the two countries.
and adopts decisions and guidelines on all important
matters of the organisation. About Pashupatinath Temple
• Dedicated to Lord Shiva, Pashupatinath is one of the
• The organisation has two permanent bodies ➔
most important religious sites in Asia for devotees of
○ the SCO Secretariat based in Beijing.
Shiva.
○ the Executive Committee of the Regional Anti-Terrorist
• It is the largest temple complex in Nepal and stretches
Structure (RATS) based in Tashkent.
on both sides of the Bagmati River.
• Membership ➔ It was announced in 2001 by leaders of • Temples dedicated to several other Hindu and Buddhist
China, Kazakhstan, Kyrgyzstan, Russia, Tajikistan, and deities surround the temple of Pashupatinath.
Uzbekistan. Then the organisation has expanded its

61 | P a g e
• Only Hindus are allowed through the gates of the main (b) It is an Open Standard, Open Source
temple. Architecture, Secure, Scalable and Evolutionary Process
• It was conferred the status of a World Heritage Site in Automation Engine for Universities and Higher
1979 by the United Nations Educational, Scientific and Educational Institutions.
Cultural Organization (UNESCO). (c) It is a platform to give easy access to high value and
technical agricultural equipment at farmer’s doorstep.
Q.4) The Gaza Strip is a small boot-shaped (d) A single platform which provides services to all Indian
territory along the Mediterranean coast between : citizens from Central to Local Government bodies as well
(a) Israel and Jordan as Private Organizations.
(b) Lebanon and Jordan
(c) Syria and Egypt Answer (b) It is an Open Standard, Open Source
(d) Egypt and Israel Architecture, Secure, Scalable and Evolutionary Process
Automation Engine for Universities and Higher
Answer (d) Egypt and Israel Educational Institutions.
Explanation: U.S. Secretary of State Mike Pompeo met Explanation- The Global Nutrition Report 2020 has
Israeli PM Benjamin Netanyahu to discuss plans to annex released recently which identifies India as having the
parts of the West Bank. highest rates of inequalities in malnutrition.

About West Bank About SAMARTH: e-Governance platform


• The West Bank is located to the west of the Jordan River. • Recently, The Ministry of Human Resource
• It is a patch of land about one and a half times the size Development (MoHRD) has developed an e-Governance
of Goa, was captured by Jordan after the 1948 Arab- platform, SAMARTH Enterprises
Israeli War. Resource Planning (ERP), under the National Mission of
• Israel snatched it back during the Six-Day War of 1967 Education in Information and Communication
and has occupied it ever since. Technology Scheme (NMEICT).
• It is a landlocked territory, bordered by Jordan to the east • What is it? ➔ It is an Open Standard, Open Source
and Israel to the south, west and north. Architecture, Secure, Scalable and Evolutionary Process
• Following the Oslo Accords between the Israeli Automation Engine for Universities and Higher
government and the Palestine Liberation Organization Educational Institutions.
(PLO) during the 1990s, part of the West Bank came • Benefits ➔ It will help faculties, students and staff at a
under the control of the Palestinian Authority. University and Higher Educational Institutions.
• With varying levels of autonomy, the Palestinian • SAMARTH for the first time has been implemented at
Authority controls close to 40 percent of West Bank National Institute of Technology, Kurukshetra.
today, while the rest is controlled by Israel.
• Objectives ➔ The objective of this initiative is to
automate the processes of the Institute.
About Gaza strip
• This initiative will enhance productivity through better
• Location ➔ The Gaza Strip is a small boot-shaped information management in the institute by seamless
territory along the Mediterranean coast between Egypt access to information, and its utilization for various
and Israel. purposes.
• A couple of years later in 2007, Hamas, an anti- Israel
military group, took over Gaza Strip. The militia group is National Mission of Education in
often involved in violent clashes with the Israeli Defence Information and Communication Technology Scheme
Forces. (NMEICT)
• While Palestine has staked claim to both territories — • A Centrally Sponsored Scheme to leverage the potential
West Bank and Gaza Strip — Israel’s objective has been of ICT, in teaching and learning process for the benefit of
to keep expanding Jewish settlements in these regions. all the learners in Higher Education Institutions in any
time and mode.
Q. 5) Which of the following most appropriatly defines
• Launched in ➔ 2009
SAMARTH ERP:
(a) It is a platform to help the students to locate their • Significance ➔ This was expected to be a major
examination centre and see the distance between the intervention in enhancing the Gross Enrolment Ratio
current location and examination centre. (GER) in Higher Education by 5 percentage points during
the XI Five Year Plan period.

62 | P a g e
• What will it provide? ➔ It is providing connectivity, recommendation of the Rio Conference’s Agenda 21.
along with provision for access devices, to institutions, • To help publicise the Convention, 2006 was declared
learners and content generation. “International Year of Deserts and Desertification”.
• Focus areas ➔ The Convention addresses specifically
Q.6) Malathion sometimes seen in news is: the arid, semi-arid and dry sub-humid areas, known as the
(a) An asteroid that circles the sun inside Venus's orbit. drylands, where some of the most vulnerable ecosystems
(b) An endemic deep-water species of fish. and peoples can be found.
(c) A insecticide that is used to kill insects on agricultural • Aim ➔ Its 197 Parties aim, through partnerships, to
crops. implement the Convention and achieve the Sustainable
(d) A newly developed robot to take care of coronavirus Development Goals. The end goal is to protect land from
patients. over-use and drought, so it can continue to provide food,
water and energy.
Answer (c) A pesticide that is used to kill insects on • The Ministry of Environment, Forest and Climate
agricultural crops Change is the nodal Ministry for this Convention.
Explanation-
• India supplies 25 tonnes of Malathion to Iran to curb Q.8) Recently, the first Assessment of Climate Change
desert locust menace. over the Indian Region has been published by:
• Malathion is an insecticide in the chemical family (a) Niti Ayog
known as organophosphates, used to kill insects on (b) Ministry of Earth Sciences
agricultural crops. (c) Ministry of Science and Technology
• Malathion 96% ULV is used in Desert Locust control in (d) Indian Space Research Organisation
India.
Answer : b) Ministry of Earth Sciences
Q. 7) Which of following statements is/are correct about Explanation
United Nations Convention to Combat Desertification:
1. It is the only convention stemming from a direct Assessment of Climate Change over the Indian Region
recommendation of the conference of Convention on • Recently, the first Assessment of Climate Change over
Biological Diversity. the Indian Region has been published by the Ministry of
2. It is the only internationally legally binding framework Earth Sciences (MoES).
set up to address the problem of desertification. • It is India’s first-ever national forecast on the impact of
Choose the correct code given below global warming on the subcontinent in the coming
(a) 1 only century.
(b) 2 only • These projections, based on a climate forecasting model
(c) Both 1 and 2 developed at the Indian Institute of Tropical Meteorology
(d) Neither 1 nor 2 (IITM), Pune, will be part of the next report of the
Intergovernmental Panel on Climate Change (IPCC),
Answer (b) 2 only expected to be ready in 2022.
Explanation: First statement is NOT correct because It is • This is a significant step for climate science and policy
the only convention stemming from a direct in India because existing projections are put in the context
recommendation of the of historical trends in land and ocean temperatures,
Rio Conference’s Agenda 21. monsoon rainfall, floods, droughts and Himalayan
warming and glacier loss.
About United Nations Convention to Combat • The report highlights are a s follows
Desertification 1) The report indicates a rise in worldwide average
surface air temperatures by 5°C by the end of the century
• It is the sole legally binding international agreement if human activities keep emitting GHGs at the current
linking environment and development to sustainable land rate.
management or It is the only internationally legally 2) The global average temperature in the last century has
binding framework set up to address the problem of gone up by 1.1°C, according to the latest estimates by the
desertification. IPCC.
• Established ➔ 1994. 3) Another significant highlight of the assessment is the
• It is the only convention stemming from a direct projected variability in the rainfall, especially during the

63 | P a g e
monsoon season which brings 70% of the rainfall
received by India and is one of the primary drivers of its
rural agrarian economy.
4) Monsoon rainfall could change by an average of 14%
by 2100 that could go as high as 22.5%.
5) It is not mentioned if this change will be an increase or
a decrease but still represents variability.
6) Overall rainfall during the monsoon season has
decreased by 6% between 1950 and 2015.

Q. 9) Which of the following is NOT a country that


contains part of the Amazon River Basin?
(a) Columbia
(b) Peru
(c) Guyana
(d) Chile

Answer (d) Chile


Explanation :
• The Amazon basin is the part of South America drained
by the Amazon River and its tributaries.
• It is located in the countries of Bolivia, Brazil,
Colombia, Ecuador, Guyana, Peru, Suriname and
Venezuela.

24 JUNE

Q. 1) Which of the following statements is true about Atal


Pension Yojana:
1. Any citizen of India between the age group 18-40 years
can join the scheme.
2. The amount of pension will not be given for lifetime to
the spouse on death of
the subscriber.
3. It is administered by Pension Fund Regulatory and
Development Authority through National Pension System
(NPS).
Choose the correct code given below
(a) 1 and 2 only
(b) 2 and 3 only
Q. 10) Which of the following is the Northern most river (c) 1 and 3 only
in South America (d) All are correct
(a) Orinoco
(b) Parana Answer (c) 1 and 3 only
(c) Sao Francisco River Explanation : Second Statement is NOT correct because
(d) Madeira River The amount of pension is guaranteed for lifetime to the
spouse on death of the subscriber.
Answer (a) Orinoco • The flagship social security scheme of the Government
Explanation : of India - ‘Atal Pension Yojana’ (APY) - has completed

64 | P a g e
five years of implementation and garnered over 2.2 crore
subscribers. • What is it? ➔ As part of the mission, INS Kesari would
enter the Port of Male in the Republic of Maldives, to
About APY provide them 600 tons of food provisions.
• Target Segment ➔ The scheme was launched on 9th • The deployment is in consonance with the PMs vision
May, 2015, with the objective of creating a universal of Security and Growth for All in the Region ‘SAGAR’.
social security system for all Indians, especially the poor, • Significance ➔ This deployment is in line with India’s
the under-privileged and the workers in the role as the first responder in the region and builds on the
unorganised sector. excellent relations existing between these countries to
• Administered By ➔ Pension Fund Regulatory and battle the COVID-19 pandemic and its resultant
Development Authority through National Pension System difficulties.
(NPS). • The operation is being progressed in close coordination
• Eligibility ➔ Any citizen of India can join the APY with the Ministries of Defence and External Affairs, and
scheme. The age of the subscriber should be between 18- other agencies of the govt.
40 years. The contribution levels would vary and would
be low if a subscriber joins early and increases if she joins About Sagar Mission(Security and Growth for All in the
late. Region)
• Benefits ➔ It provides a minimum guaranteed pension • Background ➔ SAGAR is a term coined by PM Modi
ranging from Rs 1000 to Rs 5000 on attaining 60 years of in 2015 during his Mauritius visit with a focus on the blue
age. economy.
• The amount of pension is guaranteed for lifetime to the • Priority ➔ It is a maritime initiative which gives priority
spouse on death of the subscriber. to the Indian Ocean region for ensuring peace, stability
• In the event of death of both the subscriber and the and prosperity of India in the Indian Ocean region.
spouse, the entire pension corpus is paid to the nominee. • The goal ➔ is to seek a climate of trust and
• Tax Benefits ➔ Contributions to the Atal Pension transparency; respect for international maritime rules and
Yojana (APY) are eligible for tax benefits similar to the norms by all countries; sensitivity to each other`s
National Pension System (NPS). interests; peaceful resolution of maritime issues; and
• Analysis ➔ The scheme has been implemented increase in maritime cooperation.
comprehensively across the country covering all states • It is in line with the principles of the Indian Ocean Rim
and Union Territories with male to female subscription Association.
ratio of 57:43.
• However, only 5% of the eligible population has been About IORA
covered under APY till date. • IORA is an inter-governmental organisation to
strengthen regional cooperation and sustainable
Q. 2) What are the goals of Mission Sagar: development within the Indian Ocean Region.
1. Seek a climate of trust and transparency. • IORA has identified six priority areas ➔ namely:
2. Respect for international maritime rules and norms by (1) maritime security,
all countries. (2) trade and investment facilitation,
3. Peaceful resolution of maritime issues (3) fisheries management,
Choose the correct code from the statements given above: (4) disaster risk reduction,
(a) 1 and 2 only (5) academic and scientific cooperation and
(b) 1 and 3 only (6) tourism promotion and cultural exchanges.
(c) 2 and 3 only • Established in ➔ 1997.
(d) All are correct • Membership ➔ IORA has 22 Member States and 9
Dialogue Partners.
Answer (d) All are correct
• Secretariat ➔ Mauritius.
Explanation- As part of India’s outreach amidst the
ongoing COVID-19 pandemic, ships have departed for • Structure ➔ IORA’s apex body is the Council of
Maldives, Mauritius, Seychelles, Madagascar and Foreign Ministers (COM) which meets annually.
Comoros, to provide Food Items, COVID related
Medicines including HCQ Tablets and Medical Q.3) Consider the following statements:
Assistance Teams under Mission Sagar. 1. Toda Tribe is a pastoral tribe of the Himalayas in the

65 | P a g e
north eastern region of India. Explanation: The World Investment Report 2020 has
2. The traditional Toda dress is a distinctive shawl which been released by the UN
is called putukuli. Conference on Trade and Development (UNCTAD) .
Which of the above statement is/are correct
(a) 1 only About World Investment Report
(b) 2 only • The World Investment Report has been published
(c) Both 1 and 2 annually since 1991.
(d) Neither 1 nor 2 • It covers the latest trends in foreign direct investment
around the world.
Answer (b) 2 only • India’s Investment Scenario ➔ India was the world’s
Explanation- First Statement Is NOT correct because 9th largest recipient of foreign direct investments (FDI) in
Toda Tribe is a pastoral tribe of the Nilgiri Hills of 2019 with $51 billion in foreign investment.
southern India. • In 2018, India was the world’s 12th largest recipient
• It is in news because Many women and indigenous Toda with $42 billion foreign investment.
artisans from the Nilgiris are • Global Scenario ➔ Global FDI flows are forecast to
producing thousands of stylish, embroidered masks for decrease by up to 40% in 2020 from their 2019 value of
local residents, police, and sanitary workers. $1.54 trillion. This would bring FDI below $1 trillion for
the first time since 2005.
About Toda tribe
• Residence ➔ Toda Tribe is a pastoral tribe of the Nilgiri About UNCTAD
Hills of southern India. • UNCTAD is a permanent intergovernmental body
• Language ➔ The Toda language is Dravidian but is the established by the United Nations General Assembly in
most unusual and different among the languages 1964.
belonging to the Dravidian family. • It is part of the UN Secretariat.
• They live in settlements of from three to seven small • It reports to the UN General Assembly and the
thatched houses. Economic and Social Council, but has its own
• Work ➔ They traditionally trade dairy products, as well membership, leadership, and budget.
as cane and bamboo articles, with the other Nilgiri • It is also a part of the United Nations Development
peoples. Group.
• Toda Embroidery - In the Toda language it is called • Objectives and roles ➔ It supports developing countries
pohor. The traditional Toda dress is a distinctive shawl to access the benefits of a globalized economy more fairly
which is called putukuli. and effectively. Along with other UN departments and
•Significance ➔ Considered a grand garment, it is only agencies, it also measures the progress made in the
worn for special occasions like visits to the temple, Sustainable Development Goals, as set out in Agenda
festivals and finally as a shroud. 2030.
• Depiction ➔ The embroidery is done by Toda women • Headquartered ➔ Geneva, Switzerland.
and has distinctive red and black (and occasionally blue) •Other Reports ➔
thread work in geometric designs on unbleached white a) The Trade and Development Report
cotton fabric. b) Technology and Innovation Report
• Recognition ➔ It has got a Geographical Indications c) Digital Economy Report among others.
(GI) Tag.
Q. 5) Recently, three Olms have been put on display in
Q.4) World Investment Report is issued by : Slovenia’s Postojna Cave, where they hatched in 2016 in
(a) UN Conference on Trade and Development a rare successful breeding. With reference to this,
(UNCTAD) Consider the following statements :
(b) World Economic Forum 1. In contrast to most amphibians they are entirely aquatic
(c) World Trade Organization and eats, sleeps, and breeds underwater.
(d) Organisation for Economic Co-operation and 2. They are found in large subterranean aquatic karst
Development systems formed in limestone and dolomite rocks, and may
be found in cave entrances.
Answer (a) UN Conference on Trade and Development Which of the above statement is/are correct
(UNCTAD) (a) 1 only

66 | P a g e
(b) 2 only berths, terminals and associated facilities will be
(c) Both 1 and 2 constructed by private firms in the public-private-
(d) Neither 1 nor 2 partnership (PPP) mode.

Answer (c) Both 1 and 2 Q. 7) Which of following statements is/are correct about
Explanation- Dexamethasone:
1. It is a generic steroid widely used in other diseases to
About Olms reduce inflammation.
• The Olms are eyeless (skin covered eyes) pink 2. It is banned in-competition by the World Anti- Doping
salamanders also known as the "baby dragon" and Agency.
"human fish" for its pale skin-like colour. Choose the correct code given below
• They feed on detritus and endemic cave vertebrates and (a) 1 only
hide in crevices or bottom sediment when disturbed. They (b) 2 only
can survive without food for up to 10 years. (c) Both 1 and 2
• A long-lived species that only matures to a sexual stage (d) Neither 1 nor 2
after the age of 12-15 years and reproduces very slowly.
• It can live a century and breeds only once a decade Answer (c) Both 1 and 2
usually in laboratories throughout Europe or deep in caves Explanation:
away from people. • It is the world’s first coronavirus treatment proven to
• They have a powerful sensory system of smell, taste, reduce the risk of death among severely ill patients.
hearing and electro sensitivity. • According to scientists, the drug has been proven to
• Habitat ➔ They are found in large subterranean aquatic reduce the risk of death significantly in COVID-19
karst systems formed in limestone and dolomite rocks, patients on ventilation by as much as 35% and patients on
and may be found in cave entrances. oxygen by 20%.
• They are an entirely aquatic species and prefer waters
ranging from 5- 15°C. About Dexamethasone
• Found in Bosnia and Herzegovina, Croatia, Italy, • Dexamethasone is a generic steroid widely
Slovenia and France. used in other diseases to reduce inflammation.
• Threats ➔ The olm requires clean water, and is therefore • The drug helps stop some of the damage that can happen
vulnerable to water pollution. when the body’s immune system goes into overdrive as it
tries to fight off coronavirus.
Q.6) Vadhavan Port recently seen in news is proposed in: • It is used to treat a range of diseases, including
(a) Gujarat rheumatism, asthma, allergies, and even to help cancer
(b) Andhra Pradesh patients better handle nausea triggered by chemotherapy.
(c) West bengal • The drug -Dexamethasone is banned in-competition by
(d) Maharashtra the World Anti-
Doping Agency (WADA) but its usage is allowed out-of-
Answer (d) Maharashtra competition if athletes have the requisite Therapeutic
Explanation- usage Exemption (TUE).
• Recently, the Ministry of Environment, Forest and
Climate Change has said that there is no bar on building a About Steroids
port in the ecologically fragile zone. • Steroids are a man-made version of chemicals, known
• The Ministry said that the ports as an industry are as hormones that are made naturally in the human body.
allowed in the ecologically sensitive Dahanu taluka area, • Steroids are designed to act like these hormones to
which includes Vadhavan in Maharashtra’s Palghar reduce inflammation known as corticosteroids.
district, where the Centre plans to build an ambitious new • There are also anabolic steroids used by bodybuilders
major Vadhavan port estimated to cost Rs 65,544.54 and athletes.
crore.
• Vadhavan will be set up as a corporate port under the Q.8) Yanomami , recently seen in news is an indigenous
Companies Act. tribal group native to which of the following regions?
• It would be developed as a landlord port, with the port
company building the basic port infrastructure, while a) Amazon Rainforest
b) Miombo woodlands

67 | P a g e
c) Sumatra Rainforest • During the summers in the region around Buenos Aires,
d) Andaman & Nicobar Islands the pampero storms are a welcome feature marking the
end of long periods of high humidity and extreme heat.
Answer : a. Amazon Rainforest
Explanation
• Recently, a Yanomami indigenous boy died in Brazil
after contracting Covid-19, raising fears for the Amazon
tribes.
• Brazil is home to an estimated 8,00,000 indigenous
people from more than 300 ethnic groups.
• Guarani, Kaingang, Pataxo Ha Ha Hae Tupinamba,
Yanomami, Tikuna and Akuntsu are popular tribe of
Amazon.
• Yanomami, also called South American Indians, live in
the remote forest of the Orinoco River basin in southern
Venezuela and the northernmost reaches of the Amazon
River basin in northern Brazil.
• They numbered around 27,000 individuals throughout
their range.
• Yanomami live in small, scattered, semi-permanent
villages and speak the Xiriana language.
• They practice hunting and slash-and-burn agriculture.
Q. 10) The Atacama Desert is the driest non-polar desert
in the world. The desert owes its extreme aridity to a
Q. 9) Which of following statements is/are true about
constant temperature inversion due to:
1. It is a burst of cold polar air from the west, southwest
(a) Benguela current
or south in the south of Brazil, Argentina, Uruguay,
(b) Agulhas current
Paraguay and Bolivia.
(c) Humboldt current
2. The pampero storms mark the end of long periods of
(d) Brazil current
high humidity and extreme heat during the summers in the
region around Buenos Aires.
Answer (c)Humboldt current
Choose the correct code given below
Explanation :
(a) 1 only
• The desert owes its extreme aridity to a constant
(b) 2 only
temperature inversion due to the cool north-flowing
(c) Both 1 and 2
Humboldt ocean current and to the presence of the strong
(d) Neither 1 nor 2
Pacific anticyclone.
• The Atacama Desert is a desert plateau in South America
Answer (c) Both 1 and 2
on the Pacific coast, west of the Andes Mountains.
Explanation :
• The Atacama Desert is the driest nonpolar desert in the
• The pampero is a burst of cold polar air from the west,
world, as well as the only true desert to receive less
southwest or south on the pampas in the south of Brazil,
precipitation than the polar deserts.
Argentina, Uruguay, Paraguay and Bolivia.
• Most of the desert is composed of stony terrain, salt
• The Pampero is most common at winter in the southern
lakes (salares), sand, and felsic lava that flows towards the
hemisphere (principally between May and August).
Andes.

68 | P a g e
Centres and Krishi Vigyan Kendras.
3. This campaign will work in mission mode for 125 days
with an outlay of Rs. 50,00000 crore.
Choose the correct code given below
(a) 1 and 2 only
(b) 2 and 3 only
(c) 1 and 3 only
(d) All are correct

Answer (a) 1 and 2 only


Explanation : Third Statement is NOT correct because
This campaign will work in mission mode for 125 days
with an outlay of Rs. 50,000 crore. (50 thousand crore and
not 50 lakh crore)

Garib Kalyan Rojgar Abhiyaan


• Aim ➔ To empower and provide livelihood
opportunities to the returnee migrant workers and rural
citizens.
• Coverage ➔ It will be implemented across 116 districts
in the six States namely Bihar, Uttar Pradesh, Madhya
Pradesh, Rajasthan, Jharkhand and Odisha.
• Duration and Outlay ➔ This campaign will work in
mission mode for 125 days with an outlay of Rs. 50,000
crore.
◎ Features ➔ It will involve 25 different types of works
to provide employment to the migrant workers on one
hand and create infrastructure in the rural regions of the
country on the other hand.
• Implementation ➔ It will be operationalised through the
Common Service Centres and Krishi Vigyan Kendras.
• Ministerial Involved ➔ It will be a coordinated effort
between 12 different Ministries/Departments, namely
Rural Development, Panchayati Raj, Road Transport &
Highways, Mines, Drinking Water & Sanitation,
Environment, Railways, Petroleum & Natural Gas, New
& Renewable Energy, Border Roads, Telecom and
Agriculture.
• Background ➔ Common Services Centres are a
strategic cornerstone of the National e-Governance Plan
(NeGP).
• It was approved in May 2006 to introduce e-governance
on a massive scale under the National Common Minimum
25 JUNE Programme.
• Aim ➔ It aims to provide high quality and cost-effective
Q. 1) Which of the following statements is true about video, voice and data content and services, in the areas of
Garib Kalyan Rojgar Abhiyaan: e-governance, education, health, telemedicine,
1. It aims to empower and provide livelihood entertainment as well as other private services.
opportunities to the returnee migrant workers and rural • Features ➔ A highlight of the CSCs is that it offers web-
citizens. enabled e- governance services in rural areas, including
2. It will be operationalised through the Common Service application forms, certificates and utility payments such

69 | P a g e
as electricity, telephone and water bills. • Habitat ➔ It is a freshwater crocodile native to southern
Iran to the Indian subcontinent.It is extinct in Bhutan and
Q. 2) Which of the following is INCORRECT in context Myanmar.
of Krishi Vigyan Kendra(KVK): • IUCN Status ➔ Vulnerable
(a) The first KVK was established in 1974 at Puducherry.
• CITES ➔ Appendix I
(b) It is an integral part of the National Agricultural
Research System. • Threats ➔ a) Habitat destruction b) Entanglement and
(c) The KVK scheme is financed in the ratio of 60:40 drowning in fishing
between center and state government. equipment and c) Increasing incidents of conflict with
(d) They produce products like seed, humans.
planting material, bio-agents, and livestock.
Estuarine or Saltwater Crocodile:
Answer (c) The KVK scheme is financed in the ratio of • It is considered as the Earth’s largest living crocodile
60:40 between center and state government. species native to saltwater habitats and brackish wetlands.
Explanation- Third Statement is NOT correct because • Habitat ➔ It is found throughout the east coast of India,
The KVK scheme is 100% financed by the Government Southeast Asia and northern Australia.
of India. • IUCN Status ➔ Least Concern
• Threats: a) Illegal hunting b) habitat loss and c)
About Krishi Vigyan Kendra antipathy toward the species because of its reputation as a
• It is an integral part of the National Agricultural man-eater.
Research System (NARS).
• Composition ➔ It consists of scientists, technical staff, Gharial
administration support staff and auxiliary staff. • Distribution ➔ It was once found across Bangladesh,
• Established ➔ The first KVK was established in 1974 Bhutan, India, Myanmar, Nepal and Pakistan. However,
at Puducherry. currently it survives in several severely fragmented
• Functions ➔ KVKs also produce quality technological populations in India and Nepal.
products like seed, • Protected areas ➔ National Chambal Sanctuary and
planting material, bio-agents, and livestock and make it Katarniaghat Wildlife
available to farmers. Sanctuary.
• Funding ➔ The KVK scheme is 100% financed by the • IUCN Status ➔ Critically Endangered
Government of India. • Wild Life (Protection) Act,1972 ➔ Schedule I
• CITES ➔ Appendix I
Q.3) Consider the following statements regarding Marsh • Threats ➔ Construction of Dam, barrages, and water
Crocodile: abstraction, entanglement in fishing nets, River bed
1. It is a saltwater crocodile native to cultivation and sand mining.
southern Iran to the Indian subcontinent.
2. It is listed as endangered in the IUCN list of threatened Q.4) Recent excavations at Kodumanal village have
species. thrown light on burial rituals and the concept of afterlife
Which of the above statement is/are correct in megalithic culture. With reference to this, Kodumanal
(a) 1 only village is located on the northern banks of:
(b) 2 only (a) Amaravati River
(c) Both 1 and 2 (b) Shimsha River
(d) Neither 1 nor 2 (c) Moyar River
(d) Noyyal River
Answer (b) 2 only
Explanation- First Statement is NOT correct because It is Answer (d) Noyyal River
a freshwater crocodile native to southern Iran to the Indian Explanation:
subcontinent.
• Second statement is NOT correct because It is listed as About Kodumanal Exacavations
Vulnerable in the IUCN list of threatened species. • The researchers have identified 250 cairn-circles at the
village in Erode district.
About Mugger or Marsh Crocodile: • Earlier excavations revealed that the site served as a

70 | P a g e
trade-cum-industrial centre from 5th century BCE to 1st Explanation- Second statement is NOT correct because
century BCE. The civil services board is headed by chief secretary of a
• The rectangular chambered cists, each two metres long state.
and six metres wide, are made of stone slabs, and the
entire grave is surrounded by boulders that form a circle. About Civil services Board
• The grave could be of a village head or the head of the • Background ➔ To insulate the bureaucracy from
community as the size of two boulders, each facing east political interference and to put an end to frequent
and west, are bigger than other boulders. transfers of civil servants by political bosses, the Supreme
• Believing that the deceased person will get a new life Court had in 2013 directed the Centre and the states to set
after death, pots and bowls filled with grains were placed up a civil services board to consider transfers and postings
outside the chambers. of bureaucrats among others.
• As per rules, all states should have a civil services board
Kodumanal village to decide on transfers and postings of the bureaucrats.
• It is a village located in the Erode district in the southern • Functions ➔
Indian state of Tamil Nadu. ○ The board is mandated to decide on the transfer of a
• The place is an important archaeological site. civil servant before
• It is located on the northern banks of Noyyal River, a completion of his or her fixed tenure.
tributary of the Cauvery. ○ The rules mandate the civil services board to submit an
annual report on January 1 to the central government
Megaliths Culture about the date of the meetings held by them.
• Megaliths refer to large stone structures that were
Composition ➔
constructed either as burial sites or as commemorative
• The civil services board is headed by chief secretary of
sites.
a state.
• The burial sites are the sites with actual burial remains,
• It has senior most additional chief secretary or chairman,
such as dolmenoid cists (box-shaped stone burial
Board of Revenue, Financial Commissioner or an officer
chambers), cairn circles (stone circles with defined
of equivalent rank and status as member.
peripheries), and capstones (distinctive mushroom-
• In addition, it will have Principal Secretary or Secretary,
shaped burial chambers found mainly in Kerala).
Department of Personnel in the state government as
• The megalithic culture lasted from the Neolithic Stone
member secretary.
Age to the early Historical Period (2500 BCE to CE 200)
across the world.
How do they see the board to be lending officer’s
• In India, the majority of the megaliths are of the Iron
supremacy over them?
Age (1500 BC to 500 BC).
• If any officer is to be transferred before completing his
• Megaliths are spread across peninsular India,
minimum tenure, the board will record the reasons for the
concentrated in the states of Maharashtra (mainly in
transfer.
Vidarbha), Karnataka, Tamil Nadu, Kerala, Andhra
• It will seek views from the concerned officer and then
Pradesh and Telangana.
give a judgement on whether the tenure of the officer is to
• The chambers containing the mortal remains were
be ended mid-way.
usually made of terracotta.
• The final authority will be the CM.
Q. 5) With reference to Civil Services Board, Consider
Q.6) The Khadi and Village Industries Commission
the following statements :
(KVIC) has rolled out a
1. The board can decide to transfer a civil servant before
unique project to produce Neera. Neera is extracted from
completion of his or her fixed tenure.
which of the following :
2. The civil services board is headed by chief secretary of
(a) Bamboo plants
India.
(b) Palm Trees
Which of the above statement is/are correct
(c) Sugarcane
(a) 1 only
(d) Jatropha
(b) 2 only
(c) Both 1 and 2
Answer (b) Palm Trees
(d) Neither 1 nor 2
Explanation-
• The Khadi and Village Industries Commission (KVIC)
Answer (a) 1 only

71 | P a g e
has rolled out a unique project to produce Neera and Palm in Bihar, the birthplace of Sita, will now have a different
jaggery which has huge potential to create employment. look with traditional Madhubani Paintings adorning their
Aim ➔ The project that aims at promoting walls.
Neera as a substitute to soft drinks while also
creating self-employment to Adivasis and About Madhubani Painting
traditional trappers was launched at Dahanu in • Madhubani art (or Mithila painting) is a style of Indian
Palghar district of Maharashtra. painting, practiced in the Mithila region of the Indian
subcontinent.
About Neera • It is characterized by line drawings filled in by bright
• Neera, extracted from palm trees before sunrise, is a colours and contrasts or patterns.
nutrient-rich health drink consumed in many states. • The themes of these paintings are figures from nature
However, due to lack of institutionalized market and mythological events.
technique, the commercial production and large scale • The themes widely painted are of Hindu deities such as
marketing of Neera has not commenced yet. Krishna, Rama, Siva, Durga, Lakshmi, Saraswati, Sun
• There are about 10 crore palm trees across the country and Moon, Tulasi plant, court scenes, wedding scenes,
and a wide range of products like candies, milk social happenings.
chocolates, palm cola, ice-cream and traditional sweets • It has been accorded with the GI (Geographical
can be produced from neera if properly marketed. Indication) status.
• India has an abundance of palm fields in states like • Used to paint the walls of room, known as KOHBAR
Maharashtra, Gujarat, Goa, Daman & Diu, Dadra and GHAR in which the newly wedded couple meets for the
Nagar Haveli, Tamil Nadu, Uttar Pradesh and Bihar that first time.
can make India a leading producer of Neera globally. • This painting is done with a variety of tools, including
fingers, twigs, brushes, nib-pens, and matchsticks and
Khadi and Village Industries Commission (KVIC) using natural dyes and pigments.
• It is a statutory body established under Khadi and • It is characterised by its eye-catching geometrical
Village Industries Commission Act,1956. patterns.
• Ministry ➔ It is an apex organisation under the Ministry • Has five distinctive styles – Bharni, Katchni, Tantrik,
of Micro, Small and Medium Enterprises. Godna and Gobar.
• Bharni, Kachni and Tantrik style were mainly done
• Functions ➔ seeks to – “plan, promote, facilitate,
Brahman & Kayashth women, who are upper caste
organise and assist in the establishment and development
of khadi and village industries in the rural areas in
coordination with other agencies engaged in rural
development wherever necessary.”

Q. 7) Identify the painting from the description given


below :
1. It is characterized by line drawings filled in by bright
colours and contrasts or patterns.
2. The themes of these paintings are figures from nature
and mythological events.
3. It has been accorded with the Geographical Indication
status.
4. It is used to paint the walls of room, known as
KOHBAR GHAR.
Choose the correct answer given below
(a) Malwa Painting
(b) Kalighat Painting
(c) Madhubani Painting
(d) Phad Painting

Answer (c) Madhubani Painting


Explanation: All government buildings at Punauradhama

72 | P a g e
women in India and Nepal. of the mineral resources of the country.
• Godna & Gobar style is done by the Dalit & Dushadh • The portal has been designed, developed and
communities. implemented by the National Informatics Centre (NIC).
• The portal is also integrated with NGO Darpan Portal of
NITI Aayog.
NGO-DARPAN

• NGO-DARPAN is a platform that provides space for


interface between Non-Government organizations
(NGOs)/Voluntary Organizations (VOs) and key
Government Ministries / Departments / Government
Bodies.
• It is an e-governance application offered by NITI Aayog
to electronically maintain data and transparency regarding
NGOs/VOs in the country.
• The NGO-DARPAN was earlier maintained by
erstwhile Planning Commission, which has been replaced
by the NITI Aayog in 2015.

Q. 9) What is the name of grassland found in Argentina


and Uruguay ?
(a) Prairie
(b) Llanos
(c) Pampas
Q.8) SATYABHAMA Portal aims to: (d) Veld

a) Promote Women Farmers And Entrepreneurs. Answer (c) Pampas


b) Combat the menace of child labour, trafficking and
sexual abuse.
c) Promote research and development in the mining and Explanation :
minerals sector. • In parts of South America, the plains are known as
d) To help MSMEs directly register their cases relating to pampas
delayed payments by Central
Ministries/Departments/CPSEs/State Governments.

Answer : c) Promote research and development in the


mining and minerals sector.
Explanation
SATYABHAMA Portal

• Union Ministry of Mines has launched a portal


"SATYABHAMA (Science and Technology Yojana for
Atmanirbhar Bharat in Mining Advancement)".
• Its aim is to promote research and development in the
mining and minerals sector.
• It has been launched to increase efficiency and
effectiveness of the Science and Technology Programme
Scheme.
• Under the Science and Technology Programme Scheme,
the Ministry of Mines promotes research in applied
geosciences, mineral exploration, mining and allied areas,
mineral processing, optimum utilization and conservation

73 | P a g e
Answer (c) Brazil

Explanation :
• The Mato Grosso Plateau (Planalto do Mato Grosso) is
a plateau in central Brazil occupying most of the state of
Mato Grosso.
• It contains mostly savanna and woodland.
• It is an ancient erosional plateau
• In the south it gives way to floodplains called the
Pantanal.
• The plateau is home to indigenous peoples, such as the
Xavante.

• These are areas of grasslands and rich soil. Pampas are


found in northern Argentina and Uruguay.
• The main products of the pampas are cattle and wheat
grain.

Q.10) Mato Grosso Plateau is an ancient erosional plateau


and contains mostly savanna and woodland. It is a pleatue
in :
(a) Bolivia
(b) Argentina
(c) Brazil 26 JUNE
(d) Chile
Q. 1) Which of the following statements is true about
Financial Action Task Force:
1. It is an intergovernmental organization founded in 1989
during the BRICS Summit
in Paris.
2. It is located at the Organisation for Economic
Cooperation and Development (OECD) headquarters.
3. It works for combating money laundering and terrorist
financing.
Choose the correct code given below
(a) 1 and 2 only
(b) 2 and 3 only
(c) 1 and 3 only
(d) All are correct

Answer (b) 2 and 3 only


Explanation : First Statement is NOT correct because It is
an inter-governmental body established in 1989 during

74 | P a g e
the G7 Summit where they live
in Paris. year-round.
• Indian officials have attended the virtual 32nd plenary
meeting of Eurasian Group on Combating Money ABOUT GOLDEN LANGUR
Laundering and Financing of Terrorism (EAG). •Scientific Name ➔ Trachypithecus geei.
• Its diet is herbivorous, consisting of ripe and unripe
About EAG fruits, mature and young leaves, seeds, buds and flowers.
• What is it? ➔ It is a regional body established in 2004. • Habitat ➔ Golden langurs occupy moist evergreen and
• Currently, it is an associate member of the Financial tropical deciduous forests as well as some riverine areas
Action Task Force(FATF). and savannas in Assam and Bhutan.
• Members ➔ It comprises nine countries namely India, • Colour ➔ Their hair ranges from dark golden to creamy
Russia, China, Kazakhstan, Kyrgyzstan, Tajikistan, buff and their faces are black and hairless except for a
Turkmenistan, Uzbekistan and Belarus. long pale beard. It has been noted that their fur changes
• Purpose ➔ To ensure cooperation and integration of colors according to the seasons.
EAG member-states • Conservation Status ➔
into the international system of anti-money laundering ○ Endangered by the IUCN Red List
and combating financing of terrorism in accordance with ○ Listed as Appendix I on the CITES website.
the recommendations of FATF.
• Threats ➔ The main reason for low numbers of golden
langurs is because of their localized habitat and the rapid
About Financial Action Task Force:
loss of this habitat due to
• It is an inter-governmental body established in 1989
deforestation.
during the G7 Summit in Paris.
• Secretariat ➔ It is located at the Organisation for Q.3) Consider the following statements regarding
Economic Cooperation and Development (OECD) National Institute of Public Finance and Policy:
headquarters in Paris. 1. It is an autonomous body set up jointly by the RBI and
Objectives ➔ Planning commission .
• set standards and promote effective implementation of 2. It is registered under the Societies
legal, regulatory and operational measures for combating Registration Act, 1860 and undertakes research, policy
money laundering and terrorist financing and other advocacy in areas related to public economics.
related threats to the integrity of the international financial Which of the above statement is/are correct
system. (a) 1 only
• Members ➔ It comprises 37 member jurisdictions and (b) 2 only
2 regional organisations representing most major (c) Both 1 and 2
financial centres in all parts of the globe. (d) Neither 1 nor 2
• India is one of the members.
◎ The FATF Plenary is the decision making body of the Answer (b) 2 only
FATF. It meets three times per year.

Q. 2) Which of the following is INCORRECT in about Explanation- First Statement is NOT correct because
Golden Langur NIPFP is an autonomous body set up jointly by the
(a) Its diet is herbivorous, consisting of ripe and unripe Ministry of Finance, the erstwhile Planning Commission,
fruits. and several state governments.
(b) It is currently listed as endangered in red list of IUCN. • Urjit Patel has been appointed chairman of the National
(c) It is an Old World monkey found in a small region of Institute of Public Finance and Policy (NIPFP). He will
western ghats. succeed Vijay Laxman
(d) Chakrashila is India’s first wildlife sanctuary with Kelkar.
golden langur as the primary species.
About NIPFP
Answer (c)It is an Old World monkey found in a small • What is it? ➔ NIPFP is an autonomous body set up
region of western ghats. jointly by the Ministry of Finance, the erstwhile Planning
Explanation- It is an Old World monkey found in a small Commission, and several state governments.
region of western Assam, India and neighboring Bhutan

75 | P a g e
• It is registered under the Societies Registration Act, Recovery Facility of the AIIB and co-financed by the
1860. Asian Development Bank(ADB).
• Formation ➔ It was founded in 1976. • Implementation ➔ It will be executed by the
Functions ➔ Department of Economic Affairs, Ministry of Finance.
○ It undertakes research, policy advocacy and capacity
building in areas related to public economics. Q. 5) With reference to Horseshoe Crab, Consider the
○ One of the major mandates of the institute is to assist following statements :
the Central, State and Local governments in formulating 1. Catching and killing of a horseshoe crab is an offence
and reforming public policies by providing an analytical in India.
base. 2. Horseshoe crabs are marine and brackish
◎ Funding ➔ It receives an annual grant from the water arthropods.
Ministry of Finance and various State governments. Which of the above statement is/are correct
However, it maintains an independent non- government (a) 1 only
character. (b) 2 only
(c) Both 1 and 2
◎ Location ➔ New Delhi.
(d) Neither 1 nor 2
• Governing Body ➔
○ It includes the Revenue Secretary, Economic Affairs Answer (c) Both 1 and 2
Secretary and the Chief Economic Advisor from the Explanation- Horseshoe crabs face an uncertain future in
Union Finance Ministry and representatives from NITI Odisha, their largest
Aayog, RBI and three state governments. habitat in India, even as the world gets ready to celebrate
○ It also includes three distinguished economists, the first- ever ‘International Horseshoe Crab Day’ on June
members of sponsoring agencies and other invitees. 20, 2020.
○ It is involved in appointing the Chairman and the
Director. About Horseshoe Crab
○ The usual tenure of a chairman is four years, which can • Horseshoe crabs are marine and brackish water
be extended. arthropods. They are not true crabs, which are
crustaceans.
Q.4) Government of India has signed a $750 million • The crabs are represented by four extant species in the
COVID-19 Active Response and Expenditure Support world. Out of the four, two species are distributed along
Programme with which of the following? the northeast coast of India.
(a) Bank for International Settlements • Only T gigas species of the horseshoe crab is found
(b) Asian Infrastructure Investment Bank along Balasore coast of Odisha.
(c) International Bank for Reconstruction and
• Conservation ➔ The crab was included on September 9,
Development
2009, in the Schedule IV of the Wild (Life) Protection
(d) African Development Bank
Act, 1972, under which, the catching and killing of a
horseshoe crab is an offence.
Answer (b) Asian Infrastructure Investment Bank
Explanation: Government of India and Asian ◎ Their significance ➔
Infrastructure Investment Bank(AIIB) has signed a $750 ○ The horseshoe crab is one of the oldest marine living
million “COVID-19 Active Response and Expenditure fossils whose origin date back to 445 million years before
Support Programme”. the dinosaurs existed.
○ One of their ecological functions is to lay millions of
About Program eggs on beaches to feed shorebirds, fish and other
wildlife.
• Aim ➔ To assist India to strengthen its response to the
adverse impacts of the COVID-19 pandemic on poor and ◎ Threats ➔ Poachers kill them for their meat that is
vulnerable households. popularly believed to have aphrodisiac qualities.
• Programme Beneficiaries ➔ Families below poverty
About AIIB
line, farmers, healthcare workers, women, widows,
• The Asian Infrastructure Investment Bank (AIIB) is a
people with disabilities, senior citizens, construction
multilateral development bank with a mission to improve
workers and vulnerable groups.
social and economic outcomes in Asia.
• Funding ➔ It is financed under the COVID-19 Crisis
• HQ ➔ Beijing, China.

76 | P a g e
• It is established by a multrilateral treaty - AIIB Articles 1)Constraints on Government Powers,
of Agreement. 2)Absence of Corruption,
• India joined AIIB in 2016 as a regional member of the 3)Open Government,
Bank. 4)Fundamental Rights,
• Members ➔ Further, 14 of the G-20 nations are AIIB 5)Order and Security,
members including France, Germany, Italy and the 6)Regulatory Enforcement,
United Kingdom. 7)Civil Justice,
• China is the largest shareholder in AIIB with a 26.06% 8)Criminal Justice.
voting power, followed by India with a 7.62% voting
power. Q. 7) Which of the following is true about Kumbhar
Significance of AIIB ➔ Sashaktikaran Yojana :
◎ The United Nations has addressed the launch of AIIB 1. An initiative of the Ministry of Rural Development for
as having potential for “scaling up financing for empowerment of potter community in the remotest of
sustainable development” for the concern of global locations in the country.
economic governance. 2. It provides training for advanced pottery products and
• The capital of the bank is $100 billion, Market linkages and visibility through KVIC exhibitions.
equivalent to 2⁄3 of the capital of the Asian Development Choose the correct code given below
Bank and about half that of the World Bank. (a) 1 only
(b) 2 only
Q.6) Rule of Law Index is released by: (c) Both 1 and 2
(a) International Law Commission (d) Neither 1 nor 2
(b) International Labour Organization
(c) World Justice Project Answer (b) 2 only
(d) Transparency International Explanation: First statement is NOT correct because It is
an initiative of the Khadi and Village Industries
Answer (c) World Justice Project Commission (KVIC) for empowerment of potter
Explanation- community in the remotest of locations in the country.
• The program reaches out to the potters in - U.P., M.P.,
Rule of Law Index Maharashtra, J&K, Haryana, Rajasthan, West Bengal,
• It is released by the World Justice Project, an Assam, Gujarat, Tamil Nadu, Odisha, Telangana and
independent organisation. Bihar.
• It is a quantitative assessment tool designed to offer a • It provides the following support -
detailed and comprehensive picture of the extent to which 1)Training for advanced pottery products.
countries adhere to the rule of law in practice. 2) Latest, new technology pottery equipment like the
• The World Justice Project defines the rule of law system electric Chaak.
as one in which the following four universal principles are 3)Market linkages and visibility through KVIC
upheld: exhibitions.
1)The government and its officials and agents are
accountable under the law. About KVIC
2)The laws are clear, publicized, stable and fair, and • It is a statutory body established by an Act of Parliament
protect fundamental rights, including the security of (Khadi and Village Industries Commission Act of 1956).
persons and property. • In April 1957, it took over the work of former All India
3)The process by which the laws are enacted, Khadi and Village Industries Board.
administered, and enforced is accessible, efficient, and • It is an apex organization under the Ministry of MSME,
fair. with regard to khadi and village industries within India.
4)Justice is delivered by competent, ethical, and • It seeks to plan, promote, facilitate, organise and assist
independent representatives and neutrals who are of in the establishment and development of khadi and village
sufficient number, have adequate resources and reflect the industries in the rural areas in coordination with other
makeup of the communities they serve. agencies engaged in rural development wherever
• India has secured 69th rank of 128 countries. necessary.
• It measures countries’ rule of law performance across
eight factors: Q.8) Gharial , a fresh-water crocodile, often seen in news
is globally found only in :

77 | P a g e
a) India and Bangladesh
b) India and Pakistan
c) India and Nepal
d) India and Bhutan

Answer : c) India and Nepal


Explanation : Recently Government of Uttar Pradesh has
released Gharials (Gavialis gangeticus) in the Ghaghara
river for the conservation and protection in natural habitat.
About Gharial
• Scientific name ➔ Gavialis Gangeticu.
• Habitat ➔ They are a fresh-water crocodile.
• They live in deep fast-flowing rivers.
Distribution ➔ Globally Gharial is found only in India
and Nepal.
Q.10) Which of the following city is not part of New
• India: Girwa River, Chambal River, Ken River, Son England region:
River, Mahanadi River, Ramganga River
(a) Boston
• Nepal: Rapti-Narayani River
(b) Worcester
Status ➔ (c) San Francisco
○ Critically Endangered— IUCN Red List (d) New Hampshire
○ Listed under Schedule I of the Wild Life (Protection)
Act, 1972. Answer (c) San Francisco
○ Listed on Appendix I of Convention on International • Washington DC is a District of Colombia.
Trade in
Endangered Species of Wild Fauna and Flora (CITES).
◎ Conservation Efforts ➔ Breeding Centres of Kukrail
Gharial Rehabilitation Centre in Lucknow, Uttar Pradesh,
National Chambal Sanctuary (Gharial Eco Park, Madhya
Pradesh).
• Threats ➔ Hunting for skins, trophies and indigenous
medicine, and their eggs collected for consumption,
Decrease of riverine habitat as dams, barrages, irrigation
canals and artificial embankments were built; siltation
and sand-mining changed river courses.

Q.9) Which of the following is called as the coffee port of


the world:
(a) Santos
(b) Sao Polo
(c) Monte Video
(d) Buenos Aires 27 JUNE

Answer (a) Santos Q. 1) Which of the following statements is true about


Damodar River:
1. It is a river flowing only across the Indian states of
Jharkhand and West Bengal.
2. It rises in the Palamu hills of the Chota Nagpur plateau.
Choose the correct code given below
(a) 1 only
(b) 2 only
(c) Both 1 and 2
(d) Neither 1 nor 2

78 | P a g e
Answer (c) Both 1 and 2
Explanation : Government of India, West Bengal
Government, the Asian Infrastructure Investment Bank
(AIIB) and World Bank have signed a loan agreement for
a project titled ‘West Bengal Major Irrigation and Flood
Management Project’.
• Aim ➔ To improve irrigation service delivery,
strengthen flood risk management and improve climate
change resilience in the Damodar Valley Command
Area(DVCA).
• Significance ➔ The project is intended to benefit about
2.7 million farmers from five districts of West Bengal.

About Damodar river


• It is a river flowing across the Indian states of Jharkhand
and West Bengal. Explanation- The Baltic countries of Estonia, Latvia and
• Origin ➔ It rises in the Palamu hills of the Chota Lithuania have opened their borders to one another,
Nagpur plateau in the state of Jharkhand. creating a coronavirus “travel bubble” with an improvised
• Main Tributaries ➔ Barakar, Konar, Bokaro, Haharo, idea to boost travel into their countries.
Jamunia, Ghari, Guaia, Khadia and Bhera. • Aim & objective ➔ The ‘Baltic travel bubble’ aims to
• The river was earlier known as the River of Sorrows as facilitate the citizens of these three countries to travel
it used to flood many areas of Districts of West Bengal. within the region without hassles.
• However, those who are coming from any other than
Q. 2) The Baltic countries have opened their borders to these three countries would be required to follow self-
one another,creating a coronavirus “travel bubble” with isolation guidelines and stay in quarantine for exactly 14
an improvised idea to boost travel into their days.
countries. With reference to this statement, Which of the About Baltic Countries
following is a Baltic country?
(a) Norway • Location ➔ Baltic countries are located in the
(b) Armenia northeastern region of Europe, and comprises Estonia,
(c) Slovenia Latvia, and Lithuania, on the eastern shores of the Baltic
(d) Lithuania Sea.
• The Baltic states are bounded on the west and north by
Answer (d) Lithuania the Baltic Sea, which gives the region its name, on the east
by Russia, on the southeast by Belarus, and on the
southwest by Poland.
• Membership ➔ The region has been part of the
European Union since 2004 and since 2007 has been a
member of the European Schengen Free Travel Area.
• Resources ➔ The Baltic region is not rich in natural
resources but is leaders in cutting technology
developments and start-up hubs.
• India and Baltic countries ➔ share a strong bond based
on historical connect and common linguistic roots.

Q.3) Consider the following statements regarding


Troglomyces twitteri:
1. It is a type of Bacteria that attack insects and millipedes.
2. They live on the outside of host organisms and belongs

79 | P a g e
to an order called Laboulbeniales.
Which of the above statement is/are correct Answer (c) Both 1 and 2
(a) 1 only Explanation- Recently, Assam CM ordered the arrest of a
(b) 2 only political commentator
(c) Both 1 and 2 who had described Chaolung Sukapha as a “Chinese
(d) Neither 1 nor 2 invader”.

Answer (b) 2 only About Chaolung Sukapha


Explanation- First Statement is NOT correct because • Who was Chaolung Sukapha? ➔ Sukapha was a 13th-
Troglomyces twitteri is a type of parasitic fungus. century ruler who founded the Ahom kingdom that ruled
• It belongs to an order called Laboulbeniales — tiny Assam for six centuries. Contemporary scholars trace his
fungal parasites that attack insects and millipedes. roots to Burma.
• Where does it live? ➔ These fungi live on the outside of • He reached Brahmaputra valley in Assam from upper
host organisms; in this case, on the reproductive organs of Burma in the 13th century with around 9,000 followers.
millipedes. • In 1235, Sukapha and his people settled in Charaideo in
• History ➔ Laboulbeniales were first discovered in the upper Assam after wandering about for years, defeating
middle of the 19th century. those who protested his advance and temporarily staying
• Foundational work on the Laboulbeniales was at different locations.
completed by the American mycologist Roland Thaxter • It was in Charaideo (in Assam) that Sukapha established
(1858–1932). his first small principality, sowing the seeds of further
expansion of the Ahom kingdom.
Q.4) Vasudhaiva Kutumbhakam' which means the
“World is One Family” is taken from which of the About Ahom Kingdom
following? • The Ahom kingdom was a late medieval kingdom in the
(a) Maha Upanishad Brahmaputra Valley in Assam. • It is well known for
(b) Aitareya Upanishad maintaining its sovereignty for nearly 600 years and
(c) Mundaka Upanishad successfully resisting Mughal expansion in Northeast
(d) Kausitaki Upanishad India.
• It expanded suddenly under Suhungmung in the 16th
Answer (a) Maha Upanishad century and became multi-ethnic in character, casting a
Explanation: It is in news recently because With people profound effect on the political and social life of the entire
globally feeling "very lonely and frustrated" in the Brahmaputra valley.
confinement of their homes amid the COVID-19 • The kingdom became weaker with the rise of the
pandemic, India Council for Cultural Relations, is Moamoria rebellion, and subsequently fell to repeated
contemplating offering classes on the basics of Indian Burmese invasions of Assam. • With the defeat of the
philosophy as the credo of 'Vasudhaiva Kutumbakam'. Burmese after the First Anglo-Burmese War and the
• The Maha Upanishad is a Sanskrit text and is one of the Treaty of Yandabo in 1826, control of the kingdom passed
minor Upanishads of Hinduism. into East India Company hands.
• The text is classified as a Vaishnava Upanishad.
Who are the Ahoms today?
• The founders of the Ahom kingdom had their own
Q. 5) With reference to Ahom kingdom, Consider the language and followed their own religion.
following statements : • Over the centuries, the Ahoms accepted the Hindu
1. It was a kingdom established by Sukapha in religion and the Assamese language, scholars say.
the Brahmaputra Valley, Assam. • The Ahoms embraced the language, religion and rituals
2. Control of the kingdom passed into East India of the communities living here — they did not impose
Company hands after the Treaty of Yandabo in 1826. theirs on those living here.
Which of the above statement is/are correct
(a) 1 only Why is Sukapha important in Assamese culture?
(b) 2 only •Sukapha’s significance — especially in today’s Assam
(c) Both 1 and 2 — lies in his successful efforts towards the assimilation
(d) Neither 1 nor 2 of different communities and tribes.

80 | P a g e
• He developed very amicable relationships with the tribal (d) All are correct
communities living here — especially the Sutias, the
Morans and the Kacharis. Answer : c) India and Nepal
• Intermarriage also increased assimilation processes. He Explanation : Factors taken into consideration for fixing
is widely referred to as the architect of “Bor Asom” or MSP include:
“greater Assam”. 1) demand and supply;
2) cost of production;
Q.6) Consider the following statements regarding Ozone 3) price trends in the market, both domestic and
layer: international;
1. The ozone layer is mainly found in the lower portion of 4) inter-crop price parity;
the stratosphere. 5) terms of trade between agriculture and non-agriculture;
2. The thickness of the ozone layer is generally thinner 6) a minimum of 50 percent as the margin over cost of
near the equator and thicker near the poles. production; and
7) likely implications of MSP on consumers of that
Choose the correct code given below product.
(a) 1 only Minimum Support Price
(b) 2 only
(c) Both 1 and 2 •The MSP is the rate at which the government buys grains
(d) Neither 1 nor 2 from farmers.
• Reason behind the idea of MSP is to counter price
Answer (c) Both 1 and 2 are correct volatility of agricultural commodities due to the factors
Explanation- It is in news because Largest Ozone Hole like variation in their supply, lack of market integration
Ever Recorded over North Pole Has Now ‘Healed Itself’ and information asymmetry.
and Closed. This was announced by Scientists who were • The MSP is fixed on the recommendations of the
tracking the hole at Copernicus’ Atmospheric Monitoring Commission for Agricultural Costs and Prices (CACP).
Service (CAMS). • The Commission also makes visits to states for on-the-
spot assessment of the various constraints that farmers
About ozone layer face in marketing their produce, or even raising the
• The ozone layer or ozone shield is a region of Earth's productivity levels of their crops.
stratosphere that absorbs most of the Sun's ultraviolet • Based on all these inputs, the Commission then finalizes
radiation. its recommendations, which are then submitted to the
• It contains high concentration of ozone (O3) in relation government.
to other parts of the atmosphere, although still small in • The government, in turn, circulates the CACP reports to
relation to other gases in the stratosphere. state governments and concerned Central Ministries for
• The ozone layer is mainly found in the lower portion of their comments.
the stratosphere, from approximately 15 to 35 kilometers • After receiving the feed-back from them, the Cabinet
(9.3 to 21.7 mi) above Earth, although its thickness varies Committee on Economic Affairs (CCEA) of the Union
seasonally and geographically. government takes a final decision on the level of MSPs
• The thickness of the ozone layer varies worldwide and and other recommendations made by the CACP.
is generally thinner near the equator and thicker near the • The Food Corporation of India (FCI), the nodal central
poles. agency of the Government of India, along with other State
Agencies undertakes procurement of crops.
Q.7) What are the Factors or determinants taken into Crops Covered under MSP
consideration for fixing Minimum Support Price:
1. Demand and supply • Government announces minimum support prices
2. Cost of production (MSPs) for 22 mandated crops and fair and remunerative
3. Likely implications of MSP on consumers of that price (FRP) for sugarcane.
product • The mandated crops are 14 crops of the kharif season, 6
4. Terms of trade between agriculture and non-agriculture Rabi crops and two other commercial crops.
Choose the correct code given below • In addition, the MSPs of toria and de-husked coconut are
(a) 1 and 2 only fixed on the basis of the MSPs of rapeseed/mustard and
(b) 2 and 3 only copra, respectively.
(c) 1, 2 and 3 only • The list of crops are as follows:

81 | P a g e
1)Cereals
2)Pulses
3)Oilseeds
(4)- groundnut, rapeseed/mustard, toria, soya bean,
sunflower seed, sesame, safflower seed and nigerseed.
5)Raw cotton.
(6) - gram, arhar/tur, moong, urad and lentil
7)Raw jute.
8)Copra
9) - paddy, wheat, barley, jowar, bajra, maize and ragi
10)De-husked coconut
11)Sugarcane (Fair and remunerative price)
12)Virginia flu cured (VFC) tobacco

Q.8) Which of the following is not Caribbean country:


(a) Samoa
(b) Cuba
(c) Haiti
(d) Dominica

Answer (a) Samoa 29 JUNE

Q. 1) Which of the following statements is true about


Administration of Museums in India:
1. The National Portal and Digital Repository for Indian
Museums have been launched for digitization of the
collections of the Museums.
2. All Museums are not administered by the Ministry of
Culture.
Choose the correct code given below
(a) 1 only
(b) 2 only
(c) Both 1 and 2
(d) Neither 1 nor 2

Answer (c) Both 1 and 2


Explanation : Recently the Ministry of Culture hosted a
webinar on “Revitalizing Museums and Cultural Spaces”
to celebrate the International Museum Day.
• The National e-Governance Division (NeGD) created by
the Ministry of Electronics & Information Technology
(MeitY) provided technical assistance for conducting the
Q.9) Mona passage is situated between: Webinar.
(a) Cuba and Haiti
(b) Florida and Cuba Administration of Museums in India
(c) Dominican Republic and Puerto Rico • In India multiple ministries hold charges of various
(d) Bahamas and Dominican Republic Museums, not all Museums are administered by the
Ministry of Culture.
Answer (c) Dominican Republic and Puerto Rico • Some are run without government support by a handful
of people under a Board of Trustees.
• The Budget 2020 proposed the setting up of an Indian
Institute of Heritage and Conservation with the status of a

82 | P a g e
deemed university under the Ministry of Culture. 2. Its entire population naturally occurs only in a tiny,
• The National Portal and Digital Repository for Indian evergreen forest pocket in South Andaman’s Mount
Museums (under the Ministry of Culture) have been Harriet National Park.
launched for digitization of the collections of the Which of the above statement is/are correct
Museums. (a) 1 only
(b) 2 only
International Council of Museum (c) Both 1 and 2
•ICOM is the main and only organization of museums and (d) Neither 1 nor 2
museum professionals with global scope, committed to
promotion and protection of natural and cultural heritage, Answer (b) 2 only
present and future, tangible and intangible. Explanation- First Statement is NOT correct because it is
• It was created in 1946 and is headquartered in Paris, a rare palm tree endemic to the South Andaman Island.
France. • Recently, this endemic palm tree found a
• It serves as a network of museum professionals (35,000 second home at Thiruvananthapuram-based Jawaharlal
members in 137 countries) acting in a wide range of Nehru Tropical Botanic Garden and Research Institute
museum- and heritage-related disciplines. (JNTBGRI).
Pinanga Andamanensis
Q. 2) YUKTI 2.0 recently seen in news is an initiative
launched by the : • Name ➔ The name is derived from ‘Penang’, the
(a) Ministry of AYUSH modern-day Malaysian state.
(b) Ministry of Commerce and Industry • Status ➔ Pinanga andamanensis is an IUCN
(c) Ministry of Human Resource Development critically endangered species and one of the
(d) Ministry of Environment, Forests and Climate Change least known among the endemic palms of
the Andaman Islands.
Answer (c) Ministry of Human Resource Development
• Location ➔ Its entire population of some 600 specimens
Explanation-
naturally occurs only in a tiny, evergreen forest pocket in
South Andaman’s Mount Harriet National Park.
YUKTI 2.0
•The initiative was launched by the Ministry of Human • History ➔ It was originally described by the Italian
Resource Development. botanist Odoardo Beccari in 1934.
• It is to help systematically assimilate technologies • His description was based on an old herbarium specimen
having commercial potential and information related to collected by E.H. Man, a late-19th century assistant
incubated startups in higher education institutions. superintendent in the Andaman administration.
• It is an extension of ‘YUKTI - Young India combating • After that first identification, it was thought to be extinct
COVID with Knowledge, Technology and Innovation till 1992.
Portal’. • Uses ➔ Its use are yet to be understood fully. But this
• YUKTI which was launched to identify ideas relevant in palm holds promise as an avenue tree for gardens,
COVID pandemic. pavements and homesteads.
• The Ministry has invited students, faculty members, • Threats ➔ Natural calamities such as cyclones,
startups and other stakeholders of higher education earthquakes and rampaging by wild elephants among
institutions to register on the YUKTI portal and share others.
their technologies and innovations.
• Database shared will be showcased to the investor Q.4) The Senkaku Islands are a group
community such as businessmen, farmers and of uninhabited islands in the:
industrialists i.e. students will be able to sell their ideas. (a) Sea of Japan
• However, preliminary innovations won’t be showcased (b) East China Sea
on YUKTI 2.0 but only those which have (c) Philippine sea
commercialization potential. (d) South China Sea

Q.3) Consider the following statements regarding Answer (b) East China Sea
Pinanga Andamanensis: Explanation: It is in news recently because Japan has
1. It is an IUCN critically endangered species of endemic approved a plan to change the name of the area covering
Mahua tree of the Andaman Islands. the Tokyo-controlled Senkaku Islands — known by

83 | P a g e
Taiwan and China as the Diaoyus — from “Tonoshiro” to
“Tonoshiro Senkaku”. Q. 5) With reference to Social Protection Response
Programme, Consider the following statements :
About SENKAKU ISLANDS 1. Its aim is to support India’s efforts at providing social
• Location ➔ The Senkaku Islands are a group of assistance to the poor and vulnerable households severely
uninhabited islands in the East China Sea. impacted by the COVID- 19 pandemic.
◎ They are located east of Mainland China, northeast of 2. It will be implemented by Ministry of Health and
Taiwan, west of Okinawa Island, and north of the Family Welfare.
southwestern end of the Ryukyu Islands. Which of the above statement is/are correct
• Names ➔ They are also known as the Diaoyu Islands in (a) 1 only
Mainland China, the Diaoyutai Islands in Taiwan and the (b) 2 only
Pinnacle Islands by other observers. (c) Both 1 and 2
(d) Neither 1 nor 2
• Dispute ➔ The islands are the focus of a territorial
dispute between Japan and China and between Japan and
Answer (a) 1 only
Taiwan.
• Administration ➔ Japan administers and controls the Explanation- Second statement is NOT correct because It
Senkaku islands as part of the city of Ishigaki in Okinawa will be implemented by the Ministry of Finance.
Prefecture. • Recently, The World Bank has announced a $1 billion
assistance under
the proposed Accelerating India’s COVID-19 Social
Protection Response Programme.
Social Protection Response Programme

• Aim ➔ To support India’s efforts at providing social


assistance to the poor and vulnerable households severely
impacted by the COVID- 19 pandemic.
◎ Implementation ➔ It will be implemented by the
Ministry of Finance, Government of India.
• Funding ➔ It will be funded in two phases – an
immediate allocation of $750 million for fiscal year 2020
and a $250 million second tranche that will be made
available for fiscal year 2021:
• The first phase will be implemented through the Pradhan
Mantri Garib Kalyan Yojana (PMGKY).
• In the second phase, additional cash and in-kind benefits
based on local needs will be extended through state
governments and portable social protection delivery
systems.

About World Bank


• Background ➔ World Bank is one of five institutions
created at Breton Woods Conference in 1944.
• The World Bank is an international financial institution
that provides loans to developing countries for capital
programs.
• It comprises of two institutions ➔
○ International Bank for Reconstruction and
Development (IBRD)
○ International Development Association (IDA).
• Formation ➔ July 1944.

84 | P a g e
• Headquarters ➔ Washington DC, United States. (c) Both 1 and 2
• Motto ➔ Working for a World Free of Poverty. (d) Neither 1 nor 2
• Membership ➔ 189 countries (IBRD), 173 countries
Answer (a) 1 only
(IDA)
Explanation: Second statement is NOT correct because
• The World Bank is one of the five components of the
Study material for the differently abled developed on
World Bank Group,
Digitally Accessible Information System (DAISY) and in
which is part of the United Nations system.
sign language on NIOS website/ YouTube.
• World Bank Group also includes three more subsidiary
• Manodarpan initiative - It is being launched to provide
organisations viz.
psychosocial support to students, teachers and families for
○ International Finance Corporation (IFC),
mental health and emotional wellbeing through a website,
○ Multilateral Investment Guarantee Agency (MIGA),
a toll-free helpline, national directory of counselors,
and
interactive chat platform, etc. This initiative will benefit
○ International Centre for Settlement of Investment
all school going children in the country, along with their
Disputes (ICSID)
parents, teachers and the community of stakeholders in
school education.
Q.6) Mount Merapi, recently seen making news, is an
active volcano in:
About some initiatives
(a) Italy
• Union Finance Ministry has announced several
(b) India
initiatives to boost Education Sector.
(c) Indonesia
• A comprehensive initiative called PM e-VIDYA will be
(d) Japan
launched which unifies all efforts related to
digital/online/on-air education.
Answer (c) Indonesia
• This will enable multi-mode access to education, and
Explanation- It is in news because, Mount Merapi, an
includes:
active volcano in Indonesia, erupted twice on 21st June,
1) DIKSHA (one nation-one digital platform) - nation’s
2020.
digital infrastructure for providing quality e-content in
school education for all the states/UTs.
About Mt Merapi
2) TV (one class-one channel) where one dedicated
• Merapi (Mountain of Fire) is the most active of
channel per grade for each of the classes 1 to 12 will
Indonesia’s 130 active volcanoes.
provide access to quality educational material.
• It rises to 2,911 metres and has steep slopes with dense
3)SWAYAM online courses in MOOCS format for
vegetation on its lower sides.
school and higher education.
• It is located near the centre of the island of Java and
4) IITPAL for IITJEE/NEET preparation.
Indonesia’s cultural capital, Yogyakarta.
5) Air through Community radio and CBSE Shiksha Vani
• Mt Merapi’s last significant eruption was in 2010. At
podcast.
that time, more than 300 people were killed and some
6) Study material for the differently abled developed on
2,80,000 residents were forced to evacuate the
Digitally Accessible Information System (DAISY) and in
surrounding areas.
sign language on NIOS website/ YouTube.
7)Manodarpan initiative - It is being launched to provide
Q. 7) Which of following is true about the recently
psychosocial support to students, teachers and families for
launched initiatives by the Union Finance Minister:
mental health and emotional wellbeing through a website,
1. DIKSHA is a one nation-one digital platform for
a toll-free helpline, national directory of counselors,
providing quality e-content in school education for all the
interactive chat platform, etc. This initiative will benefit
states/UTs.
all school going children in the country, along with their
2. DAISY is a platform to provide psychosocial support
parents, teachers and the community of stakeholders in
to students, teachers and families for mental health and
school education.
emotional wellbeing through a website or a toll-free
8)Expanding e-learning in higher education – by
helpline.
liberalizing open, distance and online education
regulatory framework. Top 100 universities will start
Choose the correct code given below
online courses. Also, online component in conventional
(a) 1 only
Universities and ODL programmes will also be raised
(b) 2 only
from present 20% to 40%. This will provide enhanced

85 | P a g e
learning opportunities to nearly 7 crore students across ◎ What is it? ➔ It is an inter-governmental organisation
different colleges and Universities. within the OECD (Organisation for Economic Co-
9)National Curriculum and Pedagogical Framework - To operation and Development) system.
promote critical thinking, creative and communication • It is the only global body with a mandate for all modes
skills, along with experiential and joyful learning for the of transport.
students focusing on learning outcomes. ◎ Functions ➔ It acts as a think tank for transport policy
issues and organises the annual global summit of transport
Q.8) Which of the following statement is INCORRECT ministers.
about International Transport Forum:
◎ Member Countries ➔ 60 and India has been a member
1. It is an inter- governmental organization within the
since 2008.
OECD system and the only global body with a mandate
for all modes of transport. ◎ Headquarters ➔ Paris, France.
2. It acts as a think tank for transport policy issues and ◎ The ITF’s motto is “Global dialogue for better
recently India has become its member. transport”.
Choose the correct code given below
(a) 1 only About OECD
(b) 2 only • What is it? ➔ The Organisation for Economic Co-
(c) Both 1 and 2 operation and Development(OECD) is an
(d) Neither 1 nor 2 intergovernmental economic organisation with 36
member countries.
• Formation ➔ It was founded in 1961 to stimulate
Answer : (b) 2 only economic progress and world trade.
Explanation : Second statement is INCORRECT because ◎ Headquarters ➔ Paris, France.
India has been a member of ITF since 2008. ◎ Functions ➔ It works to build better policies for better
• NITI Aayog in collaboration with International lives. Its goal is also to shape policies that foster
Transport Forum (ITF) will launch the “Decarbonising prosperity, equality, opportunity and well- being for all.
Transport in India”(DTI) ◎ India is not a member of OECD but has been a key
economic partner.
About DTI Project
• What is it? ➔ It is a tailor-made transport emissions Q. 9) Mount Elgon is an extinct shield volcano on the
assessment framework for India. border of :
◎ Features ➔ It will provide the government with a (a) Libya and Egypt
detailed understanding of current and future transport (b) Ethiopia and Somalia
activity and the related CO2 emissions as a basis for their (c) Tanzania and Malawi
decision- making. (d) Uganda and Kenya
• The project is a part of the Decarbonising Transport in
Emerging Economies (DTEE). Answer (d) Uganda and Kenya
Explanation :
DTEE • Mount Elgon is an extinct shield volcano on the border
◎ What is it? ➔ It is an International Transport of Uganda and Kenya.
Forum(ITF) project in collaboration with Wuppertal • The mountain's highest point, named "Wagagai", is
Institute(WI) and supported by the International Climate located entirely within Uganda.
Initiative(IKI) of the German Federal Ministry for the • Although there is no verifiable evidence of its earliest
Environment, Nature Conservation and Nuclear volcanic activity, geologists estimate that Mount Elgon is
Safety(BMU). at least 24 million years old, making it the oldest extinct
◎ Aim ➔ To help governments of emerging nations to volcano in East Africa.
identify ways to reduce their transport CO2 emissions and • Mt. Elgon consists of five major peaks:
meet their climate goals.
1)Wagagai in Uganda
• Participating Countries ➔ Argentina, Azerbaijan, India
2)Sudek on the Kenya/Uganda border
and Morocco.
3)Koitobos, a flat-topped basalt column in Kenya
4)Mubiyi in Uganda
International Transport Forum(ITF)

86 | P a g e
5)Masaba in Uganda from the Sahara Desert.
• It stretches around 2,500 km through Morocco, Algeria
and Tunisia.

Q.10) Which of the following is true for Atlas Mountain


range:
1. It separates the Mediterranean and Atlantic coastlines
from the Sahara Desert.
2. The highest peak of the range is Mount Kilimanjaro.
Choose the correct code given below
(a) 1 only
(b) 2 only
30 JUNE
(c) Both 1 and 2
(d) Neither 1 nor 2
Q. 1) Which of the following statements is true about
Answer (a) 1 only National Commission for Backward Classes:
Explanation: 1. It is a statutory body established in 1993 under Ministry
• Second statement is NOT correct because the range's of Social Justice and Empowerment.
highest peak is Toubkal, which is in southwestern 2. 2. It has the authority to examine complaints and
Morocco. welfare measures regarding socially and educationally
• The Atlas Mountains are a mountain range in the backward classes.
Maghreb.
• It separates the Mediterranean and Atlantic coastlines Choose the correct code given below
(a) 1 only

87 | P a g e
(b) 2 only operationalized through:
(c) Both 1 and 2 (a) Atmanirbhar Bharat Abhiyan economic package
(d) Neither 1 nor 2 (b) Credit Guarantee Fund Trust
(c) Small Industries Development Corporations of India
Answer (b) only 2 (SIDBI)
Explanation : First statement is NOT correct because It is (d) National Small Industries Corporation
a constitutional body established in 1993 under Ministry
of Social Justice and Empowerment. Answer (b) Credit Guarantee Fund Trust
• Second statement is correct because NCBC has the Explanation-
authority to examine complaints and welfare measures • Minister of MSME launched the Credit Guarantee
regarding socially and educationally backward classes. Scheme for Sub- ordinate Debt (CGSSD) which is also
• Recently the Union Cabinet has approved a six-month called “Distressed Assets Fund–Sub-ordinate Debt for
extension to the Justice G. Rohini commission constituted MSMEs”.
to complete the task of sub-categorising 5000- odd castes • Aim ➔ To extend support to the promoters of the
in the central OBC list so as to ensure more equitable operational MSMEs which are stressed and have become
distribution of opportunities in central government jobs NPA as on 30th April, 2020.
and educational institutions. ◎ Guarantee ➔ It provides a guarantee cover worth Rs.
• The 102nd Constitution Amendment Act, 2018 provided 20,000 crores to the promoters who can take debt from the
constitutional status to the National Commission for banks to further invest in their stressed MSMEs as equity.
Backward Classes (NCBC), which was previously a
• Features of the Scheme ➔
statutory body.
○ Promoter(s) of the MSMEs will be given credit equal to
• In 2015, the National Commission for Backward Classes
15% of their stake (equity plus debt) or Rs. 75 lakh
(NCBC) had recommended that OBCs should be
whichever is lower.
categorized into extremely backward classes, more
○ Promoter(s) in turn will infuse this amount in the
backward classes and backward classes.
MSME unit as equity and thereby enhance the liquidity
and maintain debt-equity ratio.
National Commission for Backward Classes
○ 90% guarantee coverage for this sub-debt will be given
• It is a constitutional body established in 1993 under
under the Scheme and 10% would come from the
Ministry of Social Justice and Empowerment.
concerned promoters.
• It was constituted pursuant to the provisions of the
National Commission for Backward Classes Act, 1993. ◎ Duration ➔ There will be a moratorium of 7 years on
• The 102nd Constitution Amendment Act, 2018 provided payment of principal whereas maximum tenure for
constitutional status to the National Commission for repayment will be 10 years.
Backward Classes (NCBC), which was previously a ◎ Implementation ➔ The scheme will be operationalised
statutory body. through Credit Guarantee Fund Trust for MSEs
• NCBC has the authority to examine complaints and (CGTMSE).
welfare measures regarding socially and educationally
backward classes. Q.3) Consider the following match:
• In 2015, the National Commission for Backward Classes
(NCBC) had recommended that OBCs should be Waterfalls State
categorized into extremely backward classes, more 1. Athirappally - Karnataka
backward classes and backward classes. Waterfalls
• Commission shall consist of one chairman and four 2. Hundru - Himachal Pradesh
Members: a Chairperson who is or has been a judge of the WaterFalls
Supreme Court or of a High Court; a social scientist; two 3. Sahastradhara - Uttarakhand
persons who have special knowledge in matters relating WaterFalls
to backward classes; and a Member-Secretary, who is or Which of the above pair is/are matched INCORRECTLY:
has been an officer of the Central Government in the rank (a) 1 and 2 only
of a Secretary to the Government of India. (b) 2 and 3 only
• Their term is of Three years (c) 1 and 3 only
(d) 1 only
Q. 2) DISTRESSED ASSETS FUND–SUB-
ORDINATE DEBT FOR MSMEs scheme will be Answer (a) 1 and 2 only

88 | P a g e
Explanation- First pair is matched incorrectly because countries have not supported learners at risk of exclusion
The famous Athirappally during this crisis.
Waterfalls is located on the Chalakudy River ◎ In April 2020, almost 91% of students around the world
in Thrissur district of Kerala. Recently , Kerala were out of school.
government recently gave the go-ahead for the proposed ◎ Poor, linguistic minorities and learners with disabilities
163-megawatt (MW) Athirappally Hydroelectric Project. were affected more as resources for such persons may not
• Second pair is matched incorrectly because Hundru be available outside schools.
Waterfalls is on Suvarnrekha river, Ranchi ,Jharkhand. ◎ For poor students who depend on school for free meals
• Sahastradhara Fall is Located near Dehradun, or even free sanitary napkins, closures have been a major
Uttarakhand which is formed by liquidification of blow.
limestone rocks.
Q. 5) With reference to Animal Husbandry Infrastructure
About Athirappally Waterfalls Development Fund, Consider the following statements :
• Location ➔ The famous Athirappally Waterfalls is 1. The government will provide 4% interest subvention to
located on the Chalakudy River in Thrissur district of eligible beneficiaries from the Aspirational Districts and
Kerala. 3% for beneficiaries from other Districts.
• Significance ➔ It is the largest waterfall in Kerala, 2. The fund had been proposed as part of the Atmanirbhar
which stands tall at 80 feet and is nicknamed “The Bharat to facilitate incentivisation of investments in the
Niagara of India”. establishment of infrastructure
◎ Controversy ➔ Controversy about a state-proposed Which of the above statement is/are correct
hydroelectric dam on the Chalakudy River above the (a) 1 only
waterfalls began in the 1990s and continued through (b) 2 only
2021. (c) Both 1 and 2
(d) Neither 1 nor 2
• Issues with the Hydel project ➔
○ A number of families belonging to the Kadar tribal Answer (c) Both 1 and 2
group are facing displacement here.
Explanation- Cabinet Committee on Economic Affairs
○ The falls and its surroundings are part of a crucial has given its nod to set up a Rs.15,000 crore Animal
biodiversity-rich region coming under the Ecologically Husbandry Infrastructure Development Fund.
Sensitive Zone 1 of the Western Ghats.
◎ Aim ➔ To facilitate incentivisation of investments in
Q.4) Global Education Monitoring Report 2020 is the establishment of infrastructure in the animal
published by: husbandry sector.
(a) Education International ◎ The fund had been proposed as part of the Atmanirbhar
(b) World Economic Forum Bharat.
(c) UNESCO • Beneficiaries ➔ Farmer Producer Organizations
(d) Oxfam (FPOs), MSMEs, Section 8 Companies, Private
Companies and individual entrepreneurs with minimum
Answer (c) UNESCO 10% margin money contribution by them.
Explanation: UNESCO has released a report titled ◎ The balance 90% would be the loan component to be
“Global Education Monitoring Report 2020”. made available by scheduled banks.
About report ◎ Interest Subventions ➔ The government will provide
4% interest subvention to eligible beneficiaries from the
◎ Theme ➔ “Inclusion and education: All means all”. Aspirational Districts and 3% for beneficiaries from other
◎ Aim ➔ To monitor progress towards Sustainable Districts.
Development Goal 4 (SDG 4) on education as well as ◎ Duration ➔ There will be a 2 years moratorium period
other education-related points in the SDG Agenda. for repayment of loan with 6 years repayment period
thereafter.
Key Finding

◎ COVID-19 pandemic has increased inequalities in Q.6) Which of the following are the habitat of Fishing
education systems across the world. cats:
◎ About 40% of low- and lower-middle-income (a) Mangrove forests

89 | P a g e
(b) Foothills of the Himalayas 1. It is a low-cost, paper-strip test which can detect the
(c) Western Ghats new coronavirus within an hour.
(d) All of the above 2. It is developed at the Council of Scientific & Industrial
Research — Institute of Genomics and Integrative
Biology (CSIR-IGIB).
Answer (d) All of the above Choose the correct code given below
Explanation- It is in news because, The Odisha (a) 1 only
Government has started a two-year conservation project (b) 2 only
for Fishing Cats in Bhitarkanika National Park. (c) Both 1 and 2
About Fishing Cats (d) Neither 1 nor 2

◎ Scientific Name ➔ Prionailurus viverrinus. Answer (c) Both 1 and 2 are correct
◎ Habitat ➔ In India, fishing cats are mainly found in the
mangrove forests of the Sundarbans, on the foothills of What is Feluda?
the Himalayas along the Ganga and Brahmaputra river
valleys and in the Western Ghats. • It is a low-cost, paper-strip test which can detect the new
coronavirus within an hour.
• Description ➔
• Developed by Scientists at the Council of Scientific &
○ It is twice the size of a house cat.
Industrial Research — Institute of Genomics and
○ The fishing cat is nocturnal (active at night) and apart
Integrative Biology (CSIR-IGIB).
from fish also preys on frogs, crustaceans, snakes, birds,
• It is expected to cost around Rs 500 against the RT-PCR
and scavenges on carcasses
test that costs Rs 4,500 in private labs.
of larger animals.
• It uses cutting-edge gene-editing tool Crispr-Cas9
◎ Protection Status➔ system.
○ IUCN Red List: Vulnerable.
○ CITES: Appendix II Q.8) Which of the following statement is correct about
○ Indian Wildlife Protection Act, 1972: Schedule I Konark Temple:
◎ Threats ➔ Habitat Destruction, Shrimp Farming, 1. The temple is designed in the shape of a colossal chariot
Hunting, Ritual Practices, and is dedicated to the sun God.
Poaching, Poisoning. 2. The temple is believed to be built by the kings from the
•Conservation Efforts ➔ Somavamsi dynasty, with later additions from the Ganga
○ The Fishing Cat Project, launched in 2010 started rulers.
raising awareness about the Cat in West Bengal.
○ In 2012, the West Bengal government officially Choose the correct code given below
declared the Fishing Cat as the State Animal and the (a) 1 only
Calcutta Zoo has two big enclosures dedicated to them. (b) 2 only
○ In Odisha, many NGOs and wildlife conservation (c) Both 1 and 2
Societies are involved in Fishing Cat research and (d) Neither 1 nor 2
conservation work.

-In India, its presence has been documented in: Answer : (a) 1 only
Explanation : Second statement is INCORRECT because
•Ranthambhore Tiger Reserve, in Dudhwa and Valmiki Konark was built by King Narasimhadeva I of the Ganga
Tiger Reserves, in Sur Sarovar Bird Sanctuary, Dynasty (1238-1264AD) in the 13th century and is
• outside protected areas in West Bengal, in Lothian located in Eastern Odisha near the sacred city of Puri.
Island Wildlife Sanctuary in the Sundarbans, • Recently, Union government has launched a scheme for
• in Odisha's Bhitarkanika Wildlife Sanctuary and coastal 100% solarisation of Konark sun temple and Konark town
districts outside protected areas, in Odisha named as Mission Surya Nagari
• in Andhra Pradesh's Coringa Wildlife Sanctuary,
Krishna Wildlife Sanctuary and adjoining reserve forests. About Mision Surya Nagari
• The scheme will meet all energy requirements of Konark
town with solar energy, which is made under PM-vision
Q. 7) Consider the following statements about Feluda: to develop the historical Sun temple town of Konark in

90 | P a g e
Odisha as ‘Surya Nagri’.
• This aims to convey a message of synergy between the
modern use of solar energy and the ancient Sun Temple
and the importance of promoting solar energy.
• It envisages setting up of the 10-MW grid connected
solar project and various solar off-grid applications such
as solar trees, solar drinking water kiosks and off-grid
solar power plants with battery storage.
• It will receive 100% Central Financial assistance (CFA)
support of around Rs. 25 crore through the Ministry of
New and Renewable Energy.
• Odisha Renewable Energy Development Agency will be
the Implementation Agency of the project (and not union
Ministry of Renewable energy or Ministry of culture)

Konark Temple
•Konark was built by King Narasimhadeva I of the Ganga
Dynasty (1238-1264AD) in the 13th century and is
located in Eastern Odisha near the sacred city of Puri.
•The temple is designed in the shape of a colossal chariot.
It is dedicated to the sun God.
• There are two rows of 12 wheels on each side of the
Konark sun temple.
• The seven horses are said to symbolize the seven days
of the week.
• The temple was used as a navigational point by
European sailors. They referred to it as the ‘Black
Pagoda’ due to its dark colour and its magnetic power that
drew ships into the shore and caused shipwrecks.
• It is the culmination of Odisha temple architecture.
• It was declared a UNESCO world heritage site in 1984.

Q. 9) Which African river crosses Equator twice :


(a) Nile River
(b) Congo River Q.10) Arrange the following Rivers in the direction from
(c) Zambezi River West to EAST:
(d) Niger River 1. Senegal River
2. Limpopo River
Answer (b) Congo River 3. Volta River
Explanation : 4. Zambezi River
Choose the correct code given below
• The Congo River, formerly known as the Zaire River (a) 1,4,3,2
during the dictatorship of Mobutu Sese Seko, is the (b) 1,3,4,2
second longest river in Africa, shorter only than the Nile, (c) 4,2,1,3
as well as the second largest river in the world by (d) 3,1,4,2
discharge volume, following only the Amazon.
• It is the only major river to cross the equator twice. Answer (b) 1,3,4,2
Explanation:

91 | P a g e
92 | P a g e

You might also like